solucionario mortimer 5 ed.pdf

96
ji.;ljI' l. ,. l, ANSWER BOOK FOR MORTIMER'S " F T F T H T I'"O I\ ,,,1i: ,,r; : ' ,:l' r ,.i, i: , 'rl: '' Illi: Í', . Lawrenc" M. Epstein

Upload: mvhernan

Post on 13-Apr-2015

459 views

Category:

Documents


34 download

TRANSCRIPT

ji.;ljI'l. ,. l,

ANSWER BOOK FOR MORTIMER'S "

F T F T H T I ' "O I \

,,,1i: ,,r; :'

, : l ' r

,.i, i: ,;' r l : ' 'I l l i :

Í',. :

Lawrenc" M. Epstein

F'

ChapterchapterChapterChapterChapterChapterChapterCl. rpterChapterChapterChapterChapterChapterChapterChapterChapterChapterChapterChapter

123

q,

6

B9

l o11I21 3I4t516L71B1 9

Chapter 20

Chapter 21

Chapter 22

Chapter 23Chapter 24Chapter 25Chapter 26Chapter 27

Contents

IntroductionStoichiometryThermochemistryAtomic StructureProperties of Atoms and the IonicThe Covalent Bondlutolecular Geometry; Molecular OrbitalsGasesLiduids and So1idsSolut ionsReactions in Aqueous SolutionChemical KineticsChemical EguilibriumTheories of Acids and BasesIonic Equil ibr ium, Part IIonic Equil ibr ium, Part I lElements of Chemi-cal ThermodynamicsElectrochemi stryThe Nonmetals, Part I :

the HalogensThe Norunetals, Part I I :

ElementsThe Nonmetals, Part I I I :

Elements

Hydrogen and

The Nonmetals, Part IV: Carbon,Sil icon, Boron, and the Noble Gases

I{etals and Metal lurgyComplex CompoundsNuclear ChemistryOrganic ChemistryBiochemistry

The Group VI A

The Group V A

Bond

I3ti

l 1t7243142496L667 480

90939599

107LL2

119

L 2 71 3 1T4L150155l-73

@ lgg¡ by Wadswor th , Inc . A l l r igh ts reserved. No par t o fthis book may be reproduced, stored in a retr ieval system,or transcribed, in any form or by any means, elecbronic,mechanical, photocopylng, recording, or otherwise withoutthe prior permission of the publisher, Wadsworth publ ishing

Company, Belmont, Cali fornia 94OO2, a Division of Wadsworth,

I S B N 0 - 5 3 r { - 0 e ? e 1 - b

Printed in the United States of America

L 2 3 4 5 6 ' t B 9 1 0 - - - 8 7 8 6 8 5 8 4 8 3

r ' l l / \ [ ' T E R I

I NI ' I ' I iODUCTION

l . i ¡

t l

t . 2

l . ( )

| . t

| . r t

| . , )

¡ . I o

| . r , L

| " t . ¿

l . t l ,

l " l / l

I l ' r

S e e : ( a ) S e c t i o n s 1 . 1 a n d 1 . 2 , ( b ) S e c t i o n L . 2 ,( c ) S e c t i o n L . 2 , ( d ) S e c t i o n I . 2 , ( e ) S e c t i o n 1 . 1

(a) strontium, (b) antimony, (c) aluminum, (d) gold,(e ) s i l ver , ( f ) s i l i con , (g ) mercury , (h ) he l ium,( i ) sod iüm, ( j ) neon, (k ) ca lc ium, (1 ) cadnr ium

( a ) S n , ( b ) T i , ( c ) P , ( d ) K , ( e ) C u , ( f ) C o , ( 9 ) F e ,( h ) I , ( i ) C l , ( j ) c r , ( k ) M g , ( l ) M n , ( m ) L i ,( n ) P b

( a ) 4 , ( b ) I ? , ( c ) I , ( d ) 5 , ( e ) 4 , ( f ) 1 ? , ( g ) 4 ,( h ) 4

( a ) 1 3 7 . 0 , ( b ) 1 0 . 0 0 , ( c ) 0 . 9 0 0 , ( d ) 5 . 0 , ( e ) ] - I 2 ,( f ) 0 . 0 0 2 1 0

- a R - q

( a ) 5 . 9 4 x I 0 ' ,

( b ) 6 . 2 5 x l O - , 3 . 0 x l - O - ,

( d ) O . 0 9 6 , ( e ) 1 3 . 6 , ( f ) 2 . 5 x l O 2

( a ) l O s c m , ( b ) 1 0 - 6 k g , ( c ) 1 0 7 n s , ( d ) 1 O - r 6

r t n

( a ) I o 3 l j - t e r , ( b ) 1 o - 3 m 3

( a ) 1 O - 1 n m , ( b ) 1 0 2 p m , ( c ) 0 . 0 9 9 n m , 9 9 p m

1 . 5 m

l - . 2 1 k m

2 . 8 1 x l o 3 h r , 1 . 1 7 x 1 0 2 d a y

0 . 9 5 4 m 3

9 . 5 9 5 t r i p

236.6 ml sugar , 59 .2 m1 but te r , 394.3 m1 f lour , and1 I 8 . 3 m l m i l k , 7 . 3 9 m l b a k i n g p o w d e r , 4 . 9 3 m I v a n j - 1 1 a ,l - . 2 3 m I s a l t

(a ) 9s" inc rease ' (b ) 6% increase ' (c ) l0g" inc rease

( a ) 5 B % , ( b ) 2 2 K

( a ) 2 o o . 9 P L , ( b ) r ' 2 5 x 1 0 3 g a l l o Y

I . 0 5 x l O 3 g Z n

- 1

4 . 0 x l 0 - a

2OOO g a l loy; 3OO g Cu lef t over ' 60 g Ni le f t over

5L.445 cm/s

A9 km/hr

24.7 hr

r ' l l / \ l " f E R 2

J; , I '0 I CI l IOMETRY

l , , r l _ t g n J = T h e o r y , A t o m i c W

, l . l . See Sect i -on 2 . I .

) . . 2 . S e e S e c f i o n 2 - I .

, l . l A re la t i ve a tomic we igh t i s ac tua l l y a ra t io : theaverage mass of the atoms of an element compared tothe mass of an atom of a standard.

S ince CH+ is 75 .0e" by mass carbon and 25 .0% by masshydrogen, the compound consists of carbon to hydrogenin a mass ra t io o f 3 to 1 . One C a tom, however , i scombined with four H atoms i i-Eu+. One C atom,therefore, must have a mass that is 3 t imes the massof fo_ur H atoms. I f the H atom is assigned a mass of1 .OO, four H a toms wou ld have a mass o f 4 .0O. There la t i ve mass o f the C a tom wou ld be 3 t imes 4 .OO,o r 1 2 . 0 0 .

S e e T a b l e 2 . 1 .

' l ' 1 , , , @

. l . ( ¡ (a ) 24 .7 mol - Hz , L .4g x IO25 molecu les H2

( b ) 2 . 7 8 m o l H z O t L . 6 7 x 1 0 2 + m o l e c u l e s H 2 O

( c ) 0 . 5 1 0 m o l H 2 S O a | 3 . O 7 x 1 0 2 3 m o l e c u l e s H 2 S O a

I . 1 6

1 1 ?

I . I B

1 1 0

* l . 2 1

1 t ?

1 1 A

L . Z 2

L . Z O

1 a 1

1 , Q

6 . 3 8 x 1 0 6 m

pressure is

diamond.

1 t o

r 1 . 3 0

k l . 3 l

k L . 3 2

t I . J J

( a ) 4 6 4 - 5 m / s , ( b ) 4 ' 6 1 x l O 7 m ' ( c )

- 7( a ) f . 3 1 x 1 0

' m ' ( b ) O ' I 3 1 ] r n

O . O 5 7 O c m 3

( a ) 2 . 8 5 c m 3 r ( b ) 4 ' 5 0 c m 3 r n t e n s e

t.qoit .a to transform graphite anto

6 . 4 x 1 0 1 2 g A u

1 . 0 3 x 1 0 3 c m

(a) o . B58 9 , /cm3 , (b ) f loa t

3 . 3 8 0 M m

4 . a 2 2 x L o 2 7 g

t {

( a ) 2 . 9 8 x

( c ) 2 . 1 5 x

4 . 4 8 0 x t _ 0

The atomic

( a ) 4 . 6 1 3 5

( b ) 2 . 7 7 8 3

102 s a toms, (b )

fo24 a toms

g A I

w e i g h t i s 5 8 . 9 3

m o 1 P t , O . 5 2 O 2 4

x 1024 a toms Pt ,

5 . 0 1 x 1 0 2 4 a t o m s ,

( the e ]ement i s Co)

mol Ir

3 . \ 3 2 g x l - 0 2 3 a t o m s I r

rl-

2.LL (a ) 0 .14393 mol Au ' 8 '6676 x 1022 a toms Au

(b) 1 .3335 x 1020 a toms Au

l_.29 l ! = 3; the formula is CuSOr¡ . 3H2O

* 2 . 2 9 V C l 3

l'-r.lrcentage Composition

:¿ . 30 34 .42 N i in N i (co) q' ) . .3L

69 .592 Ba in BaCO3t

2 .32 49 .762 Z r i n Z rS iOa

;¿ .33 629 g Zn

' 2 . 3 4 2 . 0 0 k g C u

i .¿_. l l_ 0.6334 9 Xe¡ 0.3666 s F

, t . 3 6 2 . 9 A 9 9 L i , 2 . 0 1 1 9 N

2 - 3 7 A 2 . 8 % C , 1 7 . 4 e " H

* 2 . : 1 9 8 3 . 9 e " c , l 2 . O B H , 4 . l z o

r , ) . . 39 a4 .64% Fe2o3 i n o re

* : 1 . 4 0 1 . 5 5 4 S i n o i l

r'!!.rcmlgaI *Equations

').-!! (a) VzOs + 2Hz + VzOs + 2H2O

(b) 282iu^3 + 7C -> BaC + 6CO

(c) 4Bi + 3O2 -> 2Bizos

(d) CaCz + 2H2O + Ca (OH) 2 * H2C2

(e ) Ba (NOg)z '+ HzSO+ + BaSOq + 2HNO3

. ' . .42 (a) 3NO2 + H2O + 2HNO3 + NO

(b) Al2S3 + 6H2O -> 2AL (OH) 3 + 3H2S

(c) 3SiC1a + Si -> 2Si2C15

(d) (NH'+)zCreOz + 4HzO * Cr2O3

(e) Ca3N2 + 6HeO + 3Ca(OH) 'z + 2NHg

They would extend 6'022 x 1018 km' which is over

4 x 1O1o t imes the distance'

*2.L3 2.22 aüoms Cv

Formulas

2 . L 4 ( a ) H g B s S o ¡(e ) C+Ha

2 .L5 (a ) CoS ' (b )(e ) PsNsCI r o

(b ) NazSzo '+ ,

B z o H r e r ( c )

( c ) V r S + r (d ) NaePeOz '+ '

S + N 2 , ( d ) N e S e F s I

2.L6 f, ien'*

2 . 1 7 C 1 ¡ H 1 2 O N

2 . I B C 2 H 6 N

2-Lg ca5P3013H or cas (Por+) g (oH)

2 . 2 0 C e H s O a

2 . 2 L C e H l ¡ N O a

2 . 2 2 C e O z H s

2 . 2 3 C 7 H 5 0 3 S N

2 . 2 4 ( a ) O . 2 l O m o l C ' 0 ' 3 5 0 m o l H ' ( b ) C s H s r

(c ) 2 '87 g CsH5

2 . 2 5 ( a ) 0 . 3 6 o m o l c ' 0 ' 3 6 0 m o l H ' 0 ' 0 9 0 0 m o l S-

( b ) C r + H + S , G ) 7 ' 5 7 g C + H + S

t c2 .26 (a ) 0 .389 mo l C ' 0 ' 389( b ) 4 . 6 7 I c ' 0 ' 3 9 3 I( c ) L ' 7 7 g , ( d ) 0 ' 1 1 1

2.27 x = 6; the formula is

m o l H , 0 . 0 5 5 5 m o l N

H , 0 . 7 7 7 g N

mol O, (e ) CzHzNOz

CoCl2 ' 6H2O

4

t 2 . ! 2

2.43 (a) 2CsHr s + 2502 + l6COz + IBHzO

(b) CzHeO + 3Oz -> 2COz + 3Hzo

Note that the combustion of ethyt alcohol requires

less oxvge"';t;-;; i" of fuel thán the combustion of

oclane does'

Problems Based on Chemical Equations

2 . 4 4 1 3 8 . 1 I H 3 P o a

2 . 4 5 ( a ) 6 - 0 0 g N a N H 2 ' 3 ' 3 8 I N 2 o ; ( b ) 1 ' 3 1 I N H 3

2 . 4 6 5 . 3 2 g K N o 2 ¡ 2 ' L L g K N O 3 ¡ 3 ' L 7 q C r z o g

2 . 4 7 3 . 2 6 g H r

2 . 4 A ( a ) 2 - O 7 ' l g A s , ( b ) f 5 ' 5 % A s a o 5 ' ( c ) 1 1 ' 7 e " A s

2 . 4 9 7 3 . 7 2 N a 2 S O 3

2 . 5 o 4 - 9 9 g N H 4 s c N

2 . 5 L 1 . O B g F 2

2 . 5 2 L . L O 7 9 B z H o

2 . 5 3 1 . 2 9 I S F ¡ r

2 . 5 4 4 . 3 3 g o P ( N H e ) e ¡ B 0 ' 8 %

2 . 5 5 ( a ) 0 . 2 9 5 6 9 T i ' ( b ) 3 ' B O 7 g T i C l 3 ' ( c ) 7 8 ' 8 0 % v i e l d

2 . 5 6 ( a ) 1 ' 9 4 $ N a N 3 ' ( b ) 6 1 ' 9 % Y i e l d

r '2 .57 45 .0% Bao in mix tu re

*2 .58 62-Os" CaCO3 in rn -Lx ture

* 2 . 5 9 L 5 - 2 % C a C O 3

Reactions in Solut ion

2.60 22L g H2Soa

. l . 6 1 1 0 7 . O g K I O 3

. , . . 6 2 1 5 . 0 g N a O H

, ' . . 6 3 1 6 . 0 m l H 3 P O a s o l t n

' . ' . . 6 4 7 5 . 0 m l A g N O 3 s o l ' n

. l .65 5. O0 ml KI4nO¡+ so1 'n

. ) . . 6 6 0 . 8 4 2 g C a O

: . 6 7 2 . 8 6 g 1 2

. f . 6 8 ( a ) O . 4 6 4 9 . N a 2 C O 3 , (b ) 37 .Le" Na2CO3

CHAPTER 3

TIIERMOCHEMISTRY

l .

r {

r 0

_12

- 1 0 8 1 . 6 k J

- 7 L . 4 k J

3 . 3 1 - 2 7 2 7 . 8 k J

, ' 3 . 3 3 - 6 2 2 k J

Heat Measurements, Calorimetry

3 . 1 3 7 " C

3 . 3 - 4 0 0 c

3 . 5 L . 3 6 k J / o C

3 . 7 1 3 9 k J

3 .9 L44 J

3 . 1 1 2 6 . 7 4 0 C

1 . 1 3 8 7 3 k J

3 . 1 5 2 . 2 5 k J / o c

3 . 1 8

3 . 1 9

3 . 2 0

3 . 2 2

L 9 . 4 2 k J

+ 7 6 . 3 k J

La\^/ of Hess

3 . 2 4 + 5 0 . 0 k J

3 . 2 6 - 3 0 0 . 1 k J

3 . 2 8 - 1 3 7 6 . O k J

- L 7 . 7 8 " C

1 8 . 8 k J

2 . 4 6 J / ( 9 " c )

2 3 - 4 7 " C

9 2 . 2 g

2 . 7 5 x r o 3 k J

0 .907 g CtzÉzzOt t

Thermochemj-cal Equations

3.17 (a) endothermic, (b) exothermic' (c) endothermic'

(d) exothermic

1 Ec6H6 (1 ) *

7o , (g )+6COz (g )

czHsoH (1) + 3Oz (g)+2coz (S)

l , ; rr_f halp_iss of F_ormation

f - .34 (a ) Ag( " ) + +c lz

(g ) + Aec l (s ) AS = - r27 k r, l

( b I J N z

( g ) + o z ( s ) + N o z ( g ) A + = + 3 3 . 8 k J

(c ) ca (s ) + . c ( s raph i t e ) n 1o1 ' | ¡ - l

+ caco3 ( s )

A+ = -L2O6 .e kJ

1 r1 .35 (a )

TuzG) + c (g raph i re ) +

i * r t g l + HcN(g )

A H : = + 1 3 0 . 5 k J--{

c ( s raph i t e ) + 25 (s ) -> cs2 ( f ) A+ = +87 .86

?N z ( g ) + 2 H z G ) *

i o r ( S l + N H q N o s ( s )

A H : = - 3 6 5 . 1 k J"'t

J - , 1 9 - 1 1 2 5 . 2 k J 3 . 3 7 + 9 6 . B k J

t . 3 B - I 2 I 2 . 3 k J 3 . 3 9 - 8 4 7 k J

l : i 9 ( a ) C H 3 o H ( 1 ) + ; o r ( s ) + c o 2 ( q ) + 2 H z o ( r )

( b ) - 7 6 4 . L k J

r . 4 I ( a ) c 5 H 5 ( 1 ) r + o r ( g )

+ 6 c o z ( g ) + 3 H 2 o ( l )

( b ) - 3 2 6 7 . 7 k J

t - . 4 2 ( a ) N 2 H a ( 1 ) + 0 2 ( g ) + N z ( g ) + 2 H z o ( L )

(b ) +50.6 kJ /mol -

\ . 4 3 ( a ) c o ( N H e ) e t s l + * o z ( s )

- ' c o 2 ( g ) + N z ( s ) + 2 H 2 o ( 1 )

(b) -333 kJ,/mo1

1 . 4 4 - 1 6 3 . 0 k J , / m o l 3 . 4 5 - 3 5 I . 5 k J l m o l

t . 4 6 - 6 2 . 8 k J , / m o l

3 . 2

3 . 4

3 . 6

3 . 8

3 . 1 0

3 . L 2

3 . 1 4

3 . 1 6

( b )

( c )

KJ+

f

3H2o( l ) Ag = - ¡Z6a

3Hzo(1 ) AH = -1368

6 9 . r k J

383 kJ , 45 r kJ

+ 8 1 . 5 k J

- 1 4 9 . 9 k J

- 3 1 9 . 5 k J

3 . ¿ L

3 . 2 3

5 . ¿ )

3 . 2 7

3 . ¿ Y

Bond Energres

3 .47 -92 kJ i i n Tab le

3 .48 +96 kJ

3 . 5 0 - 2 0 6 k J

3 . 5 2 1 5 3 k J

3 .54 -23 kJ

3 . 5 6 - 3 3 0 k J

3 . 5 8 ( a ) - 1 5 0 k J

3 . 5 9 ( a ) - L 9 2 4 k J

AH: o f Hc l (g ) i s---f

3 . 4 9 + 1 1 2 k J

3 . 5 1 + 1 3 2 k J

3 . 5 3 - 1 2 1 k J

3 . 5 5 - 1 2 0 k J

3 . 5 7 - 1 2 8 8 k J

(b ) -1s9 kJ

(b) -1915 kJ

*92 .3O kJ lmo l .J . I ¡

r ' l l A l ' T E R 4

A'I'0MIC STRUCTURE

Nu_

4.L Both charge and mass determine the degree ofde)f l-ect ion of an electron from a straight- l ine pathin an electr ic or magnetj-c f ield. I t is impossibleto investigate either factor separately usingThomsonrs_method.

4 . 2 ( a ) H - , l i g h t e r m a s s

(b) l¡e2+, higher charge

* 4 . 3 ( a ) I . 8 2 x l 0 r + g , / c m 3

( b ) I - 3 2 x 1 O l 8 g

4 . 4 S e e T a b l e 4 . 2

4.5 Since Z = 29 for Cu and Z = 79 for Au, the posit ivecharge-of the Au nucleus is higher than the posit ivecharge of the Cu nucl-eus. Consequently, more wide-angle deflections \^rere observed r^rhen the Au foi-l wasused.

r 4 _ : 9 2 . 7 c m

n 4 : l 2 . 2 x l o - r 2 a

4 . 8 ( a ) 5 6 e l e c t r o n s ;

, n o( b )

- : : B it J J

4 . 9 ( a ) 7 8 e l e c t r o n s ;

A¿,(b ) I ^zn

J U

nuc l -eus : 56 protons, 82 neutrons

nucleus: 78 protons, 1I7 neutrons

10 11

t 5

2 2

3 7

80

5 8

26

35

l5

22

3 7

80

g23

36

I6

26

!?L 2 2

ó z

_J U

44

iir

II

4.LO Symbol

P

Ti

Rb

H9

U E

- 3 +H ' é

"t-

4.11 Sl¡mbol

K

Mn

Zr

Pb

Xe2 -

q Á

cd2+

Z A

15 31

22 4A

3 7 8 5

80 202

58 140

26 56

3s 12

Z A

19 4L

25 55

40 90

a2 204

54 L32

3 4 q

48 1r4

ElectronsProtons Neutrons

Protons Neutrons

19 22

25 30

40 50

a2 L26

5 4 7 8

34 46

4A 66

(a) infrared radiat ion,

(c) microwaves

4 . 2 2 1 . 1 3 x l o l s / s

4 .23 (a )

(d )

4 . 2 4

4 . 2 5

4 . 2 9 x L o r a / s , 2 . 8 4 x l o - r e J

7 . 5 0 x L o L + / s , 4 . g 7 x 1 o - r e J

l l lectromagnetic R

4 . 2 1 , (b ) b lue l igh t ,

4 .32 1875 nm

Electrons

I 9

2!

*54

36

46

( a ) 3 . o o x r o 2 o / s , L . 9 9 x 1 o - r 3 J

( b ) 3 . o o ' x l o l L / s , l . g g x L o - 2 2 J

( a ) 3 3 3 m , 5 . 9 7 x 1 0 - 2 8 , l

( b ) 2 5 0 n m , 7 . 9 6 x 1 0 - 1 6 J

4-26 251 photons

4 . 2 7 4 3 s

*4 .28 L . lA x : . o r s , / s , 263 nm

jiliI

I

4 . L 3

Isotopes' Atonic Weights

( a ) A , E ; B , D ; C , F , ( b ) A , B i C r D ; E , F ;

,^ , , 54.^ 54n- 5or , 50.- _ 53"o _ 53v(c )

26 r ' e , 24u t t 23u ' 24 - t ' 26 ' - ' 23 '

* 4 . 2 9 ( a )

(b )

( c )

^!9mag_gpegtra

4 . 3 1 9 7 . 2 4 n m

4 . 3 3 n = 6 . \ = 2

- l q

4 . 9 7 x 1 0 - -

J- l q

3 . 5 9 x 1 0 - -

J

5 .41 x Lo r+ 7s ¡ 555 nm

4.L4 The unit based or, 12",

the current standard'

4.Ls oo.tszsr\v and o.zstl lv

4.16 oz.otzLlTuxe and :z-soalf lne

4. t i s2.5%1Lí and z.sa lu

4 . 1 8 6 9 . 7 2 u

4 . I 9 2 0 . 1 8 u

4 . 3 4

* 4 . 3 5

* 4 - 3 6

(a) u l t rav io let , v is ibte, v is ib le

(b) n = 2 '+ a = f ¡ rr = 4 + g = 2, !_ = 3 + n = 2

2.279 Um corresponds to the transition n = @ to n = 5.

7.459 Um corresponds to the transition n = 6 to n = 5.

2 . L ' : . g x 1 0 - 1 8 J(a ) 3 .289 x LOrs / s , ( b )

(c) 1.3: -2 x 103 k. r /mol

L2 I3

r r 4 . 3 7

* 4 . 3 8

( a )

( c )

( a ) K = ( 3 . 2 8 g x L O t s / s ) z - 2 - , ( b ) 1 ' 3 1 6 x 1 0 1 6 ' / s

4 .50 (a ) 18 (en t i re n = 3 she l l - ) , (b ) imposs ib le (when

a = 3 , 1 c a n n o t e q u a l 3 ) , ( c ) I 0 ( 3 d s u b s h e l l ) ,( d ) 2 ( a 3 d o r b i t a l ) , ( e ) 2 ( 3 s s u b s h e l t ) ,( f ) imposs ib le (when I = 0 , ml cannot equa l +2) .( g ) 6 ( 3 p s u b s h e l l ) .

4 . 5 1 ( a ) 2 ( 4 s s u b s h e l l ) . ( b ) i m p o s s i b l e ( w h e n I = 0 ,m 1 c d o n o t e q u a l + 3 ) , ( c ) 2 ( a 4 f o r b i t a l ) ,(É) 10 (4d subshe l l ) , (e ) imposs ib le (when n = 4 ,I c a n n o t e q u a l 4 ) , ( f ) 1 4 ( 4 f s u b s h e l l _ ) , ( S ) 3 2 ( t h ee n t i r e n = 4 s h e l l ) .

2 . g 6 L x 1 0 1 6 / s , ( b ) 1 O . B O n m ,

7 2 . 9 3 n m , 5 4 . 0 2 n m

Periodic LlLw

4 . 3 9 S e e S e c t i o n 4 . 1 0

4.4O Mende leevperiodicMoseleYperiodic

4 .41 X rays a retransitionsn = 1 o r

* 4 . 4 2 z = 2 a - o , 2 B N i

* 4 . 4 3 0 . 2 2 7 r w

ouantum Numbers

stated that the propert ies of elements are

functions of lncreasing atomic weight '

showed that the properties of elements are

functions of atomic number'

bel ieved to be produced by electron

to levels deep within the atom (to the

n = 2 l e v e l ) . S e e S e c t i o n 4 ' 1 0 '

4 . 5 2 ( a ) 9 ,

4 . 5 3 ( a ) 2 4 ,

( b ) 1 7 , ( c ) 8

( b ) 1 2 , ( c ) 4

4 .54 l s 2s 2p 3s 3p 3d 4s

26-" 1' 1L 1L 1' 1t lt 1t 1L 11, 1t 1 1 1 1 1'

I

i

I

i l

rliI

i i

I i l -ectronic Confiqurations

r"' zz' zg' :g' ¡t' 396 4='

4 . 5 5 S e e S e c t i o n 4 . 1 3

4 . 4 4

4 . 4 5

* 4 . 4 6

4 . 4 7

4 . 4 8

4 . 4 9

See Sect ion 4- I2

See Tab le 4 -4

( a ) 0 . O 2 4 3 r : m . , ( b ) 3 . 5 1

S e e F i g u r e s 4 . 1 5 , 4 . L 7 t

S e e S e c t i o n 4 . 1 1

m-.-sL 1 / )

- 1 / )

+ 1 / ,

-L/2

+ 1 / )

4.:9_ (a) rzcJ- t (b) 2 5MN,

4 . 5 7 ( a ) s + X e , ( b ) + B C d r

4 . 5 8 ¡ T C L t 2 5 M n ¡ 6 3 E u r 7 7 T r

can be checked by re fe r r ing to Tab le 4 .8 .

( c ) 6 , ( d , 2 , ( e ) 1 , ( f ) 0

are paramagnetic

can be checked by re fe r r ing to Tab le 4 .8 .

( c ) 2 o c a ,

( c ) 6 3 E U r

( d ) 3 o Z n l

( d ) 7 t T r ,

(e ) 3 5Kr

( e ) g s B a- 3 6

x 1 0 m

and 4 . IB!.32

!.So

4 . 6 I

The notations

( a ) 0 , ( b ) 2 ,

b , c , d , a n d e

The notat ions

- 1 m '

l l n

Z L

a | + l

2 L 0. I - ' lZ L

n l m l

1 0 0

1 0 0

2 0 0

2 0 0

r ' l +1

m

+' l / )

+- l / )

- - t /2

- 1 / )

4 . 6 2 ( a ) 2 t ( b ) 4 , ( c ) 3 , ( d ) 4 , ( e ) 0 , ( f ) 0 , ( g ) 2

&, b , c , d , and g are paramagnet ic

L L4 15

4.63 (a) 4zAgrz rs2 2s2 z{ z t 3d1o 4s2 4d6

: " ' ¡gu 3q l o 4s2 496

5 d l o 6 s 2

( c ) z r sc3+ , l s2 zZ ' zg .u ¡g t 3q6

CTIAPTER 5

I)ROPERTIES OF ATOMS AND THE IONIC BOND

Atomic Radii

5.1 !{ i thin a giroup the atomic size increases withincreasing atomic number because the valence shell¡5rincipal quantum number n, increases. Recall thatin the Bohr atom, x = n2 x .059 nm. Ba ) Sr forexample. Within a period the atomic size for themain groüp elements decreases with increasing atomicnunber because the valence shell quanturn number isconstant whilst the effect ive nuclear charge increasesand draws the electrons closer in. p < Si.

5.2 Within a period, the main group metals are larger thanthe non-metals, as explained in 5.1 above. Howeverthe transit ion metals are smaller, because after thed shel l is f i l led the nonmetal-s add to the nextquantum level.

5.3 The init ial effect of entering the transit ion seriesis'a shrinl<age because of the increase in nuclearcharge, while obtaining minimal shielding. The higherthe I quantum number, the less effective theshielding. However as the d shel l approachescompletion i t becomes an effect iüe shield between thenext quantum shell and the nucleus.

5 . 4 ( a ) S i > S ( b ) S n > S i ( c ) c a ) S i , b e c a u s e G a ) G eand Ge ) S i (d ) A l > S i (e ) M9 > S i ( f ) s ¡_ ) C l ,and Cl > F so Si > F (g) Si > C. Al l above in accordwi th ru les in p rob lem 5 .1 .

5 .5 A lso fo l low ing above ru les : (a ) po ) 1s , (b ) p > S ,(c) Ba > Sr (d) Sr > Sb (both are main group

e lement .s ) , (e ) fn > Sn ) Ge, ( f ) pb > B i( S ) K > C a > M g .

1 i . 6 2 3 2 p m

? ñ " 4 d r 0

4 d r 0c -l-

(b ) s2Pbz+ z l s l 2s r

4fL4 5s2

( d ) z + C r 3 + : l s 2 z t '

( e ) 1 6 S 2 , l q 2 z Z '

2p6 3g' 396

2t tz' 3g6

3 d 3

3dr o 4s2 ¿gt ¿d t o

( e ) 0 ' ( f ) 0 ,

t ^ 6

( n "

(f) 53r : t t zt 296 3x2 39:

s"' sg'

4 . 6 4 ( a ) O , ( b ) o , ( c ) o ' ( d ) 3 'I

Liii

only d is Paramagnetic

4 . 6 5 S e e S e c t i o n s 4 . 1 3 a n d 4 . 1 4

4 . 6 6 2 E C t t z g C u r q l N b ¡ t + 2 M o t 4 3 T c r 4 + R ü ¡ 4 5 R h ¡ 4 6 P d ¡

47Ag¡ 54Gdr 78Pt , and 79Au ' Ha l f - f i l l ed subshe l l :

Cr , Mo, Gd. F i l led subshe l l : Cu, Pd, A9 ' Au '

4 .67 (a ) representá¿ ive , meta l , (b ) representa t ive ' non-

meta l , (c ) t rans i t ion , meta l , (d ) representa t ive '

meta l , (e ) inner t rans i t ion , meta l , ( f ) nob le gas '

norünetaI

16T7

4.63 (a) +zAg*, Lt ' z" ' zgt 3" ' ¡p t gqto 4" '

(b) s2pb2+ 2, t"l z"l 2bt 39' ¡p6 ¡al o 42'

¿ l t u s " t sp6 sa to os2

| i l / \ 1 , ' l ' L lR 5

T' IT{) I ' I IRTIES OF ATOMS AND THE IONIC BOND

,¡uic Radii

r , . l . Vüithin a group the atomic size increases withincreasing atomic number because the valence shellprincipal quanturn number n, increases.

'Recall that

in the Bohr atom, x = n2 x .059 n{n. Ba > Sr forexample. Within a period the atomic size for themain group elements decreases with increasing atomicnr¡nber because the valence shell quantun number isconstant whilst the effect ive nuclear charge increasesand draws the electrons closer in. p < Si.

' , .2 Vüithin a period, the main group metals are larger thanthe non-metals, as explained in 5.1 above. Howeverthe transit ion metals are smaller, because after thed shel l is f i l led the nonmetals add to the nextquantun leve1.

' , .3 The in i t ia l ,e f fec t o f en ter ing the t rans i t ion ser iesis'a shrinkage because of the increase in nuclearcharge, while obtaining minimal shielding. the higherthe 1 quantuñr number, the less effective theshielding. Hor^rever as the d shell approachescompletion i t becomes an effect iúe shield between thenext quantum shell and the nucleus.

' . 4 ( a ) S i > S ( b ) S n > S i ( c ) G a ) S i , b e c a u s e G a ) G eand Ge ) S i (d ) A1 > S i (e ) Mg > S i ( f ) s i ) c l ,and Cl > F so Si > F (g) Si > C. A11 above in accordwi th ru les in p rob lem 5 .1 .

' , .5 A lso fo l low ing above ru les : (a ) po > Te, (b ) p > S ,(c) Ba > Sr (d) Sr > Sb (both are main groupe l e m e n t s ) , ( e ) I n ) S n ) G e , ( f ) p b > B i( S ) r > C a > M g .

' ; . 6 2 3 2 p m

496 4dr o

ap6 aar o

a +( c ) 2 1 S c " - : I s 2

1 +( d ) z , * c r " - , 1 s 2

( e ) r 6 5 2 : l s 2

( f ) E 3 I : t " '

srt

4 . 6 4 ( a ) o , ( b ) 0 ,

zt ' 2t : " '

2* 2p-G :"'

2sz 296 ¡" t

zZ' zg' ¡"t ¡gt

5p6

3 p 6

? n 6 t ¡ 3

e ^ 6

3 d r o 4 s 2 4 p o 4 d l o

( c ) O . ( d ) 3 , ( e ) o ' ( f ) 0 ,

only d is paramagnetic

4 . 6 5 S e e S e c t i o n s 4 . 1 3 a n d 4 . 1 4

4 . 6 6 2 t + C x , 2 9 C u , 4 ¡ N b ¡ 4 2 M o ¡ 4 3 T c ¡ 4 4 R u ¡ 4 5 R h , 4 5 , P d ¡

47Ag¡ 54Gdt 78PEt and 7eAu. Ha l f - f i l l ed subshe l l :

Cr , Mo, Gd. F i l led subshe l l : Cu¡ Pdr Ag, Au.

4 .67 (a ) representá t ive ' meta l , (b ) representa t ive , non-

meta l , (c ) t rans i t ion , meta l , (d ) representa t ive '

meta l , (e ) inner t rans i t ion , meta1, ( f ) nob le gas '

norunetal

L6L7

Ionization EnergY

5.7 The f i rs t ion iza t ion energy fo l lows a t rend jus t

opposite to that of atomic size. There are anomalies

in the f irst two periods however as explained in

p r o b l e m 5 . 1 0 .

5.8 Because metal atoms are general ly larger than non-

metals their ionization energies are 1ower.

5 .9 cou lomb 's law is E = q l -92 . cons ider 9 r to be -1 ,

the charge on the electron, and q2 the charge on the

ion left behind after removing the electron. Since

q2 = +2 for the second ionization, and +1 for the

first, the second ionization energy must be at l-east

twice the first. Note that r may be the same or^^^^ . ih ] . , r ^SS fOr the Second ion iza t ion , bu t never} , U D J T U ! ) ' ¡ s

greater since the outermost electron is the f irst

removed.

5.10 The sequences Be-B, and N-o, 90 contrary to the trend

across a period, because although Be ) B in size the

electron removed from B is a p electron, higher in

energy than the s electron removed from Be. The

electron removed from O, unl ike that from N, is one

of a pair occupying a single p orbital and is subject

to electron-electron repulsion which makes i t easier

to remove (again despite the smaller size of o).

5.11 Fol lowing the usual trends (a) Sr > Rb (b) Sn > Sr

(c ) Sb > Sn (d ) Sb > B i (e ) Se > Te > Sn( f ) S > S e ( g ) A r > S

5. I2 A l l fo l low the usua l t rends except O vs- N wh ich is

exp la ined in 5 .10 . (a ) Ne > o (b ) Ne > Ar

(c ) o > S (d ) F > C l > S (e ) l ¡ > o (anomalous)

( f ) M9 > Na (g ) Mg > Ca

Elect]:on Aff ini tY

5 .13 I t requ i res the expend i tu re o f 1680 kJ o f energy to

remove a mole of electrons from a mole of gaseous

fluorine atoms to form a mole of F+ ions. When a

mole o f e lec t rons jo ins a mole o f gaseous f luor ine

atoms to form a mole of F- ions, 322 kJ of energy

is l ibera ted .

14 Wi th in a per iod there is a genera l inc rease in theenergy l iberated by electron attachment because theatoms are gett ing smaller and the attached electroncan get cl_oser to the nucleus. The same anomaliesd iscussed in 5 .10 occur , and are re la t i ve ly impor tan tbecause we are concerned with energíes much less thanioni-zation energies. Thus beryl l ium (which mustattach a p electron) is so far out of l ine that tneattachment reaction is endothermj"c. (posit ive signi n t a b l e 5 . 2 . )

Down a group dif ferences are relat ively small andunpredictable. Attachment energy is expected to beless exothermic for a larger atom but this effect isoffset by the reduced electron-electron repulsion inthe larger atom.

r , .15 The pos i t i ve s ign ind ica tes tha t energy must beexpended to forcibly attach an extra electron to theatom. The attachment reaction is endothermic ratherthan exothermic which is more conmon.

' , . 1 6 ( a ) s > C I i n s i z e ; e l e c t r o n c o m e s i n c l o s e r t o C 1and more enefgy is l iberated. (b) p must put i tsextra electron into an already occupied orbital andovercome electron-electron repulsion which detractsfrom the exotherm. (c) As noted in 5.14 Be must adda p electron. which is much further out than the se lec t ron added to L i .

' -L7 A1 l negat ive ions res is t the a t tachment o f anadd i t iona l e lec t ron ( l i ke charges repe l each o ther ) .One must expend energy to force a second electron toa t tach ; reac t ion is endothermic .

, - lB (a ) Na is la rges t . (b ) Ar has the h ighes t f i r scionization energy. (c) Na is the most reactive metalbecause i t has the lo\^¡est ionizati .on energy.(d ) C l i s the most reac t ive nonmeta l . (e ) a r i sleas t reac t ive hav ing a l l i t s va lence orb i ta ls f i l l ed . .( f ) The meta ls a re the th ree e lements Na, Mg, A I .

IB 1 9

Latt ice En-ergy, Bgrn-Haber Cycl_e

5 . 1 9 - 8 2 4 k Jmole

5 . 2 r - 3 0 1 0 k jmole

5 . 2 3 - 3 2 4 k J m o l e - l

5 .24 K CI < K2O < CaO. Choose K2O over K C1 because o f

Ereater charge and smal le t ' s tze o f 'o2- . Choose Cao

over K2O because of greater charge and smaller size

- ^ 2 +

5.25 The la t t i ce energy is usua l ly the guant i t y o f g rea tes t

magnitude and mast be large enough to compensate for

the large amount of energy required to ionize the

rnetal atom, which is usually the next most importanta ' r ¡ n + i + r zY s s ¡ ¡ e ¡ s t .

The lonic Bondr T)¡Pes- of Iols

, . r l t (a ) a l l pa i red , d iamagnet ic

(b) 4 unpaired electrons, highly pararnagnetic

(c) three unpaired electrons, paramagnetic

(d) thro electrons unpaired, paramagnetic

(e ) a l l pa i red , d iamagnet ic

( f ) a l l pa i red , d iamagnet ic

,.2.() A simpli f ied way of writ ing configurations of largeatoms is to use a noble gas symbol for the'config-uration of most of the electrons, as shown in thearls\¡/ers below.

( a ) s 2 - 1 s 2 2 s 2 2 p 6 3 s 2 3 p 6 o r s i m p l y : t A r l

(b ) cu+ Ls22s22p63"23p53dr 0 o r : [Ar ] 3dr 0

(c ) cu2+ tAr l 3de^ ¿

( d ) S c 5 ' [ A r ]

( e ) F [ N e ]

( t ) sg2+ lxe ] 4 f r q5d10 (Not " txe l s l ¡mbo l inc ludes

. rhe 5p6¡

( 9 ) p b 2 + [ x e ] 4 f t 4 5 d I o 6 p 2

( h ) c r 3 + [ A r ] 3 d 3

. lO (a ) a l l pa i red , d iamagnet ic

(b ) a l l pa i red , d iamagnet ic

(c) one d orbital has an unpaired electron,paramagnetic

(d) al l paired, diamagnetic

(e) al l paired, diamagnetic

(f) al l paired, diamagnetic

(g) the p electrons are in separate orbitals, t \^¡ounpaired, paramagnetic

(h) three unpaired d electrons occupying separateorbitals, paramagnetic

.11 (a ) [Kr ] j . soe lec t ron ic w i th Se2 , Br - , pJc+, s r2+, y3+

(b) l zn?+) r G^3+, cu+, ce¡+ , (uncommon)

2 L

5:2O '2]152 kJ mole-r

5 . 2 2 - 6 0 3 k J m o l e - I

5 . 2 6 N a 2 O N a C I

Mgo M9CI

Na3N

Mg gNe

AlzO¡ AIC13 A1N

N L

5.27 (a ) l tg t ' rs¿2s2 2p6

(b) c r2+ rs22s22p63r23p63da

4 r(c ) coz* rs22s22p63s23p63d7

( d ) p d 2 + L s 2 2 s 2 2 p 6 3 s 2 3 p 6 3 d 1 0 4 = 2 4 p 6 4 d 9

( e ) A g + L s 2 2 s 2 2 p 6 3 = 2 3 p 6 3 d r 0 4 " 2 4 p t 4 d r 0

( f ) r r s ? 2 s 2 2 p 6 3 s 2 3 p 6 3 d r 0 4 " 2 4 p 6 4 d 1 0 5 = 2 5 p 6

Same as xe

2 0

(c) lznJ : Gar, Ge+2

( d ) [ o 2 - ] , F , N e , N a t , M g 2 + , A 1 3 + , N 3 -

( e ) [ c . 2 + ] , K i , s " t f , A r , c l , s 2 - , P t - , l i n *

5 .32 (a ) [ ca2+ ] r r r3+ , s r ru+ , Ag+

(b) [as+] cd2+, rn3+, srn*

( c ) t n ¡ + l K r , B r , s . 2 - , s t ' * , Y t *

(d ) tHs l r 1+ , Pbz+

(e ) I xe l r e2 - , r - , c r * , Ba2+ , L .3+

5,33 s2 io r r= a re ra re , no negat ive "2 io t t a re poss ib le

except H- rs2 . L i t and Be2t a re a lso rs2 . The sc

group never loses just 1 electron- Only others are

the d l0s2 l i " t "d be low. To summar i r " s2 : H- , L i+ ,

8.2+; t2p6 very comrnon, including most nonmetal

negative ions and the posit ive metal ions of main

g r o u p s r r r r r a n d I I I . E x a m p l e s o 2 - r ' F - , N a + , M g ' * ,

A13+; ¿10 ions inc lude a l l the zn group, " .q . zn 'n ,

a n d m a i n g r o u p r r r , e . g . G . 3 + ; a l s o s n + + , P b t 4 ; a l s o

cu+, Ag+, Au+. Negat ive i -ons aren ' t poss ib le .

d r 0 s 2 i o n s a r e r r r * , T l * , s r r ' + , P b 2 + , A " 3 * , s b 3 * ,

ano rJl-

5 . 3 4 A 1 ' * , G . ' * , S " 3 + , N ' � - , K + , B a 2 + , a t e S 2 p 6 i o r r s .

P b i + , T 1 + , B i 3 + , a r e d l o s 2 l o n s .L ' +

c u * , c d " , a r e d l o i o t t .

5.35 Addit ional energy is requ.ired to form'Cu2+ compared

to cu+, but this is not extreme because the second

electron is from the 3d she11 which is close to the

4s shel l in energy. The addit ional energy is more

than compensated for by the greater latt ice energy

in the case of crr2*. rn the case of N-tf , the second

electron must come from the 2p shel l , fat below the

3s in energy, and the energy requirement is so great

that it cannot be compensated for by the additional

latt ice energy of the hypothetical Na2+ ion.

, . i t J ( a ) C r z O g

(c) Ag2Cr2O7

(e) l¡ i (Nog ) ¡

, . l ' . ) (a ) Fe2 (SO+ ) e

(c ) Ba (oH) 2

(e) PbCrOr

( b )

( d )

( f )

( b )

( d )

( f )

, . 4 0 ( a ) m a n g a n e s e ( I I ) s u l f a t e

(b) magnesium phosphate

(c ) lead ( I I ) carbonate

(d) mercury (I I) chloride

(e) sodium peroxide

(f) aluminum sulfate

, .41 (a ) ca lc ium perch lo ra te

(b) coba l t ( I I ) n i t ra te

(c ) t in ( I I ) f luor ide

(d) potassium permanganate

(e) iron (I I1) phosphate

( f ) mercury ( I ) iod ide , o r

' . r r , ( a ) C s , ( b ) S - , ( c ) S - ,

t , . t " l ( a ) B r , ( b ) C s , ( c ) 0 2 - ,

rrrr,,r.rclature of Ionic _Cs¡mpounds

(d ) c r2+ , (e ) Ag , ( f ) As

(d ) au+ , (e ) T1+ , ( f ) r n+

Ca3 (Poh ) 2

Mg (c1o3 ) 2

ZnCO3

/ - r r Pr

A u 2 S 3

N H a C 2 H 3 0 2

dimercury di iodide

2 3

, , . ' , ( a ) H - ñ - ó - Ht "I

ü

( c )

4 . .v , : O -

( e ) H -

( a ) ¡ r - 3

( e ) , i t , i ,l l

. . 1 t . ., . { - 3 - 3 - I ,

. . 4( a ) : o : V

IO , o - J @ - ó , O" l

I: 9 : A\-/,

.. /a\( c ) : o : v

| . . ¡ a1 , 4 , c ¡

- Q , V\-/, I' Y '

Oc\ ..

( e ) V : o :l ^

, \ : ó - j @ - ; , o\ 7 " |

"

' Y ' O

( b ) : C l - o - C l :

( d ) : N = C - C = N :

- 0.9

H

covalent compounds, Lewis structures

6 . 1 H z , N z , C - 2 , F z , C L z , B r 2 ¡ f 2 r ( S 2 k n o w n a t h i g h

temperatures)

6.2 Ba Br2 must be ionic (opposite sides of the periodic

table), containing Ba2+ and Br- ions. The simplestformula is always given for ionic compounds.

A A(a) ,ñ-c=tt, , :Fg=C-ñ:KJ structure I is better

because it minimizes formal charge. In additionstructure 2 is poor because N gained a negative

charge at the expense of the more electro-negative F

I

l G )C I V

II

' Y O

o - o -

- L I :

6 . 3

( b ) : C l :It . .

t "I

' q ] ,

( d ) , r ,t ^.. I l2+l

/ - 1 : N - S V - F :t z - t\-/ |

( c ) H

t . .H - C = O :(b) " r . .@21

"/*-*\o'ro

- ';'O"\@@z:'N=N Structure I

. / \ . r : rH .b.'v

is better because there is less formal chargeseparation. In addition structure 2 is very poorbecause it requires the same sign of charge onadjacent atoms.

6 . 4 ( a ) H - C f N : ( b ) H -

( c ) H ( d ) H -

IH - S i - H, )

H

( e ) H - ñ - r ¡

IH

( b ) H

" - l@- ó ,ot "

, 9 , O

(d ) , ó , O

| '. ¡'�:\o ' T - O ' "

' 9 'o

24

6 . 8 ( a ) H - N - N - Ht lt l

U E

( c )

* @ - I ,

.'aÍ'

\ ^N \ 7 - F :

( b ) H - N = N - H

(a) O ,ó

p r . r r r r l I d , f I C €

t , . l . t Resonance is invoked in cases where a pair ofelectrons is not local ized to the bonding region ofa particular pair of atoms, so that the f.ewisstructure scheme is inadequate. Such a molecuLe is

( e )

. F - N I - \ T - F ." I | "

t t

. n l - c - q - ñ ] .

a resonance hybrid, for which the best we can do toshow its structure is to draw various resonance forms.

The structures H-C=N: and H-N=C: represent trt¡od i f fe ren t mo lecu l_es w i th d i f fe ren t ske le tons , i .e . ,dif ferent sequences of attachments of atoms. Inresonance forms the positions of the atoms areidentical, only the electrons are shif ted.

If one atom has a formal charge of 1r, no ad.jacentatom may bear a formal_ positive eharge.

ra . . @.. @..o- :N=p=N-H +-> :NIp_N_H

H-ó-Ñ=É, this structure is,-sufficient, but a possibleresonance srructure is H -@A = ñ - i ; ,O; whicrrhowever cannot be as important as the unchargedstructure

n - O ñ - s @ = [ ¡ < - - + H - ñ = i i @ - ó , O B o r hstructures important, no uncharged structure possible.A -o . . ev :g-P=N: *-* ,9=n'=¡, O

, ! -ñ , O @,s=¡ ,l l * ' l l

:N=s :6 O

' l ! -F ,A A+ +

, S -m,< - > l l l

. tT_c -o ' , , : 'oc\ ..(9

. Y :O-N-Cl,{+

ll

6 . s t . l O , c = o , O : l ¡ = o : O O r c = N , : N = N :

(b) carbon nitrogen carbon either nitrogen

O.:;.* *

' \

, /

6 . 1 0 ( a )

, . /

o..o..( b ) , ó = Ñ - I ,

( d ) o , ñ = 3 @ - F ,

6 . 1 1 ( a ) H - N - C = N :II

H

o ' e - @ t \ . .

F :

l a ' l . F r - N T = T r I - F .

Compound (a) requires us to show resonance struc

( b ) : C I :I

. - l

: C l - C = O :

( c ) H - i l - Ñ = ó ,

(d) .. G) ..o o.. o .. oo.. o: O = N - O : : O - N - O : : O - N = O :

l " + " l l < : + " 1,9 , ¿ . , O: ,9 t A'

\J \,'

( " ) q H - c : N : + - + , é = c = Ñ , O

Resonance is indicated by double-headed arrow, +->

( c )

O , s - ñ , Oi l l

ót , l / : O = N - C I :

I' Y ' O

¿ o 2 7

6 . 1 8

A.. \9

6 . L E H - N = N = N :

4 . .6.2o \ ' l :o-c=N: <-+

6 . 2 4 ( a ) C u C l z ,

( f ) A I 2 S 3 ¡

6 . 2 5 ( a ) L i g P r

( f ) F e 2 S 3 , ( 9 ) I n 2 S

6 . 2 6 B o n d i s 5 . 8 6 % i o n i c

6 . 2 7 B o n d i s 1 5 . 3 % i o n i c

structure.

o o . . @r + - > H - N - N f N :

A:o=c=N: \./ The left form, whi-ch has

(b ) MgSe, (c ) L j - I , (d ) PbCl ¡ * , (e ) CdIz '

(g ) SnIz

( b ) B e B r z , ( c ) A u 2 0 3 , ( d ) S n B r z , ( e ) A g z S ,

: F - N - N = O :" lI

: F :

This structure is sufficient.

fY

. . g / . . . . a:F - N = 5 - 6, tr . / A possible resonance structure,"

I which however cannot be asI

,Ir important as the uncharged

r , . . ! l IF > BrF > C1F > ICI > IBr ) BrC l

r, . .r ' ) Electron aff ini ty is the energy of electron attacrunentand can be measured precisely, either direct ly orindirect ly (Born-Haber cycle). Electronegativi ty isa dimensionless numerical rat ing of an element'stendency to draw electrons to i tself . I t is anapproximation dependent somewhat on the judgement ofthe person who worked. out the rating scheme.original ly (Mull iken scale) i t was based on an averaseof ionization energy and electron aff ini ty; thecurrently popular pauling scale is based on bondenergy. In any case i t is electronegativi ty whj_chassesses the total character of the atom and thusprovides us *IEñ-T guide to judging how polar a bondwi l l be .

,. lO CS < OCl < CCI < OC = SO < InI < A1S < BeI < SiO< CaS < AlO < Cacl

Use A for electronegativi ty dif ference bel_ow.6 . z L O , ó - ñ = ó , < - > , ó = ñ - ó , O

- ó , O

the negative charge on the more electronegativeatom, wÍl- l make a greater contr j-bution to lheresonance hybrid than the right-hand form.

6 . 2 2 H - C = O : # H - CI II t

: O : , a O :" \:,/

6.23 :O=C=C=C=O: This form i-s suff icient. Other lessimportant forms wíth formal charge are shownbelow.

O,ó-"="-c=o,@ *- t @:o=c-c=c-ó, O

Bonds of Inüermediate Characterr Electronegativity

a) NaBr ,b ) N B r ,c ) P B rd ) P Se) Pbr

A = o-.8A = 0 . 4A = 0 . 4

IOnlcCovalent, nearly 0B ionicCovalent, perhaps 10% ionicCovalent, sl ightly ionicSuggests covalent. However pbl2i s s a l t 1 i k e , 4 O O o C ,suggesting ionic. A is a poor

F \ D u

g ) B H

criterion in this case.A = 0 Covalent, nearly O? ionicA - .2 Covalent, very l it.t le ionic

h )i )

t , . 1 2 a )

b )

c )

d )p \

f l

s)h )i )

characterB Br A = 1.0 Covalentr perhaps 17? ionic l ike HClB a B r A = 2 . I l o n i c

C a O L = 2 . 4 I o n i cC O A = 0.8 Covalent perhaps 108 ionicC10 A = O.2 Covalent very low polari tyC Cl A = 0-6 Covalent sl ightly ionicC Mg A = 1.3 Covalent very polarC s O L = 2 . 6 I o n i cC S A = 0 C o v a l e n t v e r y l o w p o l a r i t yC I A = 0.1 Covalent very 1ow polari tyC H A = 0.4 Covalent very low polari ty

¿ ó 29

.e6 . 3 3 ( a ) C H m o r e

€(c ) FS more

<i-->(e) FS more

Nomenclature of Covalent -Binary_Compounds

6 . 3 4 B f F s ' S 2 C L 2 , P q N + ¡ T e F 5 , S s N e

6 . 3 5 P r + O o ' B F 3 ¡ C l 2 O 3 ¡ X e F 4 , I z O z

6.36 a) phosphorus pentachlorideb) iodine pentoxide, or di iodine pentoxidec) si l icon tetraf luorided) sulfur tr ioxidee) tetrasulfur tetranitr ide

6 .37 a) su l fu r hexaf luor ideb) tetraphosphorus tr isulf idec) d. ichlorine heptoxide, or chlorine heptoxided) si l icon dioxidee) dinitrogen tetraf luoride

t , l l A l " l ' l l l { 7

,llr 'tll,AR GEOMETRy; MOLECULAR ORBITALS

@NO contai.ns an odd number of electrons so one atommust be content with only seven electrons. In p C15there must be l0 electrons around p to form the fívebonds to the chlorine atoms. In N the octet cannotbe exceeded because for n = 2 there can be only fourorbitals. and. at most eight electrons; whereas forP, where n = 3, there can be as many as nine orbitalsand 18 electrons (although rarely more than 12).

AB2 l inear

Ab3 planar triangle

AB2E bent

ABa tetrahedron

AB3E trj.angular pyramid

AB2E2 bent

AB5 trigonal bipyramid

AB4E an irregular four_sided figure that may beca l led a "saw-horse"

o r a , , see_saw, , ,

AB3E2 T-shaped

AB6, octahedral

AB5E square pyramid

AB4E2 square planar

(a) AB+ tetrahedron, (b) AB5E square pyramid,

(c ) AB2 l inear , (d ) AB2E bent , (e ) aesn t r iangu larpyramid, (f) AB3E tr iangular pyramid. (9) ABaEdistorted tetrahed.ron or "sa\¿-horse,,, (h) AB3E2T-shaped, ( i) ABs tr igonal bipyramj"d

polar

polar

polar

( b )

( d )

( f )

<+OHeOHeNH

more polar

more polar

more polar

30 31

7 . 4 See previous problems for Lewis structures

(a) BHa sp3 (b ) XeFsf d2upt (c ) Bec lz sp

(d) sbF2? sp2 (e ) snc l3 sp3 ( f ) AsH3 sp3

(g) TeFa dsp3 (h ) rF3 dsp3 ( i ) s iF5 dsp3

TlBra ABa tetrahedron+

XeF3 ' AB3E2 T-shaped

SClz AB2E2 bent

AsF2- AB2E bent

GaI3 AB3 tr igonal Planar

C1F4- AB¡rEz sguare Planar

PBra ABaE distorted tetrahedron or "saw-hor

TeF5 AB5E square pYramid

sbFs2- AB5E square pyramid

T lBra- sp3 (b ) xer3+ dsp3 (c ) sc12 sp3

AsF2+ sp2 (e ) Gars sp2 ( f ) c rFa dsp3

p B r a - d s p 3 ( h ) t e F 5 - d 2 s p 3 ( i ) s b F s 2 - d 2 " p 3

Bicls- AB5E square Pyramid

SeF5 AB5E square PYramid+

C1F2 AB2E2 bent

InC12- AB2 straight

BeF3 AB3 tr igonal Planar

GeF2 AB2E bent

AsFa ABaE d.istorted tetrahedron, or

4 . .O e S C l z V : O :

A ,é -

* @ - i i , ABa sp3 re r rahed ron

v l. r ' l .

tó = 3 - ii, AB3E sp3 rrigonat pyramidII

' 9 ] '

:O = C - Cl: AB3 sp2 tr igonal planarII

. ^ 1 .

t3 = C = 3r AB2 sp l inear

+ ^( c ) s e F s ' s p '

( f ) CdBrz sp( i ) s i F 6 z - d 2 s p 3

AB2IJ2, sp3, bent. . 4O: v AB3E sp" t r igonal

pyrardd

( a )

( b )

( c )

( d )

( e )

( f )

(s )

( h )

( i )

sbF4 -

+AsCla

.

+q a É ' ^

XeF4

CdBr2

IJJ- I 4

I.Br2

" . i o . - 2 -

s D I 4 d s p - ( b )x e F a d 2 s p 3 ( e )B i r a - d s p 3 ( h )

C 1 2 O : C l - O -

? -

'

. . ;A s O 3 - : O v -

(d) osc l2

(e) oCclz

ABaE distorted tetrahedron, or "sa$/-

horse "

ABa tetrahedron

AB3E tr igonal pyramid

ABaE2 square planar

AB3E tr lgonal pyramid

AB2 straight

ABr*E distorted tetrahedron or , 'saw-

horse"

AB2E3 l inear

486 octahedron

7 . 5

7 . 7

( a )

( b )

( c )

( d )

( e )

( f )

(s)( h )

( i )

( a )

(d )

(s)

\ 4 , 1

( b )

( c )

( d )

( e )

(s)

( a )

( d )

(s )

i l ,A S -

I

,¿,O

( a )

A s C 1 4 - s p 3GeF3 sp3IBr2- dsp3

7 . 6 / . t I

( f )

t . t - 2 ( a )

( b )

( c )

(h) xeFz

( i ) A lH+

" savt-horse "

AB2E3 l inear

ABq tetrahedron

(a ) e i c r s2 - d2sp3 (b ) se rs d t "p t ( c ) c rFe+

(d) InCl2+ sp (e) BeF3- sp2 ( f ) GeF2 sp2

(9) AsFa- d,sp3 (h) xer2 dsp3 ( i ) A lHa sp3

sp3

csz

: o :tl

. . t l: C 1 - P - C l :

" I

I: C l :32

AB4 sp3 tetrahedron

(b) ,ó = s l - i i ,

4 . .( c ) v : o - C = N :

Not. tn.t onfyconsidered. The

the form: ,ó -/ñ

,a\ .. \Y( d ) ' ' ' : o - N - F :" l l

I I

o :

- É \ . .( e ) C 1 0 g v : O -

AB2E sp2 bent

AB2 sp l inear

one resonance form need be

same results wi l l be deduced from

c = Ñ , O

AB3 sp2 tr igonal Planar

i i @ - ó, O AB3E sp3 tr ieonal|

" ^ , , r ¡ m ; A

, ¿ , O

. , r l q s ¡ ¿ u

sp3 bent

O,ó ,(a ) o3c1oc1o3 (c lzoz)O,A - " l@

o, l ,

o,ó ,(b) o2NoNoz (NzOs )

( d ) F N N F : F - N = N - F : N ' s

a re abou t I 20 " . N = N ¡o r ra i "

,6 , O- ó - " l@- u ,"

, l 'o

. ." "p2 so NNF angles

r ig id so molecu le i s

Each C l i s sp3 (ABa) so the molecu le cons is ts o f twote t rahedra jo ined a t a corner (cent ra l O) .

ó , O. . @- o - N (and

reson-anceforms)

Each N is sp2 (AB3) so the molecu le cons is ts o f twot r iang les (w i th N a t cen ters ) jo ined a t a corneroxygen. Free ro ta t ion is a l lowed.

(c ) HONO(HNOz) H - ó - Ñ = ór N is sp2. Non l inear

mol-ecule with ONO angle of about 12Oo and NOH angleof alput 1O9.5o. Freely r+Lating so .four atoms arenot coplanar.

:

\

\oN

/

( e )

( f )

" , j ,o

"pyramid

H C N H - C = N : A B s P l i n e a r

/?l.. .. 1Cr - A .:XeO3 v:O - Xe \-/ - ó, \J or tó = i i: = ó"

" t " l ll r ñ l l

: O : V O :

AB3E sp3 tr igonal Pyramid

Note that the use of the alternative Lewis

structure with expanded octet (d-p 1T bonding)

makes no difference in the ileduction of the s

34

cop lanar w i th e i ther bo th F 's on same s j -de (c is ) o roppos i te ( t rans . ) .

(e ) c lssc l ,q i - :

- :

- q i ' s ' s a re sp3. Mo lecu le

i s non l - inear and f ree ly ro ta t ing a t a l l jo in ts . A f ta n g l e s a r e a b o u t 1 0 9 . 5 0 .

ó',tl¡ l

(a) oXeFa r i i - i ie - i i , Xe is d,=p, 1or sp3d2) ," / t/ \,

, ' f , . AB5E square pyramid' I '

(probably most stabl-e with O opposite the unsharedp a i r )

(b ) (HO) s IO, a lso wr i t ten Hs106 and ca l l_ed per iod ica c i d .

,ó g" - ó ,O AB2E sp2 bent

. . . . 4,ó = i i - ó , \ 7 AB2E2 sp3 ben t

2 - O , ó - 3 O - ó , O A B 3 E s p 3 r r i e o n a l

(b ) Seo2

( c ) c I O e

(d ) so3

H

" - lo- a,ot ",¿,o

AB4 sp3 tetrahedron

3 5

( c ) O S F 4 : F

w 2

ó ,fl

" l l

H - O - I - O. . , / t \

. t a l ' .

V . ^ . V/ / ' : u : \

/ I \H l

H

Á .t lI

- s -

- H I i s d2sp3¡ AB5 oc tahedronaround I with O at corners.H atoms rotate freely from

. . 5 corners .n

F : S l s d s p " , A B s , t r i g o n a l"

bipyramid (probably most: stable with O in axial

posit ion) .

k1 , ' , , r l . ¡ r Orb i ta ls r p lT-dT Bond ing

(ols and O*ls are shown only for Li2 to C2)

L íz Bee 82

++o*2s

t+o2s

++o * 1 s

t+o ls

B . O . = 0 ,

diamag.

- o z

oup

t f _f i*2p r*2p

+ +1t2p 1t2p

++o * 2 s

t+o2s

t+o*1s

t+o l s

B . O . = 1 rparamag.

F . ^

++o * 2 s

t+o* I s

B . O . = 2 ,diamag.

++tt2p

t+1T* 2p

t+T2P

++Tr2p

(d) (HO) '+XeOz n _ ^ . ?,., _ ",i,, -/ "í

n" 9'

o"'

' n / "

/ 4 ,-\. Á., , \ ' '

n

++o2s

++o*1s

t *ols

B . O . = 1 ,

diamag.

N2

o-rp

, ' *Zp tT-rp

t +;-r

t+

++o2s

f +

Xe is d"p3 , A86 oc tahedra l a r rangement o f o rscent ra l Xe, 4 dang l ing Hrs f ree ly ro ta t ing .

7.16 The Cl bonding pai-rs are more voluminous, and thusare more I ike1y to be equatorial where there are o2 neighboring pairs at 90o, rather than axial wherethere are 3 neighboring pairs at 90o. One couldargue that having them both axial would minimizerepulsions between the C1 bonding pairs, but theconsideration of 90o neighbors is more important.The least likeIy arrangement is to have the Cl bondqat 90o to each other (one axial Cl and one equatora'] \

f+ ++Tf 2p T2p

++lfrp

o-rp

f+r*2p

f+ffrp

7 . L 7 : X - P - 2 t :I . .I

X :

The unshared pair drives the X's closetogether, in an effect which wil l bemost pronounced for X = Fr because theP - F bonding pairs occupy the least

offer least resistance to being pushed

o2p

t+o*2s

++o 2 s

B .O .=3 r d iamag .

++62p

++6 * 2 s

++

B . O . = 2 , p a r a m a g .

t+o2p

++6 * 2 s

++62s

B . O . = f ,

volume andtogether.

36 37

diamag.

7 . I 9 ( a ) H z

o -1 t

t + .- 1 ^v ! >

B O = 1

+7 . 2 A O z '

+ 1 + t +

+( D J i l 2 (c ) HHe

+

( d ) H e z ( e ) H e z +

+ + +

1+

_L2

w 2

ñ*oJ(

T* firt , lT¡tfI ¡t

t+o o

!

w2

t+ ++TT 1T

3

á o

0 2 o z 2 -

O r k O * o?t

f+ t+,lT t TfJ<

++ ++,TT 1I

d *

t+ ++1T 'II

B O = 2

C2 is isoelectronic ( i f we disregard the order of

o a n d n o r b i t a l s ) w i t h N 2 , N O + , C N , a n d C O .N2 and CO are the neutral isoelectronic molecules.

+co co' co No

t + + + + t1T.- TiT- 1T*- F-- nf- tT.-

r + f + f + + + + + J +1I 'IT 1T 1T 11 1T

( b )

( c )

d t k

tT- tT-

r+ ++

U

B o = 3

+NO

¿ áparama9.

NO

paramag.¿z

paramag.

++o

B O = 2 ¿paramag.

7 . 2 L N 2

t *

2parama9.

ÍN 2

^ - t ,-;-

L 2

paramag.

t+

1diamag.

Oz Qz

++f +t +t ++*

t+++t +t+

ft+,IT

1T.

t+1I

T : t

^ 1 az z ¿

t

t+

ó la 2

t+

t+++++ñ

B O = 3

Since distance var ies inversely \ , r i th B-o. N2 is. | . +

sho r t e r t han N2 ' and 02 ' i s sho r t e r t han 02 .

++ ++

+l

B O = 3

++ t+

++

2paramag.

3 B 39

4 . . . . a7 . 2 4 \ J : O - N - O : < - - + : O = N - O : V T h e r e s o n a n c e

show the negative charge shared between the twooxygen atoms and the bonds intermediate betweenand double bonds.

Alternatively we can say there is a sigrna bondbetween N and each O and that 2 electrons are in adelocal ized TI bond connecting al l three atoms.

7 . 2 s O , ó - ó O = ó : + - + , ó = ó @ - ó , O " o , r u i s b e r w e e n

a single and a double bond.

O , 6 - S @ = ó : + - + , ó = B @ - ! i , g s i m i r a r r o o zcaser however there is a preferable structure whi

has no formal charges :O = S = O¡ . This isal lowable for sulfur, a 3rd period element, whichuse i ts d orbitals to exceed the octet rule, notpossible for oxygen. Furthermore, a sl ightcontribution from the minor resonance forms:

, ó = É O = o , @ - * O : o = O ' s = ó ,

suggests that the bonding is even stronger thanbond, so that it is even shorter than the d.oubledistance. There are also contr ibutions from theforms:

O,a - ' s = o ,O. . - *O:o = B - ó ,O

which average to the double bond. Multiple bondingby sulfur, when the octet rule is exceeded isattributed to d-p pi bonding.

7 .26 In add i t ion to th is s t ruc tu re , ,ó ,V

, óO- sL - ó , O" I é i 't 9 r -

si l icon a third period element may exceed the octetru1e. There are four minor resonance forms such as

O . 6 -

which j-ndicate some mult iple bondinS (d_p pi bonding)thus shortening the bond length. Note that sincethese four forms put negative formal charge onadjacent atoms they must be very minor contr ibutions.

7 , - ' I PHs Expect bond length = 1OO + 32 = I42 pm, jusr asfound experimental ly.

PFs Expect bond length = 1l_O + 64 = I74,than found. The actual bond is shortenedbonding using d orbitals on p (3rd period)suggested by three resonance forms like

much greaterLr<r z l -n ^ iV J V } / } J ¿

A S

: F - i l o= Fot "

: o :il

, J O - a , Ot "r A

4 04 T

C}IAPTER B

lsJ¿J

Sj-mpl_e Gas Laws

8.1 (a) At constant temperature the volume of a gasvaries inversely with i ts pressure.

(b) At constant pressure the volume of a gas samplevaries direct ly with the absolute (Kelvin) temperat

(c) At constant volume the pressure of a gas samplevaries directly with the absolute (Kelvín) temperat

4 . 2

lst l ine 22.4 l- íEer, 2nd I j .ne3rd l ine 1 .633 mole , 4 th l ineatm

(b ) 3 .45 a tm .

9 , 5 1 l i t e r

0 .496 a tm .

44.8 J,l-Ler,27 .5K , 5 th l i ne I L .76

8 . 1 8 L . 3 4 s " + -I l_Éer

t t . l ' l 5 . 9 7 l i t e r

r l . L9 .4 '74 arJm.

i l . . , l l - 34 .19 mole

i l . .13 58 .09 mole

( a )

( c )

6 . 5 5 a t m .

4B5K o r 2L2aC

J ' 1, . , r l cas La\^I

r t . L 2

u . l 4

l t . L 6

2O5 m1.

619K or 346aC

8 . 1 3

4 . 2 0 3 9 0 K

4 . 2 2 8 0 . 9 9

^ r 1 1 a o ^

_ - I

mol.e -

1'"{-.1 ' r i r r c i p l e

t t . . ) .4 30 . O0 l i te rs CHa requ i red , 45 .0 l i te rs 02 requ i red ,3O.O l i te rs NH3 requ i red , 90 .0 l i te r " HrO(g) p roduced

l | . : i 5 1 0 . O l i t e r s H C N

t t . . ' . 6 4 N H 3 ( 9 ) + 5 O 2 ( S ) - r 4 N o ( 9 ) + 6 H z o ( g ) 4 0 . O

t t - . ) . J 4NHg (g) + 302 (g ) + 2Nz (g ) + 6H2o(g) ; -2 .s

¡ r . . l B 0 . 5 0 l i t e r C l 2 r e m a i n 1 . 5 0 l i t e r N 2

9 . O O l i t e r N H 3

; r . . 1 9 ( a ) 3 N O - > N 2 O + N O 2 ( b ) 8 3 . 4 m I t o t a l

(c ) P" ,^ = p . . ^ = O.5OO atm.I \U2 N2U

I t , l 0 ( a ) 4 N H 3 ( 9 ) + 3 F 2 ( g ) + N F s ( g ) + 3 N H a F ( s )

( b ) 1 0 0 0 m l N H 3 , 7 5 O m l F 2 .

l i t e r s N o ( 9 )

l i ter N2

t"lL"""2,""V ,rt,L" - t

- l

8 . 3 ( a ) 7 5 0 m 1 .

(c ) 15Oo ml .

A . 4 G ) L 2 . 0 a t m .

( c ) 4 . O 0 a t m .

8.5 1000 bal loons

8 .6 7 .46 x 10 -6 acm

8 . 7 ( a ) 4 . 0 4 l i t e r

( c ) 1 1 7 . 2 K o r - 1 5 6 " C

B . B ( a ) 1 7 1 m 1 .

( c ) 3B2K o r 109 "C

8 . 9 0 . 9 1 6 m 1 .

8 . 1 0 ( a ) 2 . 3 4 a t m .

( c ) 2 2 3 . 5 K o r - 4 9 . 5 " C

(b ) 150 m l .

(b ) 0 .120 a tm .

352K o r 79 "C

2 lBK o r -55oC

( b ) 1 4 9 0 K o r L 2 L 7 " C

( b )

( b )

42

8 . 3 1 5 . 7 L g l i t e r - r 8 . 3 2 5 8 . 0 g m o l e - l

8 . 3 3 ( a ) 1 . 2 0 x l o r a l i t e r

( b ) 3 . 2 3 x 1 0 3 6 m o l e c u l e s

8 . 3 4 ( a ) B . O x 1 O - s S ( b ) L . 2 5 x 1 0 - 6 m o l e s S o 2

( c ) 2 . 8 x 1 0 I

a t m ( d ) 2 . 8 x 1 0 - 6 g .

8 . 3 5 ( . ) n " O = 8 . O O x 1 0 6

a t m a t 2 7 3 K

( b ) 2 . 1 5 x 1 O r 7 m o l e c u l e s

( c ) B . O 0 x 1 O - + %

Stoichiometry and Gas Volumes

8 . 3 6 ( a ) 4 N a ( s ) + 3 N e O ( g ) + N H 3 ( 1 ) + N a N s ( s ) + 3 N a o H ( s )

+ 2Ne (g)

(b ) 12400 ml Nzo

( c ) B 2 7 o m l .

Note : Answer in tex t i s O.K. bu t d i f fe ren t becauseequation (a) can be balanced in more than one way.

8 . 3 7 ( a ) 2 H C N ( q ) + N o 2 ( g ) + C z l ¡ z ( 9 ) + N o ( g ) + H z o ( g )

(b ) ml HCN = 5600 ml . r m1. NO2 = 2BOO ml .

( c ) 2 8 O O m l .

B . 3 8 ( a ) c a H z ( s ) + 2 H 2 o ( 1 ) + c a ( o H ) z ( s ) + 2 H z G )

( b ) 4 . 7 o g

8 . 3 9 ( a ) c a ( s ) + 2 H 2 o ( 1 ) + H ( 9 ) z + c a ( o H ) 2 ( s )

(b) 8.95 9; more than problem 38 because CaH2 alre

has H2 " in i t . "

B . 4 0 ( a ) A l + C g ( s ) + I 2 H 2 o ( 1 ) + 3 c s + ( g ) + 4 A l ( o H ) ¡ ( s )

( b ) 1 . O O l i t e r

B .4 t (a ) Laz (Cz) s + 6HzO(1) -> 3CzHz@) + 2La(OH) ¡ (s )

( b ) 0 . 3 6 1 l i t e r

r l . , t . t . 5 0 6 l i t e r s 8 . 4 3 I . 6 9 4 l i t e r

i l . , t 4 3 9 . 6 ? A 1

l t . , ' ¡ ' i (a ) .125 moles hydrocarbons ; .g125 moles.5OO moles COz; .625 moles water

(b) 2 (hydrocarbon) + 1302 + BCO2 + ]0HzO

(c) 2CaH1s + 1302 -+ BCOz + IOH2O

t . 4 6 ( a ) . 0 1 7 5 m o l e , . O 4 O O m o l e

( b ) . 0 0 2 5 0 m o l e ( c ) C z H r e

I,a!L of Partial pressures

t t - 47 n * ro = . I 94 a tm . , n * , = . 306 a tm .

l l . 48 " " . = . 1875 a tm . , "Nu = . 0625 a tm .

t t . 49 ( " ) " "o = . 25O, Xco , = . 750

(b ) . 350 mo le ( c ) 2 . 4 5 9 C o i

r r - ' ) 0 O . l B B l i t e r B . 5 l O . O 9 1 l

t t . ' i 2 . 7 2 7 a t m 8 . 5 3 . 0 4 6 9

t t . ' ;4 (a) L.379 moles of mixed gases

oxygen;

1 I .559 CO2

liter

4 U l t

(b ) 1 .00- x = moles o f N2Oa remain ing

2x = moles NO2 formed

.621 moles o f N2Oa remain ing . .75g moles o f NO2formed.

( c ) a n d ( d ) . 4 5 0 a t m X " , ^ = . 4 5 O ¡ . 5 5 0 a t m

X * o , = ' 5 5 0 '

¡ ' l " t i c fneory o f

11.55 (a) The actual volume of the moleculessmal1 compared to the voh:ne in whichconfined.

l s

the

negligiblygas is

44 45

(b) Molecules move in straight l ines untiL theycol l ide with other molecules or the walls. A]- lco l l i s ions are e las t i c , i .e . , to ta l k ine t ic energyis conserved.

(c) The kinetic energy of a gas is proport ional tothe absolute temperature and independent of the naof the gas .

(d) Attract ive forces between molecules are negl ig

8.56 Pressure is proportional to the rate at whichmolecules impact on the wal1s. If volume is reby half then there wil l be twj_ce as many moleculesper'unit volume and rate of impacts wil l be doubled¡so the pressure wil l be doubled, as predicted byBoy le 's Law.

Rela t ivenumber o igh Tmolecu le

(b) energ"r

IOOK speed = 279 m sec

5O0K speed = 624 m sec

) l 5B7K

) CO rate = L.25 x COz rate

t 6 . 0

| ( a ) 1 . 1 8 5 I l i r e r - r ( b ) 5 8 . 0

" . 676 g l i t e r -

. 9o2g l i t e r - r

3 0 . 0

I Gases

{ 'b Two main reasons are i (a ) f in i te s ize o f themolecules, and (b) attract ions between molecules.

t, l (a) Hz cl-oser to ideal because i{I is larger and hasgreater intermolecular attract ion.

(b) The gas at IOOoC wil l fol low the ideal gas lawsbetter than at 10OK because the energy of inter_molecular attract ions wil l be much less signif icantcompared to the thermal energy (kinetic energy) acthe higher temperature.

(c) More ideal at I atm. because at lO atm. themolecules are closer and attract ive forces are moreef fec t i ve .

- l

- l

When the temperature is doubled the'average speedof the molecules increases bV ñ. The increased sp,affects the pressure two \^rays; (a) more frequentimpacts, and (b) more momentum transferred perso the overall effect is E x /l = doubling ofpressure, as predicted by Anontonrs law.

8 . 5 7

Charles' Iaw can best be deduced by f irst con-sidering the volume constant. Then a d.oubling oftemperature doubles the pressure (as above) and inorder to restore the original pressure the volumewould have to be doubled (in accordance with Boylellaw) "

Rela t ive lowhigh T

number omolecu le

(a) sPeed

¿ro4 7

(d) The gas with T. = 1O0K is more ideal because the

low cri t ical temperature is an indication of weakln te rmelecu la r fo rces , i .e . , on ly a smal l amount o fthermal energy is required to prevent coalescence ofthe molecules into a l iquid phase.

8 . 6 8 ( a ) . 5 1 4 l i t e r i . O 3 2 L l i t e r

(b ) A t 50 .0 a tm the ac tua l vo lume ( .314) i s IesB tideal (.514) because intermolecular attract ions arethe dominant deviation.

At 80O atm the actual volume (.O42L) i-s greaterthan ideal (.0321) because excluded volume is thedominant deviation.

( c ) A t 5 O a t m , 6 1 1 ; a t B 0 O a t m , L . 3 L 2

8 . 6 9

l r r te rmolecu la r Attract ive Forces

London forces are universaldirect ional and are due toby f luctuations in electron

of on ly . 10-1 s

and completely non-temporary dipoles caused

density on a t ime scaleseconds. Dipole-dipole forces are due

(.I IAPTER 9

t,TQUIDS AND SoLIDS

to permanent dipoles "-,r."á by unlompensated. polarbonds. The negati.ve end ot oie *oi..rrfu attracts thepositive end of the other. Many molecules have nodipole moment and such forces ale absent. UsuallyLondon forces are dominant (see table 9.1, p . 2it6).Expect Clz to have highest Tc

highest "a" vaIue, indicatingattract io¡1.

because i t has the

greatest inlermolec' t .2 (a ) OFz must be bent

so the bond dipolesso the bond dj_poles

(two unshared pairs on O atom)add vec tor ia l l y . BeF2 is s t ra iqh tcancel each other

8 . 7 0 ( a ) 2 2 . 4 1 a t m . ( b ) 2 1 . 9 3 a t m .

(c) The van der Waals equation shows that the pris s l igh t ly less than the idea l .

8 . 7 1 ( a ) 2 . 2 4 1 a t m (b ) 2 .236 a tm

(c) The van der !{aals equation shows that the presis very sl ightly less than ideal; but closerrelat ively than in problem 70, because in problemthe pressure is higher and behavior is less idea1.

8 . 7 2 ( a ) 3 0 . 6 2 a t m . ( b ) 3 0 . 4 8 a t m .

(c) The van der htaals equation shows that the presgis s l igh t ly less than idea1. The d i f fe rence is 0 .4compared Lo 2.L42 in problem 70. At higher tempebehav io r i s c loser to idea1.

He has lower intermolecular forces than H2 as shownby an a value of only .0341 compared Eo .244 for [ I1r

(b) PF3 is a pyramidar- molecule. rf you consider theunshared pair on p to be pointed up\^rard each of thePF bond dipoles has a downward component, which addtogether. BF3 is planar and the btn¿s are at 12Oo so

l l " - : : : " t .anr ( i .e. , rhe vecror ial sum) of rhe rhree

(c) S¡' ,* is distorted tetrahedron with the S atompractical ly on an edge (a1so sometimes carred a ,,sawhorser " a "see-sahr r "

o r "a isp i reno iá r , , doub le wedgedshaped) so that the SF bond áipoles cannor canceleach other. SnF4 is tetrahear-af wit¡, sn at the centerand the resultant dipole moment is ,ero. (Consider

one pair of SnF bonds and observe that i ts resultantmust be equal and opposite to the resu.l-tant of theother two. )

t - J HgC l2 r CuC I2 , BF3 , CHa ¡ pC15 ¡ XeF2 , SFG , XeF ¡ , .

4 The only arrangement

a ? ?

ó . 1 ¿ rboththere

Cl atoms axia1.which has zero

If one or bothdipole moment has

are eguatorial- t o a

( a ) 1 . 7 7 x 1 0 - " m " (b ) . o4772 must be a dipole moment.

A . 1 5 . l 5 B n m

48 4 9

9 . 6

Al I Lrond dipoles cancel in pF5; the axial contr i_but- ions are equal and opposite, and the equatorialare tr igonal planar (as in BF3). pF3 is a tr igonalpyramid with al l bonds contr ibuting, as discussed in9 . 2 b .

Both the N - H and the N - F bonds are polar, butthe negative end of the NF3 molecule is directed awalrfrom the unshared electron pair and is part ial lycancelled by i t . The negative end of NH3 is in thedirect ion of the unshared pair and the molecule dipomoment contains contributions of the same sj-gn fromboth the bond dipoles and the unshared pair.

The case of phosphorus is dif ferent because p ismuch less electronegatj-ve than N. As a result pHbonds are practically non-polar and the dipole momentof PH3 is due only to rhe unshared pair. ;r ' ; ;á;-"-are more polar than NF bonds and make such a largecontr ibution that the counter_effect of the unshiredpair is relat ively tr ivial . The argrment,s can bevisual ized in the f igures below.

').9 As one goes to higher atomic numbers, the atoms (andconsequently the molecules) get larger. Their oucerelectrons are more diffuse and further from thenucleus. Such electrons are more easi ly subject tofluctuations and also more polari-zab1e ly ,r.igfrboringtemporary dipoles. Thus London forces aie greater inthe rarger morecures and the crystals can retain theirstructures to higher temperatures.

') '10 Dipole*dipore forces exist only in those with a d. ipolemoment: CH3C1¡ CH2CL2, and CHC13. Chlorine is a verylarge atom (compared. to the others) ¡ and Londonforces bet\r¡een Cl atoms in neighboring moleculesprobably dominate over other considerations. Thusexpect that the boi l ing points wil l increase in theorder writ ten, with CHa far below the rest.

'l:lls ¡Ilqgoge!_ iond

NHs NFs D H ^

1p

¡ i ¡

,|.

, r 1 l

o I t HFe

9 . 1 3

both these cases the averagie nu¡nber of H bonds per

HF ) NH3 because n' isthan N. Thus the H -and causes the H bond

mol-ecule can be aLunshared pair and

so much more electronegativeF dipole moment is very largeenergy to be very great. In

most l , because NH3 has just IHF has jus t one H.

H bonding can occur both by (1)any H in NH3 joining an unsharedpair on the O atom in H2O, or

H2O joining the unshared pair on the

Any H in NHzOH can join anunshared pair on the O in H2O,

H or eifher H in H2O can join anyof the three unsT-rared paj-rs inNH2OH shown above. ,

t t

z i" , i \ /j\ Despite electronegativi ty considerations, H2Oexceeds the other two because the average number ofH b:nds per molecule can be as high as 2.

9.7 Assu¡ne that the compound.s are al l straight, andisoelectronic except for the S core. Then CS2 l ikeco2 has zero dipore moment because the cs diporesare equal and opposite in direct ion. The o l tom ismore electronegative than S and so the CO bond dipoleis greater than the CS and gives a resul_tant equal tothe dif ference, with the negative end toward O.-

Q" = o, @ rn. r,ewis structure reveat-s formal cwhich act to produce a "dipole', in the oppositedirection to that attributabte to the electro_

can be thought of as being formed by a hydrogen

b o n d b e t w e e n H F a n d F , i . e . , , F , O - - - - H - i ,

but in fact the HF2 ion (bette; writ ten FHF-;is slmmetrical!

o a

( a ) r ¡ - Ñ - nI

H

negativi-ty dif ference. The net sma1l dipole moment ithe result of these contradictory tendencies.

( 2 )

N i n

(b)

either H inN H s .

H

IH - N -

5051

t ' l re Liquid StateThe H on the O in CH3OH (as H*) can join an unsharedpair on the O in H2O¡ or eiH in H2o can join an unsharedpair in the O in CHTOH.

(d) H ¡either H in CH2O can part icL| . . pate. But nonetheless the

H - C = o : so lub i l i t y o f CH2O in H2O isgreatly enhanced by n bondlng

of either H in H2O to an unshared pair on the O inCH2O.

9.L4 One part of the explanation has nothing to do withH-bonding; i t is that the HSO+- salts have much IeEalattice energy to overcome than the SOa= saltsbecause of the smaller anionic charge. H_bondingplays a role also. In both SO+2- and HSOa- the Oatoms can all H bond to the H atoms in H2O, but theHSOa- has the additional advantage that its H arom(which is attached to an O) can H bond. to the O in

H e o .

9 . 1 5 The boiling point data suggest that there is anadditional force of attraction between the unlikemolecules that doesntt exist in either pure l iquid.That additional attraction is an H bond between Cand the O in acetone. The three Clrs pul l enough

'¡. I7 (a) : f the cri t ical temperature is high Lhere muscbe strong attract j .ons, suff icient to cause themolecules to coalesce into a l iquid despite the highkinetic energy associated with high temperature.(b) At the surface a molecule has fewer neighbors thanin the bulk. I f intermolecular attract ions are greac

the molecule has a strong,inducement to move into thebulk phase and increase its number of nej.ghbors,Ieading to a high surface tension. Hign surfacetension means a large tendency to reduce the surface_to-volume rat io, i .e., to minimize the number ofsurface molecules rel_ative to bulk molecules.(c) rf one compares molecules of simirar size andshape, then a high viscosity indicates strongattract ions- Molecules strongly attracted tá eachother will not move easily pasu one another and flowwilL be sluggish. Hor^/ever lh.." are many substanceswith l i t t te intr insic intermolecular attract ionwhich are viscous because the molecules are very longand become entangled (e.g. petroleum tubricantsl.(d) e 1ow vapor pressure indicat.es Iarge inter_

molecular attractions which retard the escapingtendency. A molecule can escape only i f i t has enoughenergy to overcome the attract ions of i ts neighbors.

( c ) HIt . .

H - C - O - H *1 . .

IH

H bonding is of little importance because Cl is notstrong donor, but the O in acetone is. Converselynone of the H atoms in acetone is "H bond active', sOH bonding is of no importance in pure acetone.

9 . L 6 Both molecules have permanent dipole moments andhave similar london forces. H-bonding is present lnboth and because of the 1arge H bond energy it is thrdominant force of attraction in both. The dif

(f) tne higher the boiling point the greater theattract j_ons must be. Boi l ing points vary in theopposi-te way from vapor pressure, i .e., lo", vaporpressure means high boi l ing point. (See part á. ¡

' )- IB The energy a molecule must have to escape from itsneighbors is much greater than the average energy.The Maxwell-Boltzmann curve shows that at any onetime a small fraction of the molecules have sufficientenergly to escape. These are the only ones whichescape, and so the average energy of the remaindermust decrease. I f average energy decreases,temPerature decreases.

electron density from C to render the H atom prone tH bond formation with a strong donor. In pure CHC1¡

lies in that ethylene diamine has t\^/o _NH2 groupsmolecule h¡hich leads to twice as many average H b

(e) Enthalpy of vaporizationintermolecular attractions ;the more endothermi_c is the

is a direct measure ofthe greater the attractionsvaporization process.

per moLecule and thus much more attractive forcebetween molecules.

5 25 3

, ) ] q

f i11in9 i t only part ial ly, the space above wil l

increase the l iquid,s vapor pressure untiL i tthe imposed pressure.

9 . 2 L ( a ) . 0 1 0 a t m : 7 . 1 o C ( b ) . 0 2 5 a t m : 2 L . 3 0 C

Phase Diagrams

a ) ')

1 ? L

1 0 . 0

P/Atn.

1 . 0

'I

. 0 1

An equil ibr ium state is one in which the quanti t ielof substances in equili_brium undergo no change with!1*.: r f a l iquid is placed in a Jrosed container

the l iguid per second, thus maintaÍning the populaconstant.

9.2O The boi l ing point is that temperature at whi_ch the

populated with molecules (of the l iquid substance)the vapor phase- That populat ion remains constantwith t ime as indicated by the constant pressure ofthe vapor, Actual ly the equil ibr ium state is adynami-c condition in which many molecules leave theliquid phase per second and an equal number ofmolecules (not necessari ly the same ones) return tO

vapor pressure is the same as that imposed on thesurface of the l iquid. Vapor pressure is a strongfunction of temperature; i f the pressure on thesurface is increased, raising the temperature wil l

P/Atu¡.

. L 7 5

.100

. 0 0 1 3 r / ' c- o JL 9 9 - L 6 9 - L 5 7 - L 5 2

Solid Xr is denser than l iquid at l .O0 Atm. ThisfoLlows from the posit ive slope of the l iquid-sol idl ine which is "normal. ' , An , 'abnormal" substance l iker^rater is denser in the liquid state and correspondsto a negative slope.

(a) Minus loC. The pressure is ini t ial ly so low thatthe entire sample is vapor. Vfhen the pressure reachesabout .006 atm the sample condenses to sol id ice.Eventual ly at very high pressures (ru100 atm) the icewj- l1 melt as you cross the almost vert j_cal sol id-l iquid l ine.

(b) 5OoC. When the pressure reaches ,r, .12 atm thevapor condenses to a liquid and undergoes no furtherphase change. A t exceed ing ly h igh pressures (> IOTOOOatm) there are dif ferent forms of ice which are stableas h i -gh as 50o and more (no t shown in f ig . 9 .9 ) .

100

1 0

I . U

Estimate the boi l ing points as closely as possibl lfrom the graph; there wil_l be honest dif ferencesbetween observers. Approximate values are: 19o, 6tc82" .

(c) Minuscondensessuperhighice which

5ooc. At tu 4 x 1o-5 atm. the vaporto a solid, which never melts. Ho\,rever atpressures there are other exotic forms ofbecome stable.

3 02 01 0 40T/x

roxf-

- 3' ¡ .25 (a ) t x 10 -

a tm. A t -10oC the sample is a l l vaporand remains so as i t is heated.

(b ) O.5 a tm. A t - lOoc i t i s so l id (o rd inary i ce) andat tuOoC it melts. At about g2oc i t vaporizes andremains vapor.

Note: The pressure scale is non_linear but appT1t: ly logarirhmic in order to-shov¡ afftea tures c1ear lv .

5 455

(c ) 1 .1 a tm. A t - IOoC i t i s o rd inary i ce and a t' r ,0oC ( jus t s l igh t ly less) i t me l ts . A t about lO3oCit vaporizes and remains vapor.

9.26 (a) Minus 60oC. This is between the normal subl ima-tion temperature and the triple polnt so at about 4Atm. the gaseous sample condenses direct ly to sol idCOz¡ and remains so. I t can never become l iquidbecause the sol id-l iquid l ine slopes away in thenormal fashion.

(b) ooc. This is far above the tr iple point but

. below the cri t ical point so at a suff iciently highplessure (n 30 etm. ) the sample condenses to a l iquidand remains so.

9 .27 (a ) 2 a tm. The so l id ( "d ry i ce" ) sub l imes a t about- '72"C and remains gaseous.

(b) 6 atm. This is above the tr iple point so thesol id wil l melt to l iquid COz at about -56oC. Thenat about -53oC the l iquid wil l vaporize and remainvapor.

9.28 Consider the sol id and 1iquid forms of X inequ i l ib r ium: X(s ) i=+ X(1)I f , as i s usua l , the so l id i s denser , then an inc rin pressure wil l favor formation of the sol id (LeChate l ie r ' s p r inc ip le ) . The ske tch on the le f t besho\^/s lhat this will be so only if the slope of themelt ing point l i -ne is posit ive. The oppositesituation, which prevai ls for water, is sketched on+ h a r i ¡ h +

' ) .29 Yes , i ce can be pur i f ied by sub l imat ion in anapparatus in which the pressure is always less thanthe t r ip le po in t p ressure o f about 0 .006 a tms.Likewise the condensj-ng surface would have to bebelow the subl imation temperature for the pressureused. (G iven by the vapor p ressure curve fo r iee- )

' l ' ypes o f Crys ta l l ine So l ids

9 .30 The answercolumns on

9 . 3 1 ( a ) S i , a

( e ) B F 3 ,

( f ) P F 3 ,

e . 3 2 ( a ) o z ,

appears in tab le 9 .4 int h e l e f t .

the f i rs t th ree

( b ) B a , a

netv¡ork crystal , covalent bond.s.

meta l , meta l l i c bonds .

(c ) Fz , mo lecu la r c rys ta l o f non-po la r mo lecu les ,London forces.

(d ) BaFz, j -on ic , e lec t ros ta t i c a t t rac t ions .

9 .33 (a ) BrF has s t ronger Londond ipo le -d ipo le fo rces .

(b) BrCl has stronger Londond.ipole forces, which C12

non-polar, London forces.

po1ar, London and dipole-dico1e.

non-polar, London forces.

non-polar, London forces.

forces as well as stronqer

fo rces p lus d ipo le -I a c k s .

mel t ing ;po la r mo lecu la r .

Br2 wh ich is

( b ) B r z ,

molecular

molecular

molecular

molecular

(c ) BrzO, mo lecu la r non-po la r , London fo rces .

(d ) Ba, meta l , meta l l i c bonds .

(e ) BaBre , ion ic , e lec t ros ta t i c a t t rac t ions .

( f ) BaO, ion ic , e lec t ros ta t i c a t t rac t ions .

(c ) Cssr i s ion ic and most l i ke ly h ighstronger forces than BrCl which is

(d) Cs which is metal l ic, compared tonon-po1ar molecular (London only).

(e) C diamond which requires the breaking of covalentbonds. C12 non-polar mo.lecular requires over-coming only London forces.

565 7

9.34 (a ) s r ; i t requ i res b reak ing meta l l i c bonds whereasC12 requires overcoming only London forces.(b ) SrC lz ; ion ic c rys ta l . S iC la i_s a non_po1ar

mor-ecule requir ing overcoming only r,ondon forces(c) SiBrr+. Both are non_po1ar molecular but SiBramust have greater London forces because bromineis a much bigger atom than chlorine.(d ) SC14. These seem c lose , bu t SC14 is po la r wh i leS iCI r i s no t , so there is the add i t io ia l á ipo fe_d ipo le fo rce .

(e) SiC is a covalent net\,rork sol_id while Siclk ismerely a non_polar molecular sol id bound or.rfy ¡yLondon forces.

9 .35 (a ) f , i . Meta l l i c bonds ; versus H2 on ly(b) L iH. Ion ic c rys ta l r e . l_ec t ros ta t i c

H2 on ly London fo rces .

(c ) L iH. I t i s d i f f i cu l t to choose be tween ion icversus metal l ic crystals. Hoi^/ever LiH wi-th verv

9 . 4 0

9 . 4 I

9 . 4 2

9 . 4 4

O /'1tr

9 . 4 7

M = 5 5 . 7 7for which

M = 4 0 . 1 0

316 pm

N = 6 . 0 2 x

Cube edge

1 2 4 . 5 p m

= 2 . I 7 c m

g mole-r (The element is undoubtedly ironthe accura te a tomi .c we igh t i s 55 .85 . )

g mole- r (p robab ly Ca, M = 40 .08)

9 . 4 3 3 8 9 p m

1 0 2 3 a t o m m o l e - t

London force

forces ; ver

simple

toni-c Crvstals

e . s s ( a )and

( b )

e . 5 6 ( a )and

( b )

e . 5 7 ( a )and

( b )

The zinc sulf idefour ca t ions .

5 4 1 . 6 p m

3 8 4 . 5 p m

594 pm

584 pm

BaO, CaS, NaBr ,

9 . 4 6 I 2 7 . 6 p m

9 . 4 8 3 1 4 p m

9 . 5 0 2 2 2 p m

9 .52 211 pm

1 4 . 9 o ; f o r n = 2 1 0 = 3 0 . 9 "

1 6 . 1 " ; f o r n = 2 , 0 = 3 3 . 8 "sma11 ions, Li- and H , has a large latt ice eneand ranl<s high among ionic .ry=tui" Ín melt ingpoant. Li ranks lGroup r) because "?'.ililnrl:;:t:.ff.ulrllt"lÍthe very 10w dens i ty e lec t ron c l0ud wh ich f i1 lsthe interj-onic space.

(d) Clz. Both are non-polar molecular but C12 hasmuch greater London forces.

(e ) ¡ tC t . I t has grea ter London fo rces , p lus d ipo le_d ipo le fo rces wh ich H2 1acks .

Crystals

9 . 3 6 1 . 4 5 3 g c m 3

9 . 3 7 3 . 5 9 8 g c m 3

o ? o ¡ ¡ _ r .uuu acom. po must c rys ta l l i ze in acub ic la t t i ce .

9 .39 n = 4 a toms. Au must c rys ta l l i ze in a face centeredcub ic La t t i ce .

The cesi.um chloride unit cel l- contai-ns one anionone ca t ion .

4L2 pm (c ) 357 p¡n

The sodium chloride unit cel_l contains four anionsfour ca t ions .

555 pm (c ) 278 pm

9 . 5 8

q ( o

9 . 6 0

9 . 6 1

( a )

( a )

( a )

1 \ I J ,

unit cel l contains four anions

( c ) 1 6 5 . B p m

( b ) 7 . 0 1 s c r n -

t D , / . 5 9 g c m

( b ) 4 : 8 2 s c m 3

AgC1, KC1

5 9

X-Ray _Diffract ion gf Crystals

9 -49

9 . 5 l

o q , a

9 . 5 4

348 pm

7 0 . 8 p m

f o r n = 1 ¡ 0 =

f o r n = 1 r 0 =

5 B

Defect Structures

9.62 Dislocations. Planes of atoms are not perfect lyparal lel. for example where two crystals join; oran extra plane of atoms extends through a portionof a c rys ta l .

point defects arising from deviat ions from perfectstoichiometry. Extra anions or cations may be lodgin interst i t ial si tes, or some latt ice points may bevacant.

Impurity atoms.la t t i ce po in ts ,

9 . 6 3 ( a ) 0 . 5 e " o x y g e n

( c ) 8 . 2 3 9 9 c m - 3

9 . 6 4 ( a ) 5 . 6 4

9 . 6 5 ( a ) 5 8 . 3 9 4

These wil l frequently be found onsubsti tut ing for the proper ions.

vacanc iesr (b ) 8 .244 cm-3

(b) 7% ca t ion vacanc ies

(b) L4e" chloride ion vacancie

r ' I IAPTER 1O

J;OLÜTIONS

' l ' l re So lu t ion Process

The forces of attracLion are less in pure Br2 (1) thanin pure Iz (s), because of the lower atomic nurnber ofBr, corresponding to lower London forces.

lo.2 (a) CH3OH, which can hydrogen-bond to H2O.

(b) NaCl, which separates into ions, each highlyhydrated.

(c ) CH3F, wh ich is more po la r , thus more l i ke H2Owith regard to intermolecular attract ions.

| ( ) . 4

l o . 5

Whether one melts or dissolves 12 the same inter-molecular attract ions must be overcome; furthermorethe interaction of Iz and CCla molecules is sl ightand not very dif ferent from the Iz - 12 interactionin l iquid iodine. However when ionic substancesdissolve in water, very large hydratj_on energies arel iberated to compensate part ly or entirely for theenergy required to melt the crystal.

Smal1 i-ons, highly charged ions.

1a¡ Fe3+, more h igh ly charged. A lso s l igh t ly smal le r ,but this is a minor dif ference.

+( b ) L i ' , m u c h s m a l l e r .

(c ) F , smal le r .

(d ) SN2+, a b i t smal le r .

(e) e13+, because of both smaller size and sreatercharge.

z +( f ) MS

' , much smal le r .

Water usually appears in hydrates as water ofhydration of the positive ion. It may also occupyinterstit ial spaces in the crystal structure.Finally, it may be H-bonded, particularly to anoxyanion.

6 0 6 I

1 0 . 7 First one must consider the enthalpy of solut ion neafsa tura t ion . I f i t i s negat ive , i .e . , the so lu t ionprocess is exothermic, then raising the temperature}owers the solubility in accordance w"ith Le Chateprinciple. By shif t ing back to the left the reactionwill absorb the heat one supplied in attempting toraise the temperature,

L O . B - 4 7 k J m o l e - r

_ - lI U . I - U - / U 5 K J M O J - E

1 0 . 9 - 1 5 k J m o l e - l

This value applies to the combined processes:+ +

N a ' ( S ) + N a ' ( a q )

c r (S) + c1 (aq)

which release energy; as well as the process ofseparating r¡rater molecules from each other toaccommodate the ions, which required energy.

I O . I I - - 7 g 2 k J * o 1 . - l

This value applies to the combined processes:

n¡* (9 ) + nb+ (aq)

F ( g ) + F ( a q )

1 0 . 1 2 . 0 4 8 1 m o l e s , 2 . L 2 9 C O 2

1 0 . 1 3 . 0 0 4 0 9 m o l e s , 0 . 1 8 0 9 N 2 O

ConSentratj-on_of Solutions

IO.14 Temperature-independent measures are: percentagecomposit ion, mole fract ion, and molal i ty. Temperadependent measures are molari ty and normali ty.

1 0 . 1 5 M = > < r n ( 1 - y l 1 0 0 )

1 0 . 1 6 x ( c H 3 o H ) = . 2 9 4 ¡ x ( H z o ) = . 7 0 6

1 0 . 1 7 X ( C e H s o H ) = . 2 L 7 ; X ( C 2 H 5 O H ) = . 7 8 3

1 0 . 1 8 X ( u r e a ) = . 0 3 2 3 I O . 1 9 7 I . 4 %

L O . 2 O 1 3 . 3 e " 1 O . 2 I 7 . 0 1 q K O H

1 0 . 2 2 1 4 . 8 9 K M n O 4 1 0 . 2 3 5 O . 6 9 H B r 3 3 . 7

1 0 . 2 4 ( a ) 5 7 . 6 9 ( b ) 3 3 . 9 m l

I 0 . 2 5 ( a ) 5 . 5 1 M ( b ) 6 . 9 3 m

1 0 . 2 6 ( a ) 3 . t 7 6 M ( b ) 3 . 6 0 m

t . o . 2 7 3 . 3 5 M t o . 2 8 0 . 7 8 9 M

| o . 2 9 2 5 . 5 m l 1 0 . 3 0 B . l t m l

1 0 . 3 1 0 . 3 1 6 M 1 0 . 3 2 O . 4 O O M

1 0 . 3 3 X s o l u t e = . 0 8 4 3 , X s o l v e n t = . 9 1 5 7

1 0 . 3 4 ( a ) 2 N a 2 0 2 ( s ) + 2 H z o ( I ) + 4 N a o H ( a q ) + 0 2 ( 9 )

( b ) . 0 7 3 3 M

Pressure Boi 1i Po in t o f So lu t ions

.265 Atmos.

.171 Atmos.

_ - lt Q . ¿ q m o l e

( a ) P _ = 1 . 1 1 7 A t m o s .A

c . 1 A

0 . 8 8 6

- t

Y¿-¿ g moJ -e

1 0 . 3 6

r - u . 5 t

1 0 . 4 0

( b ) . 8 3 8

t t ) . 4 2 ( a ) . 4 3 4 A t m o s .

(b ) Ac tua l p ressure ( .400) i s less than idea l ( .434) .Negative deviat ion from Raoultrs Law.

(c) Heat must,be evolved when the l iquids are mixedsince intermolecular forces are greater than in thepure l iquids.

(d) Since the vapor pressure is lowered a highertemperature is required to boi l and so, i f anazeotrope is formed, j_t must be a maximum boil ingazeotrope.

6 2

10 .43 (a ) . 497 Aünos .

(b) Vapor pressure shows a very strong positivedeviation from Raoultrs Law.

(c) Heat must be absorbed upon preparing the solutionsince there is less force of attract i-on betweenmolecules in the mixture than in the pure liquids.

(d) Mixtures boi l relat ively easi ly. These two could,form a minimum boilinq azeotrope.

1 0 . 6 5 T = 2 7 . 1 A t m o s .

A pressure o f 27 . f A tmos. on the sa l t water wou ldjust maintain equi l ibr ium in the rate of f low of hraterbetween one side of the membrane and the other.

_Addit ional pressure would be necessary to favor theflow from the salt side to the fresh side, the rateof f low increasing with increasing added pressure.

sglut-isJnq o-f_ Electrolytes

10.66 i = 3 ions per mo lecu le

1 0 . 6 7 í = 2 . 6 7

1 0 . 6 8 t f = - ¡ . 4 0 o C

1 . O . 6 9 t f = - . 2 4 2 " C

1 0 . 7 0 % i o n i z a t i o n = 3 . 5 %

LO.44

1 0 . 4 5

LO"47

1 0 . 4 9

1 0 . 5 1

ro .52

(a) 9og mole- l

3 3 3 9

- 1 . 7 5 " c

L22.59 mole-r

(b) .611 Atmos.

1 0 . 4 6 1 4 . 8 9

10.48 -11.8%C,/m

l - 0 . 5 0 I 5 4 . 3 9 m o l e - l

1 0 . 5 4 2 7 . 0 g

10 .56 62 .A g mo le I

10 .58 186 9 mo le - l

1 0 . 6 0 3 3 . 6 9

Ii i i iO.62 334 g mole-l

(b) 109 mm

J-75.69/moIe round off to 176

1a) PUz = .800 Atmosr nO, .200 Atmos.

( b ) x w z = L . 4 8 7 x l o - s , * o . O . 7 g 7 x l o - s

( c ) - . 0 0 2 3 6 o C

1 0 . 5 3 2 . 7 7 e " C / m

1 0 . 5 5 1 6 1 . 1 4 C

1 0 . 5 7 6 9 . g g m o l e - '

Osmotic Pressure

10 .59 2 .9 I A tmos .

10 .6 I 67 ,OOOI mo le -1

10 .63 (a ) . 01055 A tmos .

L0.64 1BO g mole- l

64 65

( l t^ t , , t ' i lR f f

REACTIONS IN AQUEOUS SOLUTION

l letathesis Reactions

1 1 . 1 ( a ) z n s ( s ) + 2 H - + H z S 1 g ) + z n 2 +

( b ) s r 2 + + c o 3 2 + s r c o 3 ( s )

(c) l¡o reacti-on

(d) ¡¿g2* + 2OH + Mg (oH) 2 (s )

( e ) p o , * t - * 3 H * + H ¡ p o +

L . 2 ( a ) e c - + I - ' A 9 I ( s )

( b ) s o s 2 * + 2 H + + H z o + s o z ( g )

(c ) No reac t ion

( d ) s r 2 + + 2 o H + N i 2 + + s o ¿ + 2 - * s r s o + ( s ) + N i ( o H ) z ( s )- +

( e ) O H + H ' + H z O

1 .3 ( . ) pbz+ * H2s (g ) + pbs 1s ¡ + 2H+(b ) ga2+ + s2 * zn ' * + soa2 - - * Baso+ ( s ) + zns ( s )(c) ¡ ¡o react ion

(d) Hgzcoals) + 2u- + 2cr -> Hg2cr2(s) + H2o + coz (g)(e ) Fe (oH) g ( s ) + H3pO4 + Fepo+ ( s ) + 3H2o

L.4 (a) 2NHa+ + So4' - * ¿u '* + 2oH -> caso,+ (s) + 2Hzo

+ 2NHs (g)

(b) No reaction) + a -

( c ) C d - ' + S - - + C d S ( s )

( d ) F e z ( c o s ) s ( s ) + 6 H + + 2 F e 3 t + 3 c o 2 ( 9 ) + 3 H 2 o(e) pb2+ + so42 + pbso,+ (s )

Oxidat-En-Reduc ti o-n Reac tio_n s

11.10 (a ) zn is ox id ized f rom 0 to 2+ ; Zn is the reduc ingagent r C12 is reduced f rom O to l - ; C12 is then v i ¡ l i o i - n .rgenc .

(b ) ReCls i s reduced, Re goes f rom 5+ to 4+; ReCl5is the ox id iz ing agent . SbC13 is ox id ized . Sb goesf rom 3+ to 5+; SbC13 is the reduc ing agent .

(c) Mg is oxidized from 0 to 2+; Mg is the reducingagent . CuC12 is reduced, Cu goes f rom 2+ to 0 ,CuCl2 is the ox id iz ing agent .

(d ) No is ox id ized , N goes f rom 2*reducing agent. O is reduced fromox id iz i -ng agent .

(e ) I {O3 is reduced; W goes f rom 6+ox id iz ing agent . H2 is ox id ized ,I + ^ i q J - L r o r a ¡ l r r ¡ i ¡ ¡ . . J agenr . .

Oxidation Numbers

1 1 . 5 C l 0 3

c l 0

C1O'+2 -

CrOa2 -

soe

1 1 , 6 ( a ) 6 + ,

( s ) 3 * ,

1 1 . 7 ( a ) 2 * t

( s ) 5 * .

I 1 . B ( a ) 3 * ,

( s ) 3 t ,

1 1 . 9 ( a ) 5 * ,

(s) 5* ,

C I 3 + ; O H N 1 f ;

I \Tq+. \T^^ \ r :L ' v ¿ , " 3 + ;

Mn7+ ; COs- C4+¡2-

cr6+; so,* 2-

s6+;

As5+ ; po , * 3 - p5+ .

( d ) 1 + , ( e ) 4 + , ( f ) 2 - ,

c15+;r . l 1 r .

CI1+¡

54+;

( b ) 6 + ,

(h) 5+

(b ) r - ,( h ) 6 +

(b) 3 f ,

( h ) 5 +

( b ) 6 + ,

( h ) 6 + ,

Cl0z

Nos

MnOa

Cr zoz3 -

AsOa

( c ) 5 * ,

( c ) 6 + ,

( c ) 4 + ,

/ A l a ¿

( d ) 5 + ,

( c )

t i I

4 * , ( d ) 6 + ,

6 +

( e ) 5 * , ( f ) 6 + ,

( e ) 4 + , ( f ) 5 + ,

( e ) 6 + , ( f ) 6 + ,

Eo 4+¡ NO is theO to 2 - ; 02 is the

to 0 ; WO3 is theH goes f rom 0 to l+ t

6 66 7

I t . l I (a ) C lz i s reduced f rom O to 1_ ; C l2 i s the ox i -d iz ingagent. NaBr is oxidized, Br goes from _l to O, NaBris the reducing agent.

(b ) Zn is ox id ized f rom O to 2+; Zn is the reduc ingagent . HCI i s reduced, H qoes f rom 1+ to 0 ; HCI i sthe oxidizing agent.

(c ) Fe2O3 is reduced, Fe goes f rom 3+ to 0 ; Fe2O3 isthe oxidizing agent. A1 is oxidized from O to 3+iA1 is the reduci_ng agent.

(d ) OF2 is reduced, O goes f rom 2+ox id iz ing agent . H2O is ox id ized ,0 ; Hzo is the reduc ing agent .

(e) HgO is both oxidized and reduced and is bothreducing agent and oxidizing agent. The Hq isreduced from 2-r to 0 while the O is oxidizád from2- to O. The reac t ion is usua l ly ca l led a"decompos i t j_on"

ra ther than, ,ox j -da t ion_reduc t ionr , ,wh ich i t i s techn i -ca l1y .

I . I2 (a ) 2H2O + 4MnOa + 3C102 -+ 4MnOz + 3C10u + 40H(b) BH+ * Cr2o72 + 3H2s + 2cr3 t + 3s + 7H2o(c) 6H2O + p4 + IOHOC1 + 4H3pOa + 10Cf + lOH+(d) 3cu + 8H- + 2No3 * 3cu2f + 2NO + 4H20(e) PbOz + 4Hf + pblz + 12 + 2H2O

1.13 (a ) 4sb + 4H+ + 4No3 + Sbao5 + 4No + 2H2o

(b) Bnar + 5H2Soa + HzS + Atz + 4Na2SO4 + 4H2O( c ) 2 I O s + 4 H 2 O + 5 S O z + I z * 5 S O a 2 - + B H +(d) 2NF3 + 3A1C13 + Nz r 3Cl2 + 2A1F3

(e) AsaO5 + 4CI2 + IOH2O -+ 4H3Asóa + BHCI

L . L 4 ( a ) 6 F e 2 + + 6 H + + c r o 3 + 6 F e 3 r + c l _ + 3 H 2 o(b) 3p t r 16H- + 4No3 + lBCl + 3p tC1s2- + 4No + 8H2O(c) cu + 4H+ + soa2- + g r r2+ + so2 + 2H2o

(d) p¡ + pbo2 + 4H+ + 2Soa2 + 2pbSo+ + 2H2o(e) MnO2 + 4HI + MnI2 + 12 + 2H2O

(a) 24Ag- * 4AsH3 + 6H2o -> 24Ag + As4o5 + 24H+

(b) 2Mn2+ + 5Bio3- + 14H+ + 2Mno4 + 5Bi3+ + 7H2o(c) 2NO + 4No3 + 4H-r + 3N2O4 + 2H2Q

(d) 2MNOa f 11H+ + 5HCN + 5I -> 2Mn2* + BH2o + 5ICN(e) 3zn + l-2H+ + 2:H2Mooa -> 3zn2t + 2Mo3+ + BH2o

( a ) 2 H + + 2 r o 3 + 4 c l + S 2 o 3 2 - + 2 s o + , - * 2 r c l , + H 2 o(b) 3Se + 2BrO3 + 3H2O + 3H2SeO3 + 2Br

(c ) 5H3ASO3 + 2MnOa + 6H+ + 5H3AsOa + 2Mn2+ + 3HeO(d) Hsroo + 7r + 7H+ + 6H2o + 4r .2

(e) 12n+ + 3pb3oa + 6Heo - r 6pb2+ + 3pbo2

(a) e r - + C103 + 6H+ -> 3rz + c l + 3H2o

(b) 4zn + No3 + loH+ -> 4zn2* + NH++ + 3H2o

( c ) 3 H 3 A s O 3 f B r O 3 + 3 H 3 A s O a + B r

(d) 2H2SeO3 + H2S + H* + 2¡zo + 2Se + HSoa

(e) 4HzO + 2ReO2 + 3Clz - ) 6C l_ + 2HReO¡ + 6H+

(a) 6Fe2+ + c r2o72 + l4H+ + 6Fe3t + 7H2o + 2cr3+(b) 5HNO2 + 2MnOa + H* + 3H20 + 5NO3 + 2Mn2+

(c) 3As2S3 + 5C- r -O¡ + 9H2O + 6HgAsO,+ + 95 + 5C1

( d ) 2 I O 3 * 3 N 2 H a + 6 H e O + 2 I + 3 N 2

(e) 3cu + 2Nog + BH+ - 3cu2* + 2No + 4H2o

(a) p , * + IOHOC1 + 6H2O -> lOH+ + lOC1 + 4H3pOa

( b ) x e o 3 + 6 H r + 9 r + x e + 3 H 2 O + 3 I 3

(c ) 3uo2+ + c r2o72 + BH+ * 3uoz2+ + 2cr3r + 4H2o

(d) BrO3 + 3 !2C2Oa -> Br + 6COz + 3H2O

(e) 4NO3 + 3Te + 4H+ + 2lzo + 4NO + 3TeO2

( a ) 2 4 H - + 3 4 r + H 9 s ( r o e ) z + s H g r { ' - * B r , + L 2 H 2 o

(b) Mnoa + BH+ + 4Mn2+ + I5 l2p2oz2- + 5MnHzpeoz3-

+ 4H2O

( c ) 3 C s ( N H 2 ) 2 + 4 e r o ] + 3 H 2 O ' - > 3 C O ( N H z ) z + 3 S O a 2

+ 6H- + 4Br

to 0 ; OFz is theO goes from 2- to

1 1 . 1 5

I l - . 1 6

1 1 . 1 7

1 1 . 1 8

1 1 . I 9

L J - . ¿ U

6 9

( d ) 2 r J H * * 5 c o ( N o 2 ) . t - * l l M n o f + L 4 H 2 o + 5 c o 2 +

+ 3ONO3 + 11Mn2+(e) z¡t+ + 2cNS + 3ro3 + 6c1 + Heo _F 2cN + 2soa2

+ 3 I C 1 2

( f ) 26H2o * 2Cr r3 + 2 lCI2 _> 2CrO+t - * 610r - + 52H+

+ 4 2 C I

I I .2 I (a ) BOH + 52- + 4 I2 -> Br + So, *2- + 4H2O(b) H2O + 3CN * 2MnOa -+ 3CNO + 2MnO2 + 2OH(c) 4eu + BCN * 02 + 2H2O -> 4Au (CN) 2 + 4OH( d ) S i + 2 O H + H 2 o + S i O g 2 - + 2 H 2(e) 2cr (oH) g + 4oH + 3Bro + 5Hzo + 2croar- * 3Br-

1 7 . 2 2 ( a ) 2 0 H + 2 A I + 6 H 2 O + 2 A 1 ( O H ) 4 + 3 H 2(b) zo¡ l - + s2o3 2-

+ A}CI - + Hzo + 2soa 2-

+ Ac I - �( c ) I z + j C I z + I B O H + 2 H 3 I O 5 2 - + 6 H 2 O + 1 4 C 1( d ) 2 B i ( o H ) 3 + 3 s n ( o H ) + 2 - * 2 B i + 3 s n ( o H ) 6 2 -(e) 3NiO2 + 2Fe + 6H2O -f 3Ni (OH) 2 + 2Fe (OH) 3

1 1 . 2 3 ( a ) 5 H c t O 2 + o H + 3 H z o + 4 C I o 2 + c I( b ) B M n O , + + B O H + I + B M n O a 2 - + I O a + 4 H 2 O(c) p+ + 2H2o + 4oH + 2pH3 + 2 lpo32( d ) s b H 3 + o H + 3 H 2 O _ + S b ( O H ) a + 3 H 2( e ) C O ( l l H r ¡ , + 3 O B r * C o z + N z + 3 B r + 2 H 2 o

LL.24 (a ) 4 ¡ ln (OH) 2 + 2H2O + Oz + 4Mn (OH) s(b) 3c1z + 6OH - ' 5c I r - c lo3 + 3H2o(c) 2HXeO,+ + 2OH + Xe + XeO6a- + 02 + 2H2O

Note: There is no unique ans\¡¡er to this problem; i tdepends on an arb i t ra ry cho j -ce o f the 02 to Xe ra t ios .For ins tancef another answer i s :

4oH + SHXeoa + 6H2o + 66 ,z + 3Xeo5+- + 5X"( d ) 2 a s + 6 o H + 2 A s o 3 3 - + 3 H ,

(e) 4on- + 2s2oaz ¡ oz + 4so32- + 2H2o

1 1 . 2 5 ( a ) g a t + 3 N O 3 + 5 O H + 1 B H 2 O - + B A 1 ( o H ) , *

(b ) 6oH + 2Nj -2+ * Br2 + 2NiO(OH) + 2H2O +

( c ) 3 5 + 6 O H - + s o s z - + 2 s 2 - + 3 H 2 o

( d ) 1 O 0 H + 4 I 2 + S 2 o 3 2 + 2 S o 4 ' - * B I - *

(e ) s2- + 4Ho2 -+ 4oH + so42

LI .26 (a) p¡S + 4H2O2 -> AHzO + pbSOr+

(b) 2Cr (oH) 3 + 3H2O2 + 4OH - t 2CrOq

(c) 2MnOa + 5H2o2 + 6H+ -> 2j/rn2t + 5o2

(d) Ag2O + H2O2 -> 2Ag + 02 + H2O

+ ?I\TH ̂

2F,r

5 H 2 O

B H 2 O

+ B H 2 O

/ \c ids and Bases ; A Bas ic Ox ides

1. I .27 Hydron ium is usua l ly cons idered to be H3O+, thoughin fact more water molecul-es on the average areassoc ia ted w i th i t . T t i s imposs ib le fo r H* to ex is tindependently since there are no core electrons, andso the f ie ld a round the t iny p ro ton is so in tensethat the H+ wil l st ick to anything near i t .

I I t A M ^ h ^ ^ r ^ + ; -I L . z e l r u r r u ¡ r r u u r c á c i d : H N O 3 , H C l

P o l y p r o t i c a c i d : H 2 S O a ¡ H 3 p O 4

Normal sal-t ;

Ac id sa l - t :

N a 2 S O a , K C 1

NaHSOa ¡ KHC2O4

1L.29 (a ) HwO3 + NaOH -+ NaNOs + H2O

( b ) 2 H N o 3 + M g ( o H ) z + M g ( N o s r z

(c ) 3HNO3 + A1 (oH) 3 -> A1 (NOs) e

1 1 . 3 0 ( a ) K O H + H B r - > K B r + H 2 O

(b) 2KOH + H2SOa + KzSO+ + 2H2O

( c ) : X O H + H 3 p O 4 + K ¡ p O + + 3 H 2 O

I 1 . . 3 1 ( a ) 3 N a O H + H 3 P O a + N a 3 p O 4 + 3 H 2 O

(b) 2NaOH + H3pOa + NazHpO+ + 2H2O

( c ) NaOH + H3POq + NaHzpO¡+ + H2O

+

+

2 H 2 O

3 H z O

7 077

I I , l ; l ( ¡ ) b rom ic ac i d (b) potassium bromate

(c) hydrobromic acid (d) sodium nitr i te

( t , ) po tass ium hydrogen su l fa te

(f) potassium sulf i te (g) sodium hydrogen carbonate

(h) sodium dihydrogen phosphate

( i ) copper ( I I )n i t ra te

1 1 , 3 3 ( a ) H C l o a ( b ) H r

(d ) H ro3

( f ) ca (HCo3 ) 2(S) Cu (woz ) z

IL .34 (a ) H2O + C12O -> 2HCLO

(b) H2O + N2O5 + 2HNO3

( c ) S O g f H 2 O + H 2 S O 4

( d ) C O z * H 2 O + H z C O ¡

(e) CaO + H2O + Ca (OH) z

( f ) Na2O * H2o + 2NaOH

( g ) P + O r s + 6 H 2 O + 4 H 3 p O a

I l . 3 5 ( a ) C I z O z , ( b ) N 2 O ¡ , ( c ) s o z , ( d ) B 2 O 3 ¡ ( e ) A l e O g

( f ) z n o , ( S ) K z o

1 1 . 3 6 ( a ) C a ( O U ) z + C O 2 + C a C O 3 + H 2 O

(b) COz + 2OH + HzO + CO32

(c) znO + 2H+ + Hzo * , I - ' *

(d) BaO + SO2 + BaSO3

(e) FeO + 2HC1 + FeClz + H2O

11.37 An amphoteric oxide can be dissolved by either acido r b a s e . z n o i n a c i d ( s e e 1 1 . 3 6 c )

z r r o + 2 H + * H 2 o + z n 2 +

Upon treatment wi_th base:

ZrrQ + H2O i 2OH -> Zn(OH) a2

Uquivalent Weights_and Normal Solutj-ons

L I . 4 9 ( a ) L / a , ( b ) L / 6 , ( c ) I / 5 , ( d ) L / 6 ,

( c ) H I O

(e ) NaHSOs

Volumetric Analysis

t - 1 . 3 B 0 . 3 B 5 B M

l - 1 . 4 0 4 L . 2 B e " M g ( o H ) 2

I I . 4 2 6 9 . 6 2

L L . 4 4 ( a ) . O 4 4 7 5 9 N a C l

1 1 . 4 5 ( a ) M n O a + B H - +

1 1 . 3 9 0 . 0 3 0 5 4 M

1 1 . 4 1 7 2 . 5 % H 2 C 2 O 4

1 1 . 4 3 . 1 1 0 1 M N a o H

( b ) . B 9 s %

t L a + o +

5 F e - ' + 5 F e " ' + M n - ' + 4 H 2 O

- + 3 N z + 6 H 2 O + 2 B r r

- u --> 2 f + S tOe

-

( e ) I / 2 ,

I T . 4 6

r1, .47

t l _ . 4 8

( b ) 2 O . 0 % F e

(a) 3NzH+ + 2BrO3

( b ) 2 4 . 0 % N z H 4

. 3 4 1 % F e

(a) 12 * 2S2Os2

( b ) . L 5 B 6 g t z

( f ) L / 4

t 1 . 5 0 H C l 6 . 0 0 N ¡ H 2 S O a

l . I . 5 f H C l 6 . 0 0 M ¡ H 2 S O q

L L . 5 2 5 7 . 0 m l H z S O ' +

1 1 . 5 4 . 4 6 5 N b a s e

1 . 1 . 5 5 ( a ) 9 0 . 0 9 e q u i v .

I t - 5 6 ( a ) 6 4 . 0 g e q u i v .

t L . 5 7 ( a ) . 1 2 0 0 N

( c ) . 0 1 s 0 M

r . t . 5 8 5 7 . 8 6 % F e

l 2 . 0 O N ' H 3 P O ' + 1 B . O O N

3 . 0 0 M ' H 3 P O a 2 . 0 0 M : '

1 1 . 5 3 . 4 0 9 N a c i d

-1 (b) 1 acidic H/molecule

-l (b) 3 acidic H,/molecule

( b ) . 0 7 5 0 N

7 3

CIIAI"]'ER 12

CHEMICAI KINETICS

t2 . ' l

Energy 4

t ra , r

4

AH

L _

eactan ts

l : l .B Rates are proport ional to the concentrat ions of the

reactants to the po\¡¡er of their coeff icients only i f

the reaction occurs in a single step. Most overal l

reactions represent the sum of several consecutiveqtcns (the 'rmechanism"). Note lhat single step

reactions usually involve only t\^¡o moleculest

occasional ly three, white most balanced reactions

.involve many molecules.

t:1.9 Most col l is ions are ineffect ive because the col l ision

energy ( (Ear f , i .e - r too smal l to "damaqe" the

col l j .ding molecules. Even i f the energy is great

enough the molecules may not be properly orienLed with

respec t to each o ther to reac t r i -e . , key a toms and

bonds are in the wrong Places.

l . l .10 Usually i f a thermodynamical ly feasible reaction does

noc succeed, i t is because i t is much slower than

competing side reactions. In this case the decompo-

sition of perbromate to bromate and oxygen could be

the trouble' or the very rapid oxidation by perbromate

of other species present. By the same token'

perbromates probably are very fast oxidizing agents'

1, , , ,¿

Ll-I l (a) The activated complex contains both reactants in

a single species' enriched by the energy Ea,f, and in

a configurat,ion that can easily revert to either the

reactants or to the Produqts.

- - r_

tIE a , x

- l

(b ) . O4o sec - l

Reaction Rates, Rate Equations

L 2 . t (a) rate = k tNol 2

[H2 ]

(b ) k un i ts a re M-2 se" - l

Note: M, molari ty, symbolizes the

( c ) r a t e = k ' [ N o ] 2 [ H z ] , h " r e k r =

L2.2 (a) Rate of form. of C = ktAl tB l(b ) k = 1 .56 x 1o -2 M- l sec

I

L 2 . 3 ( a ) R a t e o f f o r m . o f C = k l B I 2

( b ) k = . 0 3 3 M - r r . " - t

I2 .4 (a ) Zero order , k un i ts a re M sec- l

Rate = . lOQ M sec r

(b ) F i rs t o rder , k must be in un i ts o f sec- l

Rate o f d isapp. = .0050 M sec- l

(c) Second order, k must be in units of M-l sec

Rate o f d isapp. = 2 .5 x lO 4

M sec- l

1 2 . 5 ( a ) . O O B O M s e c - r

( c ) . 2 o o M - l s e c - r

12.6 (a ) Rate(a) rz ln i t ia l Rate = . l8B

(b) Rate = practical ly zero

(Actual ly [NO] must approach zero asymtotical ly)(c ) Rate(c ) rz ln i t ia l Rate = .074

(d) na te(d) rz rn i t ia l Rate = 4 t imes

(e) Rate(e) ,z In i t ia l Rate = B t imes

un i ts mo le l i te r - l

2k

74 7 5

(b) Earf is the minimun energy necessary to raise thereactants into the activated complex configuration,or the minimum energy of collision which can resultin product formation.

(c) The reaction order ís the sum of the dependenciesof rate upon concentration, i.e., the sum of theexponent,s of concentrations in the rate equation.

(d) A catalyst changes the reaction rate, usuallyspeeding it up greatly, though very little catalystmay be required and it is not consumed in thereaction.

(e) Chemisorption is the process in which moleculesstick to a surface with considerable energy,comparable to chemical bond energies- Chemisorbedspecies are usually very reactive.

(f) The rate determining step is the one step in amechanisÍi that goes so slow1y that its rate isessentially the reaction rate.

(g) A reaction intermediate is a species which isquite unstable $¡ith respect to further reaction.It exisüs in small concentration during the courseof a reaction in which it is being formed and reactedh'ith rapidly.

(h) A zero-order reaction proceeds at a rateindependent of the concentrations of the reactants.

- k2 [No2]k1 [N2os] +- k ths [No] [Nzgs] - - k-2 [No2]kr [N2os]-

kz [Noz] + kg [No]

- krk g [Nzos ] [N-o] -- kz lNoz l + kg [No ]

12.L4 Rate of dÍsappearance of A = rate of appearance of

products

= ( k r kg / kz ) [A ]

12 .15 Ra te o f change o f [ o ] = o = k r l oe l - kz [o2 ] [ o ]

- k g [ o e ] [ o ]

Uropping the k3 term, kr [Os] = kz [Oz] [O)

t o l = ( k r l k z ) I o g l / [ o z ]

Ra te o f d i sapp . o f 03 = k r [ o ¡ ] - kz [ oz ] [ o ] + kg [ os ] [ o ] '

But as stated earlier the first tvto terms above

balance out to zero and rate = ks [og] [o] I

r a te = ks [ oe ] $ t / kz ) l og l / l oz l

= ( l ¡ ¡ k3 , / k2 ) l ogJ2 / l oz ] = k [og ] ' / l o r j

I : r . 1 6

L2.L2 ICI + H2 + HI + HCl s low

ICl + HI + HCl + Ie fast

Net overall reaction is the sum of the above.cancels out; i t is the unstable intermediate.

H I

L2.L3 Rate o f change o f [NO¡ ] .

' k2 [No2] [No3]

= O = k r [ N z O s ]

- ks [No] [Noe]

+ kg [No] )

Ener

kr [NzOs] = [ r ¡Os] (kz [NOz]

I rñ^ = k r [NzOs]kz [Noz ] + k3 [No]

Rate of disapp. o f Nzos = k r [N2o5 ] - kz [Noz ] [Nos ]

_ kz [Noe ] k r INzos ]kz [Noz] + k3 [No]

- T - -

Er=1-0

eacEanEs

= k 1 [ N 2 O 5 ]

76

12.17 Rate o f change o f [No3] = O = k l [NO] toz l - kz lNos j

- k g [ N o s ] [ N o ]

Many substancesi ca11ed "negative catalysts¡ ' arerea l l y inh ib i to rs o r re ta rders , i .e . , they reac t w i thand remove intermediates. But retarders and inhibitorsare consumed i-n the reaction. One can conceive of atrue negative catalyst in a mult istep reaction, i f thecatalyst enhances the formation of an intermediate not.othen^/ise present in the mechanism which is kinetical lys tab le ( i .e . , separa ted by a la rge energy bar r ie r f romthe f ina l p roduc t ) .

I2.2O A homogeneous catalyst is present in the same phaseas the reactants and is regenerated in that phase asproducts are formed. A heterogeneous catalyst is aseparate phase, such as a sol id catalyst for gasreactions. The surface attracts one of the gaseousreactants and holds i t in pl-ace unti l the other gaseousreactant can contact i t . In addit i-on to faci l i tat ingcontact between reactants the catalyst, in cases ofchemisorption, wi l l act j-vate the adsorbed species.

Rate EquatÍons an-d_ Temperature

Neg lec t ing the k3 te rm: k r tNOl [Oz]

[No3] = (k t /kz )

Propagation Cl

cHs

Terminations 2Cl_

Ener

Uncata lyzed Path

CataLyzed Path(usual ly moreconp l ica ted)

= kz [Nos ]

tNol [o2]

= ( k r ks / kz )Rate o f fo rm. o f NO2 = ks tNO: l [NO]

tNol 2

[o2 ]

The above ís f irst order in 02 and second order in NOas requ i red .

12 . lB In i t ia t ion C l -z -> 2CI

+ CH+ + HC1 + CHs

+ C lz + CHsCl + C l

+ C f z i 2 C H 3 + C e H e ;

+ C l + C H 3 C I

L 2 . L 9

L 2 . 2 I 2 6 7

L 2 - 2 3 2 4 8

L 2 . 2 5 3 . O

1 2 - 2 7 2 . 9

12 .29 178

- l

KLI MOIC

- t

kJ mole -

- t - l

M - s e c -

- t

- tmole

-

1 2 . 2 2 1 3 9 . 3 k J m o l e - l

L 2 . 2 4 7 . g x 1 o s M - l s e c - 1

- 3 - 1 - lL 2 . 2 6 4 . 7 x 1 0 - M s e c

L 2 . 2 8 6 6 B K

L 2 . 3 0 5 2 . 3 k J m o l e I

M

KLJI

Ail_ * _

eac é r t L >

A catalyst provides a path with a lower Ea,¡ so that

a much larger fract ion of col l is ions are effect iveand the rate is thereby increased. As shown AH isunaffected by the height of the barrier since weobtain the same products at the same energy leve1 bye i ther pa th . Both E_r , and " . r , . ru decreased by tñe

same amount of energy by the catalyst, so that bothrates are increased by the same factor.

7B79

CHAPTER 13

CHEMICAL EQUILIBRTUM

(s)( h )

( i )

L 3 . 7 ( a )

( e )

( i )

n independent of P, depends only on Tt no change.

Catalysts affect ratesf not Kt no change.

Addit ion of A affects the equil ibr ium concen-

t ra t ions bu t no t K (see (g ) above) ; no change.

r i g h t , ( b ) 1 e f t , ( c ) I e f t , ( d ) n o e f f e c t ,

r igh t , ( f ) no changer (g ) no change, (h ) r igh t ,

r ight

Chemical Equit ibr ium, LeChat-el ier ' q Principle

1 3 . 1 ( a ) K = [ c o ] 2 Í o z l / l c o z J z

L ) . ¿

1 3 . 5

1 3 . 6

( b ) K = [ o 2 ]

( c ) K = [ o z ]

( d ) r = l c s z ] [ H z ] u / l ¡ ¡ " s J t l c r ¡ u l

( e ) r = t c q z / L c o 2 l

(a ) le f t , towards less moles o f gas

( b ) l e f t , ( c ) l e f t ' ( d ) l e f t , ( e ) l e f t

endolhermic 13.4 exothermic

(a) K must decrease

(b) K, and Kp are dependent on temperature only and

do not change \^/ith pressure.

(a) [cJ must decrease

(b) [C] wi l l increase to a greater extent than

expected for an inert mixture of gas.

(c) Adding A at constant volume drives the reaction

to the r ight so that [C] increases. However i f

A is added at constant pressure [CJ would be

decreased by the di lut ion, but to a less extent

than expected for an inert mixture of gas.

(d) Catalyst affects only rates, no effecl on

equi librium concentrations .

(e) Removal of B at constant volume drives the

reaction to the left so that [C] decreases. But

at constant pressure the removal of B wil l

concentrate C in a smaller volume and [C] will

increaser but not as much as expected for an

inert mixture.

( f ) K decreases

Proplems Basgg o-n Equilibriun llonstants

r3.b 61.0 l r3.e) .0942 a1- jm2) \--_,--l

1 3 . 1 0 ( a ) I C I 2 ] = . 0 5 0 M ' I P c 1 s ] = . 0 6 0 M

(b) .042 mole l i ter - l

( a ) [ so2 ] = . 0040 , [ oz ] a t eq - = . 0036

(b) 278 (mole/ l i ter ) I

- f ¡ - t

7.6 x 10 mole l i ter -

- ( - t

B-2 x 10 '

mole l i ter -

- 4 - l5.47 x Lo mole t i ter

-

.0684 mole l i ter

t - 3 .11_

L3 .T2

l - J . l _ J

1 3 . l _ 4

I 3 . I 5

1 3 . 1 6

r 3 . 1 7

lHz l = [ I z ] = . 319 mo les l i t e r

I n r l = 2 .362 mores f i t e r - r

(a ) [Hz ] = l t z l = I . 35 x I o -3

(b) .0127 moles l i ter - r

- 1

mol.es l r fer

( c ) 2 1 . 3 %

13.18 .0253 mo] .e l i te r I

1 3 . 1 9 . O 3 O O m o l e l i t e r l

BO B1

t ^

(a ) [Hzo ] = [ co1 = . 0335 M, [Co2 ] = [Hz ] = . 0665 M

( b ) 3 . e 4 ( c ) 3 . e 4

C}IAPTER 14

.I'}]EORIES OF ACIDS AND BASES

/\cid-Base- _Concepts

f4 .1 (a ) An ac id p rov ides H+(aq) ions . A base prov ides

oH (aq) ions. Neutral izat ion is the reaction between+ -

H' and OH to produce H2O.

(b) An acid provides the characterist ic cation derived

from the solvent. A base provides the characterist icanion of the solvent. In a neutral izat ion thecharacterist ic cation and anion combine to form one

.or two molecu les o f so lvent .

(c) An acid is a proton donor and a base is a proton

acceptor. Neutral izat ion in aqueous medium is the

transfer of a proton from the conjugate acid of water

to the conjugate base of water to form two molecules

o f water .

(d) A Lewis aci-d is an acceptor of an efectron pair

and a base is a molecule with an unshared electronpair whích i t can share with an acid. The neutral i-

zation reaction is the formation of a covalent bond

between acid and base uti l iz ing the base's unsharedp a i r .

l , l -2 (a ) HzO is ne i ther an ac id nor a base j -n the

Arrhenius concept.

(b ) H2O can be e i ther ac id o r base:

H 2 o + H 2 o i H 3 o - + o t t -

ac id l base2 ac id2 base l

(c ) H2o can be e i ther a base ' s ince i t has two

unshared pairs, or a Lewis acid-base adduct. In the

autoprotolysis reaction the acid part of the adductl f l - r a n r n # n n \ a o f q S h i f t e d t O S O m e O t h e r b a s e , i . e . ,\ e ¡ r v r ! v u v

another water molecule in a base displacement

reaet ion .

1 3 . 2 0

L 5 . ¿ L (a ) ONC1 = .436 mo1e. NO = .564 mole ,

(b ) To ta l mo les = 1 .282 mole

(c ) P^- -^ , = .340 a tm, Pr ,^ = .440 a tm 'ONCI NU

(d) .368 a tm

2 . L 9 x l o - 1 0 m o l e l i t e r - r

6 . 7 L x l 0 - e a t m

L 3 . 2 2 ( a )

(b)

L 3 . 2 3 ( a ) f 4 . 4 a t m - '

( b ) x = I . L 2 x

L3.29 (a ) Kp

( b ) - 7 8 1

CI2 = .282 mole

n" r , = .220 a tm

L3.24 (a ) .563 (mole /L í te r )2

(b) K .^ = 8 .77 x 10-4 a tm-2lr'

K = L . ' 7 7 6 ( m o l e / 1 i - t e r ) - 2

L3-25 3 .35 x lo -3 mole l i te r - t

1 3 . 2 6 2 . 4 4 \ 1 0 - 2 a t m

1 3 . 2 7 n * " , = 1 2 . 6 3 a t m , P f O t : 5 0 . 0 a t m , * * n . = ' ' U '

13 .28 n" r . = PnCt , = .75 a tm

Or ig ina l Pec15 = 1 .00 a tm

Percent d issoc . = 75s"

1 O - 3 m o l e l i t e r - l ,

0 ,50 a tn -

atm of NO2; I .22 a ium of N2O4

K = . 0 6 9 4 a t mp

B 3

14.3 Many reac t ions ean be in te rpre ted as redox , e -g- ,

N a H + N H 2 + N a N H 2 + H 2

which in the ammonia solvent system was interpreted

as the level ing of the base H- to the base NHz-, is

also interpreted as an electron transfer betr^teen

H ( 1 - ) a n d H ( l + ) t o f o r m H ( O ) .

In reactions of elementary sulfurr i t can be

considered a reducing agent or a Lewis acid. See

rniddle of page 364 fox such an example, or below:

H - ' S : - + ' é : + H - ' S - 3 r -

Lewis base f,ewis acid

l . 4 . 4

In redox te rms S(2- ) and S(O) reac t to fo rm S(1- ) .

Solvent system: NHaCI + NaNH2 + NaCl + 2NH3

acid base salt solvent

+ -Bronsted: NHq + NH2 -> NH3 + NH3

ac id l base2 ac id2 base l

+Lewi s: NHr*

adduct of

+

T

NHz -> NHg + NH3

base ne\ , r+adduct+displaced base

acid H+ and base NH3

I4 .5 (a ) KOH + HNO3 + KNOg + H2O

(b) CaO + 2H2o + Ca (OH) z

(c ) C lz + H2O + HCI + HOCI-

(d) 2Na + 2H2o '> 2NaoH + H2

( e ) Z n ( O n ) z + 2 N a O H + N a 2 [ Z n ( O H ) ' + ]

| , l .7 (a ) NHz , (b ) Czo+z- , (c ) OBr , (d ) H2Poa

1 , 1 . B ( a ) H g A s o + , ( b ) P H , * r , ( c ) u C z t t s o z , ( d ) H S ,t -

( e ) H P o 4 -

+I , l .9 (a ) NHq r (b ) HzCzO+ , (c ) HOCI ' (d ) NHg

l 4 . l o ( a ) H F + H F ¿ H z F - + F

ac id l base2 ac id2 base l

(b ) HNog + HF ? H2No3 ' + F

base2 ac id l ac id2 base l

( c ) H F + c N ¿ H c N + F

ac id l base2 ac id2 base l

(d ) H2Po4 + co32 i HPo22- + HCo3ac id l base2 base l ac id2

( e ) N z H + + H S O a ? N z H s r + S O 4 2base l ac id2 ac id l base2

( f ) Hc2o4 + HS ? Czo+z- + H2Sac id l base2 base l ac ld2

l . l .1 ,1 See tex t ans \^ ¡ers i o ther examples are :

(a) Hocl + oH -> ocl + H2o

(b) Heo + 02 -> oH + oH

( c ) H C o s * H 2 o ? c o . ' - + H 3 o +

(d . ) NH3 + CHgNHe ? NHr- * CH3NH3+

I t . l 2 ( a ) H z N O H + H 2 O ? H ¡ N O H - + O H5 -

(b ) N + 2NHg - ¡ lNH2-

( c ) H + H 2 o + H z + o H

(d) HSo3 + H2o ? HzsOg + oH

I l . I i (a) An amphiprotic substance may act as a Bronsted

ac id o r base, i .e . r may e i ther donate or accept a

proton.

(b ) Examples are HF¡ HSo3 ¡ N2Hs+, HC2H3O2

: O - H

'l

Lewis base disPlaces iadduct

oH- fron adduct I

Br on s,t-ejl- Iowry _Co.nc ept

L4 .6 (a ) H2AsO3 ¡ (b ) HAsO+ 2- ,

B 4

, . +1 { - o - I i + 0 H

l l1-l r

new adduct ldisPlacedI b a s e

( c ) N O 2 , (d ) s

B5

: O -

@

1 ¡ \ . ¡ i . O

ioH-Ó-se-Ó-H" t "

. ,9 ,o

L4.L4 When an acid is stronger

of the solvent (cation)

that cation.+ -

H C I + H z O + H 3 O + C l

Al1 acids stronger than

so they are " level led"

The same language rs

are stronger than thethe so l ven t ¡ € .$ . r

o z - + H 2 o + o H + o H

H + H 2 O + H 2 + O H

All bases stronger than

14.15 AmPhiPro t ic re fe rs to a

as e i ther ac id o r base '

(usuallY not soluble in

solut ion either bY acid

f

(a ) Ac id sequence: H3o

(b) Base sequence: NH2

Y e s ' ( b ) N o , ( c )

Acid sequence: HSO4

Base sequence: OH

( in anhydrous l iq . H2so

than the characterist ic ac

it is converted comPletelY

r ^ n ñ ó v ñ é n fr v v Y e ¡ v * _ _ *

H3o- su f fe r the same fa te '

by the solvent-

used with regard to bases whi

charac ter is t i c base (an ion) o f

100 Percent

100 Percent

OH are level led to OH '

substance that can function

An amphoteric substance

water) can be brought into

or bY base-

) H3POa > HCN > HzO > NHg

> OH > CN > H2POa > HzO

r,l" Streng-th ang Mgl-ecu]_ar Str_u-cture

. . i ( ) S ince s j -ze is no t chang ing , the grea ter the e lec t ro -negativi ty the stronger the acid. Strength increasestoward the r igh t : AsH3 ( H2Se ( HBr

Size increases goj-ng down a group. This is dominante f fec t so ac id s t rength íncreases l Í kewise .

H e S < H 2 S e < H 2 T e

H 3 P O a ' ( b ) H 3 A s O a , ( c ) H 2 S O a , ( d ) H z C O g ,

9 -

' (d ) NOz , ( : ) so

, ó , Ot . .

: I -O -H

t "" o

(b ) HC10+ is s t ronger than HCl03 s ince i t has alarger formal charge on central atom. But HIO3is s t ronger than H5IO5 fo r the same reason,despite the fact that the oxidation number isgrea ter in the la t te r .

( c ) a c i d : H F + H 2 O

base: HF + HF

a c a d : H > U 3 r

base: HSO3 +

- . - . Í ,a C L O : I \ 2 r 1 5 r

f

b a s e : N 2 H 5 +

a c i d : H C 2 H 3 O 2

base: HC2H3O2

? H 3 o * + F -

? szr ' * + ¡ ' - ( in l iq . HF)

H 2 o ? H 3 o + + s o s 2 -

H z o ? H z S o g + o H

Hzo 7 Nz !tu + Hgo+

? N r H . 2 * + c 1

f

t

+n U I ¿

+ N H 3 Z N t t + ' + C 2 H 3 O 2

* H2SOa I u2c2n3o2* * nsou

N o , ( d ) Y e s

> H C 2 H 3 O 2 >9 -

> C O g - > H S

H2S > HCO3

->

' . . ) . ( a )

( e )

( c ) s a u 3

,ó, ot . .

: C l - O - H

t "' 9 ' oo

( a )

1 . .cl -o-H

I' 9 ' o

H

l s'? ' . /",ó-fó-H

o bljóil "H

H H\ x ' . ' ^ /.X ,V:.

H-O-Te -O-H. . . , \ ¡ . .zY'

'Y\

H H

( a )

I 4 . I O

T 4 . I 7

1 A 1 9 )

t / 4 l o

(b ) HeSeO+ is a s t rong ac id , hav ing a 2+ fo rmal chargeon cent ra l a tom. H6TeO6 is weak, hav ing zeroformal charge on central atom.

( b )

( b ) Y e s , ( c ) Y e s , ( d ) N o( a ) N o ,

B6

> C2H3O2 > SO,r

a 1

The Lewis Concept

L4.26 (a) The carbon atom is an acid

a higher hYbridization it can

electrons' SH- has 3 unshared

a Lewis base '

(e ) sF is the ac id ; H is the

its electron densitY has been

by the hightY electronegative

L4.27 (a ) Be is the ac i 'd center i

hYbrídization i t can accept

F- i s the base '

(g ) E lec t roph i l i c . The ac id A1C13 d isp laces CH3

(h) Nucleophil ic. The base OH displaces I .

t 4 . 2 9 ( a )

(b) H+ must be the Lewis acid' Fe acts as the base

center . A I l i t s 3d , 4s , and 4p orb i ta ts a re f i l l ed

but 4 of al l the ¡á'pairs are unshared and one can be

used for bonding whe-n re goes from dsp3 to d2tp3

hybridization.

(c ) Ag+ is an ac id w i th 5s .and

NH¡ is a base; unshared Paar on

center. BY going to

accept a pair of

pairs on S to make i t

5p orbitals empty'nÍtrogen.

!

H' i s the

point of attack because- so thoroughlY drained

F . F - i s t h e b a s e '

by going to a higher

more e lec t ron Paars '

cenuer.t o s p 3 .

. ^ : ^b a s e . u r >

from sp2 to sP

@ ' . . . @ . , r . tH-O:r*- r$=Q:+ i ' I -O-S=O: ->

I U ' I L it { iot FI :O:^

o " \ i '*z

I I - O - S = O :" l: o :IH

(d ) Th is case is i soe lec t ron ic w i th (b ) ;

ac id an Mn(CO) s - i s the base '

(b) In the f irst step the base center, O, donates apair of electrons to the electrophi le S. In the

second step the nucleophilQ, O-, attracts a proton

in an internal Lewis acid-base reaction.

(c) ahe new structure formed by the migration reaction(step 2) is more stabl-e because al l formal charges are

(b) Hzo is a base and B is the acid

"".upl i"g a Pair from o i t exPands

By

(c) s atom has an incomplete octet ' must be an acid'

t ' - tra= four unshared Pairs' a base'

(d) H- has one unshared p4ir ' a strong

acid center; can accePt a Pair and go

(e) Au is ac id center i CN is a base '

L4.2a (a) Nucleophil ic ' The base H i l isplacesf _ .

NHz .f

Noz .(b ) E lec t roPh i l i c ' the acid H' i l isPlaces

(c) Nucleophilrc ' The base Cl displaces H2O'

(d) Consider GeS2 an adduct of the base GeS and the

acid :s: . Then t ie aispfacement is nucleophil ic '

the base Ge disPtaces GeS'

(e) nucleophil ic ' The base 02- displaces oH

( f ) E lec t roph i l i c ' The ac id FeBr3 d isp laces Br

BB

q o

CHAPTER 15

IONIC EQUILIBRIUM' PART I

- 5 = ¡ o H - l ; [ c o H s N H z ] ' 3 0

1 5 . 1 3 3 . g x l o a

1 5 . 1 5 I ' 0 x 1 O - 3 t " t

t N ; l = o . l 3 M ; [ H N a ] = o ' 1 0 M

, , , l l r "a t ion o f Water , PH

t , - 2 L ( a ) [ H + ] = 2 . 0 x 1 o - 2 M ; i o g - l : 5 . o x l o - r 3 l ¿

( b ) t o H - l = . 0 4 0 M ; [ H + ] = 2 . 5 x 1 O - r 3 u

t " . 2 2 ( a ) O . 4 6 t ( b ) 1 3 . 1 8 ' ( c ) B . 7 8 ' ( d ) 7 . 6 0

t ' , - 2 3 ( a ) 4 . 1 4 , ( b ) l . O B , ( c ) 1 0 . 5 2 , ( d ) 1 2 - 6 2

t " - 2 4 ( a ) 4 . 7 x l - O 4

M , ( b ) 2 . 1 x 1 0 - r r

M ,- 3

( c ) 1 . 3 x 1 0 M

- 1 4( a ) 3 . 5 x 1 0 ! 1 , ( b )

( c ) 1 . 3 x 1 0 -

M , ( d )

l - l x l o

. r t

r t . 3 6

p H : 6 . 5 7 t o p H = 8 . 3 5

( a ) 9 . 6 < p H < I O . 0 , ( b ) 5 . 5 < P H ( 6 - 0 r

( c ) 4 . 5 < p H < 5 . 0

, r l n r ( ) n - I on E f f ec t , Bu f f e r s

r ' , r . ' l ( a ) p H - 4 . 3 5 ( b ) o . r B %

( b ) . 4 6 e "

- 6

1 5 . 3 7 3 . 2 x l - 0 -

1 5 . 3 9 0 . 7 1 M

1 5 . 4 f 4 . 5 x 1 0 - 2 t ' l

l c z H s o z l / I H C ú t s o z ] = 6 . o / L . O

' ^ " ' T t

/ f r t r r I - ^ t rr r l n 4 l / [ r l n 3 ¡ - . . . 6

Vüeak Electrolytes

- +1 5 . 1 l - 5 x l 0

- 3I 5 . 3 3 . 9 x 1 0

- 71 5 . 5 7 - 3 x 1 0

1 5 . 6 ( a ) 0 ' 0 8 2 M

- 4

: l . 5 . 7 4 . { x l 0 M

- ?

1 5 . 8 ( a ) 3 ' 5 x 1 0 -

M

- 3

1 5 . 9 ( a ) I ' g x 1 o

+ -15. lo (a ) [ r i - ] = loc r ]

( b ) 0 . 1 1 2 m o l H C I O z

( b ) 1 . 8 e "

( b ) . 7 2 %

= 8 . 0 x l O - s ; t H o c l l = ' 2 O

- 2" ? X

2 . 3 x

l 0

- 51 0

t ' , - 2 5

t " - ) - 6

t , , . , 1 8

1 , ' . t 0

l " . \ 2

| , . J l

- 5

4 . 8 x 1 0 -

1 1 ,- l ?

4 . 6 x I 0 - "

M

- 31 5 . 2 7 3 . r x l 0

- F ,

1 5 . 2 9 7 . 3 x l 0

- 6

1 5 . 3 1 2 . 5 x l _ 0

L ) . L ¿

15 .14

l 5 . l - o

I f , . I '

L ) . r o

1 ( l q

1 R ? n

(b) o - 04e"

+

l C o H s N H g I =

. I B M

0 . 3 I %

t H ' l = l ' 5 x

- 4

3 . 9 x l 0 M

1 . 2 x l 0

rH+r = '-i I ':^l^.:J'i.:ll::i ;:i::""::'ffi"i:::":-10 M benzo ic acrd

lNH,*+ l = 2 .7 x l0 -5 M; A l t the NHac l reac ts to p

. i i - * ' N H 3 a n d l e a v e s ' 1 0 M o H - '

[Nn, *+ ] = 5 .4 x lo -s M; A l l the NHa+ reac ts to p roduce

. I S - * - N H 3 a n d l e a v e s ' 0 5 M o H - '

t ' , ( a ) 3 . 8 0

r { , 5 - 5 x 1 0 -

r i t O . 7 0 M

l i ) 4 . 8 5

| 5 . 4 I

90

t ¡

91

- 3

1 5 . 4 5 7 . 8 x I 0 -

m o l

Polyprotic- Acids

t s .46 (a ) t co r l 1 -63* ' [H* ] = [HCo3 ]

t c o ? - l = 4 ' a x 1 O - r r '

( b ) 3 - 9 2

L5 .4 '7 [H - ] = tH2Aso ; l = 8 ' 5 x 10 3 M ;

¡Heso i ) = 5 ' 6 x ro -8 ; t aso i - l

1 5 . 4 8 ( a ) t s ' - l = 4 ' 9 x 1 o - 2 r M

(b) [Hs- ] = '7 '4 x ]o-8 ¡ ' t

L 5 . 4 g t s 2 - l = 4 . 4 x 1 0 - 1 6 M

15 .50 [ i l + ] = 1 ' 9 x l o -3 l ' l

Ions that function as Acids an

= 1 . 2 x 1 0 i

[ H g A s O + ] = 0 . 2 9 M

= 2 x I O - '

M

t l / \ t rTER 16

I i }N IC EQUIL IBR IUM, PART I I

I l ' , . q ñ l r r t r i I i l - r ¡ P r O d U C t-:-::=.--: ]---.--.- l !

l r , . L 2 . O x t 0 - '

_ o ^

. 3 2 . O x f O ' "

CuCOg (molar so lub i l i t y , 1 .6 x 1O-" l¿ ) i s J -ess _uso lub le than Ag2CO3 (molar so lub i l i t y , l - .3 x 10 M) .

CuS (molar so lub i l i t y r 8 .9 x l -O- re) i s less so lub lethan Ag2s (molar so lub i l i t y , 1 . I x 10- i7 ¡¿) .

- l L

l t , . l 2 . O x 1 0 - '

- 7l r , - , ) 5 . 6 x I 0 m o l A g 2 C O 3

t r , , ip Í ta t ion and Ksp

t + - 1 1l - 6 . 8 I N i - 1 = 3 . 3 x l 0

- - M

l ' , - l t ) T h e f i n a l c o n c . o f N a - i s 0 . 1 6 M a n d o f C I i s O . 3 O M1 l - h a < a i n n q ¡ f a ñ ó f r a a . l - )\ e ¡ ¡ e s u ¡ ¡ v v ! v s v s , .

t - - a

l B a ' ' l = O . O 7 O M ; t c 2 b [ 1 = 2 . L x I 0 M

r r , . I I t N O ; l = . 1 5 M ( N O l d o e s n o t r e a c t ) ;

Final- [ tr ta-1 = . OrO (no reaction )

A f t e r r e a c t i o n [ s r 2 + ] = . 0 3 0 ; t r I = 1 . 6 x I 0 - 4 l ¿

r r , l . l i r I = 9 . 5 x 1 O - + M 1 6 . 1 3 8 . 3 x 1 O - B

+I r , | , ' r tNn[1 = ! .2 . M 16. 15 tNu i l = . 21 M min imum

¡ ¡ , l ( , [ N H ? ] = 3 . 6 x l O - 3 M m i n i m u m

I t [ N H 3 ] = . 1 8 M

l r t C a ( O H ) 2 w i l l n o t p r e c i p i t a t e .

1 6 . 2 3 . 7 x r 0 q

7 6 . 4 2 . l - x 1 0 "

M

M

1 5 . 5 1 B . t B

1 5 . 5 3 8 . 6 1

1 5 . 5 5 0 . 6 0 M

1 5 . 5 7 4 . 3 x 1 0

1 5 . 5 9 5 . A 2

lgid-Base Titrations

1 5 . 6 1 ( a ) 3 ' 9 2 t ( b ) 8 ' 4 6 '

L 5 . 6 2 @ ) 9 . 4 4 , ( b ) 5 ' 2 B l

- 6

1 5 . 6 3 6 . 2 x r 0

L 5 . 6 4 2 2 - 0 0 m l

L 5 . 5 2 2 . 8 2

1 5 . 5 4 4 . 5 0

1 5 . 5 6 7 . L x 1 O - 2 M

1 5 . 5 8 2 . 9 x I O -

1 5 . 6 0 . o 7 B M

@ ) L 2 . L 6

( c ) I . 7 8

9293

Precipitat ion of Sulf ides

16.19 No Prec iP i ta te fo rms '

L6.20 l lo PreciPitate wil l form'

L6 .2L CdS wi l l Prec iP i ta te '

L6 .22 [ t l * ] = o .z3 (min imun)

L6-23 [H+] = 0.47 minimu¡n

t 6 . 2 4 [ P b 2 + ] = 2 . 3 x 1 o - 7

L 6 . 2 5 [ H + ] = o . 7 O M , l S 2 - l = z . z

l c , r t + l = 3 . 6 x l o - r s M ' [ F e

16.26 [H- ] must exceed 0 '086 M'

16 .27 (a ) tn+ l = .30 min imum

( b ) [ P b 2 + ] = 3 x 1 0 - 7 ¡ t

Complex Ions

l . 6 . 2 8 ( a ) 2 - 4 x

( b ) 1 ' 4 x

( c ) 1 ' 9 x

L6.29 AgCl w i l l

l0-2 mol,/ l i ter- ?

1 0 - M- 5

l 0 M

not p rec iP i ta te .

- 2 2x 1 0 M '' ' l = 0 . 3 0

I IAPTER I7

I I,I.JMENTS OF CHEMICAL THERMODYNAMICS

' t ' l rrr Fir j ; t Law, In_terIral_Energyr_ Enth_alpy

I l .L The f j - rs t law essent ia l l y s ta tes tha t energy isconserved during a physical or chemical changer butmay be t rans formed. I t i s usua l ly s ta ted :AE = q - w, where AE is the change in internalenergy content of the system, q is the heat absorbedand w is the work done by the system during thechange. Entha l -py , H, i s de f ined by t ¡ = E + PV. I tdif fers from E by including the PV product of thesys tem (wh ich a lso has un i ts o f energy) .

| 1.2 A state function has a value dependent only on thes ta te o f the sys tem ( i .e . , i t s tempera ture , p ressure ,vo l -ume, compos i t ion , e tc . ) and is independent o f howthat state was achieved. A1l the state functions inChapter L'7 are slmbolized by upper case letters.T h e y a r e z P , T , Y , E , H , G , a n d S .

( a ) - 1 3 6 4 . 3 k J , / m o l

( b ) C z H s O H ( 1 ) + 3 0 2 ( g ) - >

a h l - - l _ J ( ) b . ó J < J

( c , - ¿ t t . 9 K J l m o r

(a) -1170 kJ lmo l

( b ) c s ( N H z ) z ( s ) + 3 0 2 ( g )

2 c o z G ) + 3 H 2 o ( l )

Since An = o , AH" =

-92 ;2 kJ , /mo l

-3911.8 kJ lmo l

+ coz (g ) + so2 ( s ) + Nz ( s )

+2HeO (1 )

A E o = - 1 1 7 0 . 0 k J

C e H r z ( 1 ) + 9 0 2 ( 9 ) + 6 C O z ( g ) + 6 H z O ( f )

AHo = -3919.2 kJ /mol -

( c ) - 1 5 7 . 2 k J / m o L

AEo = -5459.55 kJ

/ ¡ \

I t . ' , ( a )

( b )

94

r'

L7 .7 AE" = -1405 '8

L7 .g AEo = -726 '66

17 .11 +81 .69 kJ lmo l

t ? _ l ?

L7 .8 aE" = -66 -57 kJ

17.10 -LBA-79 kJ lmol

S i n c e a l l s u b s t a n c e s i n c l u d i n g e l e m e n t s h a v e a b s o l u t eentropies, there is nothing to be 9ii":1^?Y..tabulatinformation values- (Such values could easi ly be

calculated from the formation reactions and the

abso lu te en t roP ies ' )

L 7 . I 4 ( a ) + 1 0 3 . 0 J / K ( b ) - 5 8 7 ' 6 k J

1 7 . 1 5 ( a ) - 4 9 . 2 J / K ( b ) - 1 4 s 9 ' 7 k J

L7.L6 F i rs t reac t ion AGo = +267 '4 kJ

Second reac t ion AGo = -207 '0 kJ

S e c o n d r e a c t i o n i s s p o n t a n e o u s ; f i r s t i s n o t .SOz and H2 react lo produce HzS and HzO'

L7.L7 AG" = -675-5 Yes , i t i - s poss ib le '

17 .18 BF3 is s tab le to hydro lys is ; BCl3 w i l l hydro lyze '

L \ .Lg (a ) s ince AGo fo r the reac t ion is pos i t i ve ' i t i s

not sPontaneous'

( b ) A G " = - 4 1 ' 6 k J

Spontaneous reactiont yes preparation is possi

L '7 .20 (a ) None of these preparations is spontaneous at 25

because AGo is Pos i t i ve '

Since Aci is less Posit ive for

the combustion of NO to NO2 is

general ly the decomPosit ion of

to the elements is spontaneous96

| / . 2 I A G o = - 4 8 . 3 9 k J ; y e s , s p o n t a n e o u s

| / . .¿2 AGo - -467 kJ¡ yes , spontaneous

I t . . . 3 ( a ) - 1 0 1 , 0 1 , ( b ) - L 2 O . 6 J / K , ( c ) - 1 2 0 . 6 J / K

| / . ) .4 +63.6 kJ,/mole

1 / . ) 5 ( a ) A G o = + 3 8 . 3 k J S i n c e A G " i s p o s i t i v e , i t i sNOT spontaneous.

(b ) Ac ' = -11 .8 kJ Yes , i t IS spontaneous a t 300"C.

11. .¿6 (a ) AGo = +130 kJ No, no t spontaneous a t 25oC.

(b) Aco = -26kJ Yes , spontaneous a t 1000oC.

1 I . . ' .7 (a ) AGo = -139.9 Yes , spontaneous a t 25oC.

(b) AGo = +2I .4 No, no t spontaneous a t l200oC.

l / . . : 8 2 3 9 . 7 K o r - 3 3 . 4 " C

! I . . ' . ' ) 3 4 2 . 2 K o r 6 9 . 1 o C

1 / . r 0 + 1 8 . 2 5 k / m o l

I / . | | ( a ) - 1 2 8 . 1 k J , ( b ) - 3 3 2 . 3 J / K , ( c ) - 2 9 . 1 k J ,

( d ) - 1 6 6 . 4 k J

I t . t . , . 2 o a J / K 1 7 . 3 3 2 0 5 . 5 J / K

I t . t4 So (d iamond) = 2 .438 J /K mol . S ince S" (g raph i te ) i s5.694 J/K moL, the diamond crystal is more ordered(Sova lue lower ) .

i / r ' , - 1 0 9 5 . O k J

; I l , t ,r ; . t¡ree Energy and_ Equil ibr ium

I i r { , 5 . 1 2 x 1 O - " 1 7 . 3 7 0 . 4 4 3

I i i l r 1 . 4 4 x I O G I 7 . 3 g 2 . O 7

| 1 . l r ) - 5 5 . 2 k J , / m o l L 7 . 4 I + 3 8 . 5 k J

l / | . ' A c o = + 7 g . g k J ; K = 1 . 0 * , O - t u$/

KJ

Áu

I7.L2 A change is spontaneous i f i t results in an increase

in the total lntropy of the universe' I f one observe

only the "V"tt*, tirá criterion is that it \n/ill change

sponcaneouufy i t i ts Gibbs free energy decreases'

Since there are no absolute H and G scales i t is

convenient to tabulate formation values for compound

I

ILI

(b )NOz than for NO,

spontaneous. But

the nitrogen ox

i n a l l c a s e s .o?

L l . + 5

L 7 . 4 4

+27 kJ

@ ) - 2 A . 4 8

( d ) 3 3 . 6

l l ; r l " f E R l B

} I , I I ( ' , IROCHEMISTRY

K J , ( b ) - 4 2 . 6 7 J / K ' ( c ) - 1 6 . 7 5 k J ,

' ¡ , l r , l t r c t i O n

In metal l ic conduction moving electrons carry the

charge; j-n electrolyt ic conduction moving ions carrythe charge.

As a metal is heated the vibrat i-ons of the atoms about

their latt ice points interfere with el-ectron f l-ow and

res is tance increases (conduct ion decreases) . As asolut ion is warmed it becomes l-ess viscous, ions canmove more f ree ly . and i t s conductance increases .

Anions are attracted to the anode.

Oxidation occurs at the anode.

The anode is pos i t i ve ly charged.

Electrons move from some species (usually insolution) into the anode and thence to theexternal po\¡/er supply.

r olyt ic Ce11s, Quanti tat ive Re]atignFhips

Anode

cu*cu2*+2e

( a )

( b )

( d )

9B9 9

,{

I U . b ( a ) cathode 2H2o + 2e + Hz *

Anode 2H2O2+ O + 4H+ +

Cathode 2H2O + 2e -> Hz *

Anode 2CI -> CL2 + 2e

Cathode crr2* + 2e + Cu

Anode 2CI -> CL2 + 2e

(d) cathodu cu2+ + 2e + cu

Anode (inert) 2H2o -> 02 1

20f1

4e

zví

+4H' + 4e

( b ) . 8 6 7 A

( b ) 4 . 2 9 m i n

I 8 . 1 6 2 8 . 6 s e c

1 8 . 1 8 2 8 . 3 7 9 C u

(b)

( c )

18 .7 . 5709 l l i

lB.B g io+ + 2H+ + 3e + Bi + Hzo '9759 Bi

18.9 (a) pb2+ + 2 lzo + Pboz(s) + 4n+ + 2e

(b) .558 g Pbo2

(c\ 46-6 minutes

1 8 . 1 0 1 0 . 0 6 I A g

1 8 . 1 2 2 8 8 m i n -

1 8 . 1 3 ( a ) 7 B o c

18 .14 (a ) . 15659 N i

1 8 . 1 5 3 3 3 . 8 I c

18 ,17 9 .21 l i t e r

1 8 . 1 9 1 . 1 9 M

I B . 2 O ( a ) 5 . 0 0 l i t e r

La .2 r ¡ cu2+ l = 0 .358 Mr

L8.22 5.77 hours

L 8 . 1 1 2 8 . 6 m i n '

i

I

( b ) . 8 9 3 M

t c l - l = 0 . 7 1 7 M

r¡ , , L ta ic Ce l ls Electrode Potentials

M g 'Anode

t.LMgrMg-'+2e

5" (ce11) = Eo(ca thode) - E" (anode) = .799 - ( -2 .363)

=+3 . l - 62 V

c d - _ Pr , C1-2

Cathode

CLr+2e'+2CL-

Eo ( ca th )

AgCathode-L

Ag'*e +Ag

Anode

cd*cd2++2e-

E" (ce1 l ) = - E ' (an ) = 1 .3595 - ( - . 4029 )

= L . 7 6 2 4 v

l r t . . l s ( a ) 2 . 2 7 v , ( b ) s n z + + M g + S n + M g 2 + ,

(c ) Sn e lec t rode i r @

l r r . . 2 6 ( a ) . 5 8 7 V , ( b ) C u 2 + + N i - > C u + N i - 2 + ,

(c) Cu electrode i" @

I u . . '7 (a) pr I t , (") | r- (.ql l l cr- 1aq¡ I cr, (s) I n.( b ) E " = . . 8 2 4 0 y

(c) The cf2 lc1- e lect rode is the cathode.

r , 1 r r r (a ) p t I n r ( s ) l n * ( *s ) l l n r - l 1aq ¡ l " . r ( r ) l n .( b ) 1 . 0 6 5 2 v

(c) The anode is u2 lH+

I i l . r , ) - 1 . 7 8 9 V 1 8 . 3 0 . 9 8 7 V

r01

1 8 . 3 1 - 3 2 0 v

L8.32 (a) suitable oxidizi"g "s""li ":".t:,:::

arrows (?) and below Fe-' ' but above

o"fY o""" ' in APPendix E are (reducedq + 2 . +

4 r

parenuhes is ) : g t t * 1c t ' - ) t cd '

(cd) ;

Pbso+ (Pb ) '

(b) cea+ (cd3+) ; Hocl (c le) ; Pboz (Pbso+) ;

NoT stable.

In

Yes, and the product wi l- l- be In3*+ + + 3 +

3In + 2 ln + ID , 2 In + 6H -> 2 In * 3H2

2 ln + 3C12 + 2 In3* + 6C1

Tl-- rs stable.+

T1

Yes, T l -r ? + +

3T1t + 2,rr + Tf "-,

2T1 + 2H- -> 2Tr- + H2,+ 3 + + -

2TL + 6H' + 2Tt- + 3}lzt 2TL + Cl2 + 2TL + 2CI '3 +

2Al + 3CL2 -> 2'¡1- + 6C1

No, (b ) YES, (c ) NO

- 1 . 2 0 8 V

t i2+ rs s tab le .

t i wi l l react with H+ to give Ti2+.

l . B O B V

Co2+ wil l NoT disproport ionate.

co wil t react with H+ to produce co2+.

+ . 0 4 0+ + 2 + 4 +

2uOz + 4H -> uoz + 2H2o + u+

wil l disproport ionate.

L . 3 9 7 V+

YES Au' wi l l not disproport ionate.

Au wil l not react with H+.

- 3 . 3 9 6 V

No, Eu2+ wiII not disproport ionate.

YESt +

YES, Eu- w i f l be fo rmed.

left of theT

Tl ' . The

form in

and

- 3 7 ( a )

( b )

( c ,

/ ¡ \

. 3 8 ( a )

( b )

( c )

t q ,

. | e ( a )

. . ¡ 0 ( a )

( b )

( c )

. , 1 | ( a )

( . l ] /

( c )

r . r ( a )

( k )

uoz

r r ( a )

(b )

( c )

l t ( a )

( b )

( c )

( d )

+

Au, 1au)

(c) Invert the procedure of part (a);

ca (ca3+) i c r1c r3+ ) ¡ zn (zn2+ ' , - - - . t , - , 3 t r

( d ) e u + ¡ a u 3 + ) t c t ( c r z ) ; c r " ' ( C r 2 o 7 ) ' � r r \ r '

18 .33 ( - ) ag+ (A9 ) ; No ¡ (Nzo ' � * )

(b ) r r " ' ( r r )

(c) Hzs (s) ; nz (s+) t Pb (Pbz+)

(d ) S (Hzsos )

18.34 (a) The react ion wi l l NOT go as wrÍ t ten '

(b) H2o2 + 2As+ -> 2H+ * 1?

* o'3 +

( . ) Rg+ + Fe' - + Ag + Fe

(d) No

(e ) HzSog + 2H2S + 35 + 3Hzo

18.35 (a) Pbo2 + 2CL + 4H+ -) Pb2+ + CLz + 2H2o

(b) No

(c) 2NOs + BH+ + 6r + 3rz + 2No + HzO

( d ) NO r

(e) 2Mnor+ + 3Mn2+ + 2l1zo + Sllnoz * 4H

18 .36 (a ) Hg + H92+ -> Hgzz+

(b) No

(c) No

(a) 2un3+ -t NIn + 3N1n''h + ^ - 2 +

( e ) s n ' ' + s n + 2 s n

L021 0 3

,ff

I

Gibbs Free EnergY and einf

rB .4s (a ) pb leusoa ( s ) l so42 - l l e r2+ leu(b) pb2+ * so42 + Pbso+ (s)

( c ) + . 2 3 3 v

( d ) - 4 5 . 0 k r

re .46 (a) Aglasr(s) l r - l lae+les(b) es+ + r- ? Agr

( c ) - 6 4 7 v

(d ) -62 .4 kJ

L8.47 (a) Pt lo , ts l lon- l lu* loz tg¡ Ip t

( b ) o . B 2 B V ' - 7 9 ' 9 k J

, ¡ * r r f( a ) P t l " r t s l l n - l l " ' l o r t s l l e t( b ) 1 . 2 2 9 v , ' 4 7 4 . 4 k J

( a ) 0 . 2 9 4 3 v ' ( b ) - 5 6 . 7 9 L k J ,

( a ) . 5 4 7 v , ( b ) - 2 1 I ' 1 k J ' ( c )

( a ) . 3 9 3 5 V , ( b ) - 7 5 . 9 k J ' ( c )

( a ) - 5 0 9 . 5 k J , ( b ) ' 4 3 ' 5 k J m o l e

- l

-963 .1 kJ mo l -

( a ) 1 2 5 . 2 k J , ( b ) 1 . 7 6 5 v '

( c ) H z o z , c o t * , s z o e 2 - , C � 3 , F 2

K = 7 . 2 x 1 0 3 l - 8 - 5 6 K = 2 ' 7 x 1 0

- 1 4- I J

K = 9 . 9 x 1 0 " " o r ^ r 1 . 6 x 1 0

l 1K = 1 . 2 5 x L O " 1 8 . 5 9 - . 2 8 4 V

I r r . 6 l ( a ) 2 . I 5 7 V

(b) M9 + N i2+ * Mg2+ + N i

(c ) l ¡ i i s lhe * e lec t rode.

1 8 . 6 2 ( a ) 2 . 2 4 0 7 v

(b) zn + C12 * zn2* + 2CI

(c ) zn is negat ive .

¡ r r . 6 3 ( a ) - . 0 0 6 V

(b) Because of the departure of the actual con-centrat ions from the standard. states, the actualvoltage is opposite in sign from Eo so thereaction proceeds in the opposite direct ion fromthat predicted from the Eo. Spontaneous reactioni s :

2 + TH 2 + P b - ' P b + 2 H

(c) The ca thode is the pos i t i ve e lec t rode, wh ich inth is case is Pb.

1 ¡ r . r , 4 2 . o 4 M 1 8 . 6 5 . 0 2 3 9 M

1i l , { , ( i (a ) NOT spontaneous.

(b) Now reaction IS spontaneous.

l i l . r , / + 1 . 1 9 V

iu . r , r t (a ) When ¡ t " t2+1 is doub led the reduc t ion po ten t ia l

inc reases (a lgebra ica l l y ) by .0089 V.

(b) Cutt ing [ lut2+] in half lowers the voltage by. 0 0 8 9 v .

] u . r , r ( a ) + . 0 1 0 V

(b) The concentrat ions at the end depend on therelat ive volumes of solut ion. I f the volumesare equal and both solut ions are 1.00 M to start,then

l s n 2 + l = 1 . 3 7 0 M ; [ P ¡ 2 + ] = 0 . 6 3 M .

on the other hand i f the volume of sn2* greatly

exceeds pb2+, then [srr2+] wir l hardly change while2 +

l P b I w i l l g o t o . 4 5 9 M .

I B . 4 8

I B . 4 9

1 8 . 5 0

I t , . ) I

18 .52

1 8 . 5 3

l ó . f , q

1 8 . 5 7

1 8 . 5 8

( c ) - I 2 1 . B

+ 7 2 . 6 8 J K

- 7 2 . 4 k J

- l

JK

The Nernst Equatlon

2 +1 8 . 6 0 R e d u c e [ F e - ' l , r e d u c e P " . r t n c r e a s e

n 2

104

f

105

If vice versa' [Pb+] "/iU hardly change and

lsrr ' * l wi l r 9o Lo 2 'L77 t4 '

1 8 . 7 0 ( a ) . 1 3 6 v

(b ) p t lH r I tH* t = .o25 l l [H* ] = s ' oo I n2 le t

( c ) . L75v

. 0 1 6 3 V

Ga (an) + Ga3+ (ca t ) ? Ga3+ (an) +

The anode, i .e . , the Ga in contac t

Gat* , i s the negat ive e lec t rode '

+

L8.72 The discharge reaction consumes H2Soa' The H -

combine with the o in PbO2 to form water while SOa

goes io for¡n PbSO+ on each electrode' As H2SOa is

removed the densitY is reduced'

1 8 . 7 1 ( a )

(b )

( c )

Ga (ca t )

w i t h . 3 o o M

106

TER 19

NONMETALS, PART I: HYDROGEN AND THE TALOGENS

¡ ly ( l rogen

l ' , - l w a t e r ( H 2 O ) , h y d r o c a r b o n s ( C H q ) , l i v i n g m a t t e r .H2 can be obtained from water by reduction with C,hydrocarbons, iron; or by el-ectrolysis of v¡ater.

2Na + 2H2O -> 2NaOH + H2

3Fe + 4H2O -> Fe3O4 + AHz

Similar reactions can be writ ten in which theproduc ts a re FeO and Fe2O3.

(c ) zn + 2H+ * rn ' * * n ,

(d) Zn + 2OH -> ZnOz2 + Hz

( e ) C + H 2 o + C o + H 2

( f ) cH4 | H2o + co * 3Hz

( g ) C a H 2 + 2 H 2 O + C a ( O H ) 2 + 2 H 2

l ' r . ] ( a ) H z r 2 N a + 2 N a H

(b) He + Ca + CaH2

(c) Hz + c12 + 2Hc l

(d ) 3H2 + Nz + 2NHs

( e ) C u z o t H 2 + H 2 o + 2 C u

( f ) co + 2H2 + cHsoH

( g ) W O ¡ + 3 H z + W + 3 H e O

(a) Salt l ike hydrides are ionic crystals, hard sol ids. . i + L L ' i - L * - .wrL¡ ¡ ¡ ¡ayr r * , . y . r and conduct e lec t r i c i t y whenmolten. The hydrogen is a negative ion.

(b) Interst i t ial hydrides resemble the parent metalin structure and propert i-es. The hydrogen ispresent as single neutral atoms in interst ices inthe metal latt ice.

(c) Conplex hydrJ-des are salts. The negative ion hasa central metal atom (e.9. B) surrounded byhydrogen a toms (e .9 . BH4- ) .

( a )

( b )

ütr- -

(d) Covalent hydrides are formed with all the non-

metals and are usually simple l iquids or gases

(e .g- HCl ) ; bu t w i th B , C, and S i compl ica ted

structures can result (e.g' hydrocarbons l ike

C s H r s ) .

19.5 Since Hz exhibits the least London force' H2 is the

most volat i le stüstance aside from Het and is not

very soluble in l iguids'

(d) 6Cr- * Cr2O72- + I4H+ -> 2Cr3* + 7H2O + 3C1z

(e) OK for Br2 and 12 but not for F2 since Eo (red)of F2 is more positive than that of any of theoxidizing agents above.

l e . L 4 ( a ) C a F 2 ( s ) + r ¡ e s O 4 ( 1 ) + r * ( n ) + C a S o a ( s )

2t{F(r) 4F- H2 * F2

(b ) 2Nac1 (1 ) " I t t t t , 2Na + c12

- (c) 2Bx + C12 + 2Cl- + Br2

(d) 5NaHSO3 + 2NaIO3 + Iz * NaHSOa + 3Na2SO+

* H2SO¡+ * H2O

(See answer key in text for an ionic reaction)

(e ) PBrs + 3H2O + 3HBr + H3PO3

These sol ids are very effect ive because of their

abi l i ty to dissolve hydrogen in the form of atoms'

thus making them very available to react with other

substances adsorbed on the surface.

1 9 . 6

1 9 . 8

1 9 . 9

1 9 . r 0

. 5 0 3 9

2 . O L 6 g o f

2 6 . 8 7 g o f

8 - 9 2 g o f

5 8 . 8 7 I o f

- 2 7 0 k J

-44 kJ

A H - o t s :f

-244 (b) ¡

( a ) (b) . e58s

H 2 ( b ) 2 8 . 8 9 g o f A i r

l i f t , 13.33 t i¡nes i ts own $/eight

( a )

( c )

( a )

( c )

( a )

( c )

(d )

lij

L l

I

l li

(b ) 32 .43 g o f zn

( a )

(b )

( c )

( d )

( e )

(s)( h )

( i )

H2 t C l '2 - ) 2HCI

Zn + C12'+ ZnCL2

2 P + 3 C L 2 + 2 P C 1 s

2 3 + C l - 2 + S z C 1 z

HzS * 2C!2 -> 2HCl + SC12

c o + c 1 2 + c o c l z

SO2 + CL2 -> SO2C12

3I + CL2 -> 2Cl + 13

H2O + CL2 -> HOC1 + HC1

- l - l

mole b l '244 kJ mole- l

mole -

H f " , - 2 6 9 v s - 2 7 0 ( a ) ; H z O r - 2 4 1 ' 8 v s

N H 3 ¡ - 4 6 . I 9 v s - 4 4 ( c )

- t1 0 l l A"" = -Bo5.B kJ mole

Result is very close to NaCl latl ice energy

that the lattices are similar in arrangement

The larger value for NaH is consistent wÍth

smaller size of H- compared to C1-'

L9.L2 (a ) 26 .09 g o f H2 (b ) 289 '8 l i te rs o f H2

The Halogens

r ' r - f6 (a ) 4Hr + s io2 + s iFr+ (g ) + 2H2C-

(b) Na¿COa + 2HF + 2NaF * COe + H2O

(c) HF + KF + KHF2; contains FHF ion

(d) CaO + 2ItE -> CaF2 * H2O

(See answer key for some alternatives and some ionicreactions)

+ F ( a )

i ¡ 'ur- (H bonded) (b)

indicatiand size

the

( a )

( b )

( c )

2CL

2CT

IOCI

* MnO2 +

+ PbOz +

+ 2MnOa

+ 2 +4 H + M n +

+ 2 +4 H ' + P b +

- + l 6 H i + 2 M n

l0B

2H2O + CL2

2f12O + CL2t * * B n r o + 5 c 1 2

. - L

nr <- t1

F + H F

1 9 . t 3

109

tw*

HF is a weak ac id , Eq(a) , bu t in very concent ra ted

solutÍons, reaction (b) removes F- and drives reaction

(a) to the r ight. At moderate and 1ow concentrat ions'

reaction (b) is unimportant and i t behaves l ike a

nonnal weak acid'

l g . l 8 o n e m u s t u s e c o n c e n t r a t e d H 2 S o a t o d r i v e o u t t h ev o } a t i l e h a l o a c i d , a n d c o n c e n t r a t e d H 2 S o a i s a s t r o n g

enough oxidizing agent to oxidize Br- ' to Br2' The

result is a mixture of FIBr and Br2 in the product gas'

The above two Ag

CI2 + 2Br '> 2CL

CI2 + 3I -> 2CI

Also very di lute

t 9 . 2 4 2 8 . 2 F

1 9 . 2 5 8 8 6 g o f C 1 2

l ' ) .26 L323 g o f CI2

salts doe not redissolve in

+ Bxz 1i9ht brown

* Is deep brown

Is * starch + blue complex

N H s .

l

19 . 19

Lg.2O XX 3 is an AB3E2 molecu le t o f

PYramidal Pairs' two tr igonal

giving a T-shaped molecule'

( a )

(b )

( c )

FeCl2 ' Fe I I I more cova len t than FeIT

RbCl, GrI more metal l ic than GrII

BeF2, F more electronegative than Cl. so i ts

compounds are more ionic.

(b) 2NaCl + 2H2o

(c) 2NaCl + 2H2o

(d) 6NaCl + 3H2o

(e) r¡aclOg + H2O

L9.22 (a ) HCf ' (b ) CIz '

( d ) F z , ( e ) C l e II

' L

1 q . 2 3 c a - ' + 2 F + C a F z ( s )

the 5 tr igonal bi-

pairs are unshared,

2Na + C12

e l e c t r . ^ ," * * - - - ) C L 2 + H 2 + 2 N a O H

elec t r , H2 + Nac l + Naoc l

e l e c t r . ^ - -< Jf lz | ¡rqi l- + NaClOg (s)

e l e c t r . - - , i , ^ ^ r A+ f l 2 f l \ d u t v 4

(c ) HF (u bond ing e f fec t ) ,

( f ) c l z

XX s j-s ABsE; 6 octagfonal pairs' one unshared;

square PYramid.al molecule 'I

XXt , pentagonal bipyramid, no unshared pairs'

Lg.2L (a) 2NaCr e lect r )

Ag* + cl ->

AgCl + 2NH3

Ag- + Br ->

Ag+ + Í- ->

AgCl (s) white

-> Ag (NH3 ) ,* *

AgBr (s) cream

AgI (s ) ye l low

c1

color

1 1 0 111

W

l l

l i i

i ,

IL

l

l

i

( e ) 2PbS + 3O2 + 2PbO + 2SOz also

reactions which produce higher oxides of pb sucha s P b 3 O 4 .

C H 4 + O z + C + 2 H 2 O

2 C H a + 3 O 2 + 2 C O + A H . O

CH+ + 2O2 -> CQz + 2Hza

CHAPTER 20

THE NONMETALST PART 11: THE GROUP VT A ELEMENTS

2 0 . 5 ( a )

( b )

( c )Oxygen

2O.L Free oxygen in air ' in H2O' in carbonate and sil icate

rocksf in l iving matter' Get oz by l iquifaction and

fractional-ái"i irr"t ion of airt also (minor) by

electrolYsis of water'

2O.2 (a) 2HgO '> zÍtg + 02

(b) heating 2Na2O2 +

also 4H9o -> ztlg2c- + 02

2 -or aqueousr 2O2

(c) 2NaNOg -+ 2NaNOz + Oz

(d) 2KC1O3 -> 2KCL + 3oz

(e) zt¡zo *9SE* 2H2o + 02

2 O . 3 ( a ) r * 0 2 + ¡ 9 '

(b ) 2 t ¡a * 02 + Na2O2

(c) 4Li + 02 + 2Lí2O

(d) 2Mg + oz + 2ugo

(e) 4Hg + oz '+ 2lgzo' also 2Hg + oz + 2llgo

( f ) B a * 0 2 + B a O 2

(g) C + Qz + COz' also 2C + Oz -> 2CO

( h ) S + O z + S O z

2 5 + 3 O z + 2 S O g

( i ) 2cueo t oz + 4cuo

(minor Product)

20.6 IA oxides are hard high*me1ting ionic compounds;VIA oxides are gases, l iquids, or low melt ing, and

.. are covalent. IA oxides react hrith water to givebases , V IA ox ides g ive ac ids .

, ' . O . 7 ( a ) O e ( b ) o z @ l O z 2 -2Na2O + 02

+ 2HzO + 4OH + Oz

+ 5Oz + P4Ol 0

+ 10H2O

+ 6HeO + SOz

olr o*

+ + +Tftr TiJr

a-t-

^

olr

++ t+,JI¡t

lT¡k

++ ++ ++ ++1T ?T 1I 1T

+ tTtr Tftr

t+ t+1 T f i

f+

BC*2¡2 unpr.e

+O2

rJñ

BO=fá ; l unpr .e BO=1¡ 0 unpr .e

O2

d *

T *

++ñ

eo:2L¡ I unpr.e

O *

+ + +Tt?t

'lTr<

++U

BO=Iá; 1 unpr .e

r+ +J,II 1T

t+,IT

tTr ¡t

t +1I

( j ) P q + 3 O z + P 4 O e ¡ P +

2O.4 (a ) 2CqHro + 13Oz + BCOz

(b) CsHr 23 + 9O2 + 5COz

(c) 2CgHsO + 9Oz -> 6COz + BHzO

(d) 2ZnS + 3Oe -> 2ZnO + 2SOz

T L 21 1 3

2 -2 0 . 9

2 0 . 1 0

(a) Pb * H2O2 + SO4 + 2H

(b) 2Cr (OH) 3 + 4oH + 3H2O2

(c) 2MnOa + 5HzOz + 6H- ->

(d) AgzO t H2o2 '+ 2Ag + Oz

+ PbSO+ +2 -

+ 2CrO4. L

2Mn * 5Qz

+ H 2 O

2HzO

+ B H z O

+ B H 2 O

r 0 . 1 6 ( a )

( b )

( c )

( d )

( e )

( f )

( a )

( b )

( c )

( d )

( e )

C ¡ 2 H 2 2 O 1 r ( s ) + H 2 S O a ( 1 ) + f Z C ( s ) + 1 1 H z O ( 1 )

+ H2SOa (aq)

N a N O 3 ( s ) + H 2 S O + ( 1 ) + N a H S O q ( s ) + H N O 3 ( 9 )

C u ( s ) + 2 H 2 S o a ( 1 ) + c u s o u ( . q )

* S O z ( S ) + 2 H z o ( I )

z n ( s ) + H z S O + ( a q ) - + Z n S O a ( a q ) + H 2 ( g )

z n S ( s ) + H 2 S O a ( a q ) + Z n S O 4 ( a q ) + H z S ( g )

F e 2 0 3 ( s ) + 3 H 2 S O + ( a q ) - + F e z ( S O + ) s ( a q ) + 3 H 2 O6 o

. / \ , ' "

Bo=L4 , Type ABrE molecule' one unshared pair'

bent shaPe

i

2 O . I I ( a ) - 2 8 5 . 9 k J m o l e - l o f H 2 o

( b ) - 3 3 3 . 3 k J m o l e I

o f H 2 o

General ly reactions with ozone are

than with oxygen'

20 . I2 2gg.5 kJ mole- l

Sulfur Selenium, 4q-fel lel ! .1$

more exothermÍc

underground and melts the sul-fur

w i th the a id o f comPressed a i r '

2 5 + 3 O 2 + 2 S O g

2 - .(aq,)

2 -

. 1 0 . 1 7 ( a ) s + F e + F e s r F e s + 2 H + - > F e 2 + + H 2 s

( b ) S + O z + S O z , S O z + H 2 O + H z S O s

( c ) N a 2 S O 3 ( a q ) + S ( s ) + N a 2 S 2 O 3 ( a q )

( d ) S + O z + S O z , 2 S O 2 t 0 2 + f g Q . ,

N-OH( .q) + SO3 (9) + ¡ ¡aHSOs (aq)

( e ) S + o z + S o z ¡ 2 S o 2 * 0 2 + f g Q r ,

SO3 + H2SOa -> HzSzOt

None of the above answers i-s unique. See forinstance ans\^¡er key in text.

. L B 2 S O 2 + O z + 2 S O s

SO2 + CL2 -> SO2C12

soz (g ) + H2o (1 ) ->

3soz (g ) + C103 +

Soz ( s ) + oH ,^ : , ->t q 9 ,

2 -

20.13 The c rys ta ls o f Sg mel t to a mob i le f lu id bu t i t s

v iscos i ty inc reases w i th fu r ther inc rease in

temperature as Ss r ings open and form Sx chains'

At very high temperatures various fragments boi l off '

S o , S q ¡ S 2 P a r t i c u l a r l Y '

( f ) so2 ( s ) + so3 (aq )

, ^ . , O . i - a - ' d . O\ l 3 /

: : : : : :( b ) n : O : S = O :

s / " I n r: O : V

( c ) , 4 , O. . 1

I,¿, o (6

H 2 S O 3 ( a q )- 2 -

6 O H + 3 S O q + C l + 3 H 2 O

HSog (aq)

+ H 2 O + 2 H S O g

AB2E2 t lpe, bent

AB3E type, tr igonal pyramid

(3 resonance fo rms)

ABa type, tetrahedron

resonance forms)

2O.L4 Hot water is PumPedwhich is forced uP

2 0 . 1 5 ( a ) S + 0 2 + S O z

minor amounts:2 -

+ S ( a q ) + s 22 -

* S O 3 + S z O s

F e + S - ) F e S

S + 2F2 -+ SF+, S * 3F2 -+ 5Pa

23 + CL2 '> SzCIz

S + 4HNO3 + SOz (g) + 4NO2 + 2H2O

/ f \

( s )

a l s o

s ( s )

S ( S )

@ . .

e,,o/o\,

r14 1 1 5

,B ,O, o = l = 0 ,

manv resonance forms

AB4 tlE)e, tetrahedron

(6 resonance forms)

2 O . 2 O ( a ) c H s c ( N H z ) s * H 2 o + c H s c ( N H z ) o + H z S

(b) SO2 + H2O + HeSOs

( c ) S O r + H 2 O + H z S O +

(d) A tesa + 6H2O + 2A1 (OH) 3 + 3HzS (g)

(e ) SeO¡ + H2O + HzSeO¡

( f ) TeOg + H2O + HzTeO+

(S) HzO + HzSOs -> HzOz + H2SOa

(h) HzO + H2S2O7 -> 2HzSO+

20 .2L 3O2 + 2H2S -> 2H2O + 2SO2

3O2 + 2HzTe -> 2HzO + 2TeOz

2 P b s + 3 o 2 + 2 P b o + 2 s o 2

(a1so get higher oxides of Pb)

2 N a 2 S o 3 * 0 2 + 2 N a 2 S o 4

) o . 2 2 2HC1 (aq) f Na2So3 (aq) + 2NaCl (aq) + Soe (g ) + H2o

2HC1 (aq) + Na2S (aq) + 2NaCI (aq) + HzS (g)

Na2S2O3 (_q)

+ 2HC1 (aq) - ' 2NaCl (aq) + Soz (g )

+ H 2 O + S ( s )

Ihe answer key gives the above in ionic form.

. to.23 (a) 45 + 6oH -> 2s2- * s2o32- + 3HzO

(b) s2o32- + 2H+ -> s + soz(s) + H2o

. ' . 0 . 2 4 S O 2 + S + O o - ,2 - ? -

S O g + S z O s -

Note the expand.ed octet in SF4,which. is permissible for S but nocfor O (on ly 2s , 2p orb i ta ls ,max imum of 4 ) .

' .0.26 Per-acids are polymers formed. by removing H2O fromthe simple acid. peroxy-acids have O-O bonds.

' t¡ .2'1 (a) Add eb2+ la1so many other metal-s) to form a darkcolored, very insoluble metal sulf ide. Or add

H- to form foul small ing H2S gas.

(b) Add H* to form sharp snel l ing SO2 gas.

(c) Add 8.2* to precipitate BaSo+. Other metals,

e . 9 . , p b 2 + a l s o u s e d .

(d) Add acid to form H2S 03 (and eventual ly SO2 gas)and S(s) which forms a rnÍ lky col loidal precipitate.

' t r -28 (a ) H2O2 + HOSO2CI + HCI + HOSO2 OOH

(b) HeOe + 2HOSO2CI + 2HCI + HOSO2O - OSO2OH

Structural ly the above reactions are:

(a) no -G\+ñi)-E -o" + Hcl + no-o-8-ou\Y--l ,, ,,

o o o oi l ^ l l

(b) Ho - ó -G\*fu-4r\+lcr)- s - oH + 2Hc1l \/ \1,-- tlo o

o oi l t l

+ H O - S - O - ' O - S - O Hl t l lo o

L L 7

( e )

( a )

( b )( d )

. ' .0 .25 r - 3 - r/ \

F F

oll ll two tetrahedra (each

tÓ - s - S - i l - s - ót n l ike part d) jo ined"

ll l[ " \7 corner to corner '

i \ . ¿i.i l l ll l l l

- ó - s - ó - s - ó - n. - i l " 1 1

I l u

O : O :

ó : ó 'i l l ll l t l

- 6 - ó - o - ó - s - ó - H. . f l 1 1 "

l l l lO : O :

( f )T\^ro tetrahedra (each

l ike part c) sharing

a corner .

Two tetrahedra (like

part c) joined corner

to corner.

I

( a )

(b )

( c )

r-

( d )

I16

ifl

20 .29 (a ) bent , (b ) bent , (c ) t r igona l p lanar '

(d) tr igonal pyramid, (e) tetrahedron'

(f) tetrahedron, (g) distorted tetrahedron or

,,saw-horse,, shape, (h) octahedron, ( i) octahedron

i s soH I . , ,

(b) ComPare(HO) e ,Te 'central atom-

' l . l The e lements become increas ing more meta l f i c as Zincreases, also stabi l i ty of negative oxidation statesdecreases. Mult iple bonding bet\n/een atoms occurs onlyfor nitrogen, and nitrogen is the only one whichcarrnot expand i ts octet. The hydrides becomeincreasingly less basic rn¡i th increasing Z.

N 2 * 3 H 2 + 2 N H s

N2 + 6Li + 2LisN (alsorother Group IA and Group IIAmeta ls . )

N2 r 02 -> 2No high temperature (arc)

N 2 t C a C 2 + C a N C N + C

N2 is so unreactive because the N=N bond has such a- I

high energy, 94L kJ mole -,

about the strongest bondthere is .

Cer ta in c rops , e .9 . , a l fa l fa , con ta in n i t rogen f i x ingbacteria which return nitroqen to the soi l . Othercrops exhaust i t .

Rea l ly pure N2 a t STP' 1 .250 g l i te r - l

"A i r n i t rog€r r " , L .257 g l i te r

In both cases there is hydrogen bonding between theamine hydrogens and the unshared pair on N' but thereare twice as many opportunit ies in ethylene-diamineand more extended and complex H bonded clusters canr e s u l t .

Pa is most reactive because the units of structureare monomeric, the structure is more permeable toreagent, i t has the highest vapor pressure, and mostimportant the 6OaPPP bond angle is greatly strainedand lowers the bond strength. Black phosphorus, th9most complex and highly pol lmerj-c is least reactive.

. I IAPTER 21

. I ' I I I i NONMETALS, PART I I I : T H E G R O U P V A E L E M E N T S

20.30 (a) HTe is a weaker base than.Hs O": iYt : - i '

much larger' (Same reasonang as Hur 4¡¡u

them in the fo rms: (HO)2SO2 versus .

H2SOa has more lone oxYgens on f,ne

I

I

L i

I

I

I

l lB 1 1 9

ff* - = "-4'''

l

Only Pa is soluble' the polymeric forms are noE'

P,+ , a s t r i c t l y mo lecu la r c rys ta l ' i s an e lec t r i ca l

insulatori t tá i" a poorer insulator; and black is

the besc t""á''"I"t plesuma¡ly within the covalent

bonded Planes as ín graPhite'

2L .7 (a ) P+Oo + 6HzO + HsPOs-

PaO5, + BOH + 4HPO3- + 2HzO

PaO6 + H+ - no reaction

Sb+Oe + 6HzO + no reaction

S b a O 5 + 4 O H + 4 S b O z + 2 H z O

sbao6 + 4H+ + AcL -> 4sbocl(s) + 2H2o

Bi2O3 + H2O + no reaction

Bi2O3 + NaOH + no reaction

Bi2O3 + 2H+ + zCL -> 2BiOCl (s) + uzo

r r "o s izoa + 6H+ + 2B i3+ + 3H2o

(very concentrated acid)

2]r.g GraPhite-l ike BN is:

The diamond-l ike BN has each B at the center of a

tetrahedron of N, and vice-versa. To make a modelt

make a face centered cubic ce11 of B atoms, subdivide

it into B sub cubes and put a N into every other sub

cube.

Prepared from the elements (Haber process). An AB3E

type molecule, shaped l ike a tr igonal pyramid. Verypolar. Acts as a base in water solut ion. Pure NH3

has an unusually high boi l ing point (compared to

other Group VA hydrides) because of extensive hydrogen

bonding.

+ +(a) 2NHs (aq) + Ag + Ag (NH3 ) 2

(b) NH3 (ag) + H* +t NH,**

(c ) NH3 (aq) + H2o + co2 -+ NH4 ' + Hco3

(d) 4NH3 (s ) + 3Oz ts l

he- 2Nz (g ) + 6H2o

( e ) 4 N H 3 ( g ) + 5 0 2 ( s ) P t ' h " l t

4 N o ( g ) + 6 H 2 o

( f ) N H 3 ( 9 ) + H C I ( s ) + N H + C I ( s )

( s ) 2 N H s ( g ) + 2 v ( s ) t " t l

z v l l + 3 H 2

( h ) 2 N H s ( 1 ) + 2 N a ( s ) + 2 N a + + 2 N H 2 + H z ( 9 )

Note : Na+ and NHz d isso fve in NH3 (1)

(b )

( c )

N z *

4NH3

3H2 + 2¡¡ntp f

+ 5Oz :; 4NO + 6H2O

+ 02 -> 2NO2

+ H2O + 2HNOa + NO

* 02 as in th i rd reac t ion , e tc .

3Cu + BH+ + 2NO3 * 3Cu2* + 2NO + 4H2O

2NO

3NO2

2NO

( a )

I I\ - - y ' u - n , / " \ N /

N N' i t l l

a B \ r / u \ * / t -

i l lR r - B -

\ n / " u * / \

¡ l

etc. r \^Iith

benzene-l ikeresonance

(a l te rna t ive ly , ge t NO2 in conc . HNO3)

(b) Azn r lon+ + No3 '> 4zn2+ + NH4+ + 3H2O

( c ) P , * O r o , - \ + 4 H N O s ( 1 ) + 4 H P O 3 ( s ) + 2 N 2 O s ( s )( > , ,

( d ) NH3 + HNO3 + NH+NOg

( e ) c a ( o H ) 2 + 2 H N O 3 + C a ( N o s ) 2 + 2 H 2 O

i on i c : ca (oH)z (s ) + zn l + c^2+ + 2H2o

l 1

L20I 2 I

I ' t i

2 I . I 4 ( a ) N e O r N O ¡ N 2 O 3 , N O 2 , N 2 O 4 ¡ N 2 O 5

(b ) NH+Nog h t t l

, " ro + N2o

N2 + 02 tt !

Zono (or burn NHg on Pt)

N O + N O z + N z O g

2 t i o + o z + 2 N O z

2No2 -+ ¡tgu

P + O r o + 4 H N O 3 + 4 H P O s + 2 N 2 O 5

( c )

. ' Ñ = N = ó ' s : N = t l - ó ' ^O ' ^ ' Ó é " e r

,ñ = ó, * l i j = i : (minor form)o @

, ó = ñ - R = U , + : ó = ñ - R - ó ' oo:dr ü'

,ó -Ñ=[ : * :ó=Ñ-ó ' € , f -b=ó, * :ó=B- i "

H - O - - H H - - H

The acj-d hydrogens are only those attached to oxygens.

| . lB (a ) PaOl s + 2H2O + 4HPO3

( b ) 3 P a O 1 e + I O H 2 O - + 4 H 5 P 3 0 ¡ e

( c ) P a O l s + 4 H 2 O + 2 H a P 2 O 7

( d ) P a O l e + 6 H 2 o I 4 H r p o , *

The above formulas represent (a) inf ini te polymer,o r cyc l i c po l lmer , (b ) t r i_mer , (c ) d imer , and(d) monomer . A I l poss i_b i l i t ies a re represented byH P O , for which the reaction would be:n + 2 n 3 n + 1 '

f n+24-

P+or o * - t - H2o + Hn+zPno¡n+r

l . L 9 ( a ) L i g N + 3 H 2 O - + 3 L 1 O H + N H 3

(b) A lN + 3H2O + A l (OH) 3 + NH3

( c ) C a 3 P 2 + 6 H 2 O + 3 C a ( O H ) z + 2 P H a

(d) CaNCN + 3H2O -+ CaCO3 + 2NH3

(e) NC13 + 3H2O + NHs + 3HOCI-

( f ) P C 1 3 + 3 H 2 O + H 3 P O ¡ + 3 H C L

( g ) P C 1 5 + 4 H 2 O + H s P O q + 5 H C t

( h ) H q P z O z + H 2 O + 2 H 3 P O ' +

( i ) 3NO2 + H2O + 2HNO3 + NO

1 ' o ( a ) H 2 l l 2 O 2 = H z O + N 2 O

(b) 2HNO2 = HzO + N2O3

( c ) 2 H N O g = H z O + N 2 O 5

( d ) 2 H a P 2 O 7 = A H z O + P 4 O 1 e

' 1 . 1 7 nIIt l

H - P

f,

ó ttlil

P - ót "I

II

H

I ItlP - Ól "I

II

H

- o H

i

";' .'b.: o" \ @ @ z

I N -N\ many resonance formso ' .o / \ d ' .

a !Á' 'h ' .c r ' Y . .

@ . . @ . / 2 "' N - o - N' many resonance forms

pr' " -.g:tt

N H a N O 3 + N z O + 2 H 2 O

N H 4 N O 2 + N z + 2 H z O

P b ( N O 3 ) z + P b O + 2 N O 2 + Z O 2

N a N O 3 + N a N O 2 + Z C z

2Nal{3 -} 2Na + 3N2, also 3NaN3 -+ NagN + 4N2

2 I . I 5 ( a )

( b )

/ ^ \\ e /

( d )

( e )

z L . r o (a ) 2NH3 + OCl + NzH+ + C l + H2O

(b) 5Hz + 2H+ + 2Noz + 2NHgOH+ + 2H2o

(c) NHI+F + 2HF + NFs + 3Hz

r22 1,23

{${-

| . 2 4 ( a )

( b )

( c )

( d )

( e )

( f )

| . 2 6 ( a )

( b )

( c )

( d )

( e )

( f )

1 . 2 7 ( a )

t \ - ! \/ \

c i s

Na3As + 3HzO + AsH3 + 3NaOH

A s 2 O 5 + 3 H e O + 2 H e A s O +

l4g3Bi2 + 6H2O + 2B iHs + 3Mg (OH)2

A s 4 O 5 + 6 H 2 O + H s A s O a

2SbC13 + 3H2O + SbzO¡ + 6HCl

D u . T " ¿ - D ü - ! ü ' + Il ¡ l + 4 + ! ¡ I J( b ) , ó = Ñ - ó , €

tv

, ó = Ñ - ó , * *

@ " o

( c ) , Ñ = ó t < - > , Ñ =

ool d ) ¡ Ó - N = Ó : +

I\7 1

: O : A.. \7/ñ

.. \y( e ) H - o - { = o ," l

: O : 6" \-,,

tY

: O - N = O :a \ " / ñ\./ \7

ó ,

F ! .

t. - a

N = N- / . . ,

trans

( f ) H - Ñ = N = Ñ : + - + " = f - N = N :

@ o o @2r .22 (a ) l t+ + H2NNH2 + HzNNHg* , 2H+ + HzNNHz + HgNNH¡

(b) u+ + H2NoH + H:NoH-

( " ) u+ + No2 + HNoz¡ 2HNo2 + Hzo + No + No2

(d) H+ + NH3 + NH+*

(e) 2H+ + co32 + Hzo + co2

2 I . 2 3 ( a ) 4 A s + 3 O 2 + 2 A s 2 O 3

(b) P ,+ + 3Oz + P+Oo , P ' * t 5O2 -> P+Or o

( c ) 2 P C l a t O z + 2 P O C 1 g

(d) Oz + 2NO + 2NOz

( e ) 2 S b 2 S 3 + 9 O 2 + 2 S b z O ¡ + 6 5 0 2

NHs AB3E tr igonal pyramid

NHz AB2E2 bent+

PClr+ AB¿* tetrahedron

PC15 ABs octahedron

SbCl5 ABs tr igonal bipyramid

Sb(OH)5 ( ignor ing the hydrogens)ABe octahedron

Reduction of phosphate rods with carbon usingSiOe to form a CaSiO3 slag.

Direct combination of the elements under pressureat moderate temperatures, using a sol id catalyst.

Treat phosphate rock with sulfuric acid.Alternatively make P4 from phosphate rock (part a),b u r n P 4 t o , P 4 O 1 9 ¿ d n d a d d w a t e r t o f o r m H 3 P O a .

Make NH3 by Haber p rocess , burn i t ca ta ly t i ca l l yto NO, combine with 02 and H2O (problem 27.12) toI I N O 3 .

React H2SOa v ' j - th phosphate rock , Ca3 (PO4) 2 .

Reduce As203 with carbon.

@. ^ - \ l

tla i lv o :

/ñ. . \ ? -e )+ - + H - o - N - o :" t l¡ l

o :

i ( b )

( c )

( d )

( e )

( f )

( e ) 4 H e P g O e = 6 H z O + 3 P a O ¡ q

( f ) 4 H g P O g = 6 H z O i P 4 O 5

( g ) 2 H g A s o + = 3 H z o t A s 2 o 5

2 r . 2 L ( a ) ' ó = ñ - ó , O . * - * O : ó - Ñ = ó '

%: ó = N - Ó ¡ < - )

l ^Q ,ó, t7

124 L25

úr

2L.28 (a )

(b )

( c )

( d )

2L .29 (a )

+NH+

H 4 T

As4O5

Sb2O5

Pa¡ tetrahedron, each P

other 3 P 's and has an

3HzPOz3 -

forms a single bond to

unshared Pair '

+ OH + NHs (g) + H2O

3OH + 3H2O + PH3 (g) +

+ l2OH + 6HzO + 4AsO¡

+ 2Na+ + 2OH t 5H2O + 2NaSb(oH)s

(]HAPTER 22

,IHE NONMETALS

I'ART IV: CARBON, SILICON, BORON, AND THE NOBLE GASES

r '¿rbon and S i l i con

. 1 r t forms a nelat ive ion (Ca- and Cz2-) ; has a st rongtendency toward catenation; and forms a gaseous oxide.

. 4

Catenation can occur easi ly because the C-C bond isvery strong; few bonds to C are stronger.

(a) In diamond each C is surrounded tetrahedral ly byC a toms. (c ) In S iC, S i & C a l - te rna te p laces in thediamond structure. (b) In graphite al l C's are sp2and form a continuous plane; there are only weak(London) forces betv/een planes.

Truly ionic carbides contain metal l ic cations andl ¡ - , -

e i ther C ' o r C2- an ions . Cova len t carb ides , l i keSiC, are covalent netlrork crystals, very hard andhigh melt ing. Interst i t ial carbides contain carbonatoms wi-thin a host metal latt ice without greatlychanging the latt ice or the metal l ic character;usually harden the metal.

(a ) HCN(g) hydrogen cyan ide(b) HCN(aq) hydrocyan ic ac id(c) KCN potassium cyanide(d) KOCN potassium cyanate(e) KSCN potassium thi-ocyanate( f ) Fe(CO)s i ron pentacarbony l , o r more fo rmal ly ,

pentacarbonyl iron (O)(g) NaHCOs sodium hydrogen carbonate(h) NazCOs sodium carbonate

( a ) C o ( g ) + c 1 2 ( S ) + c o c r 2 ( 9 )

( b ) C o ( s ) + S ( g ) + c o s ( g )

( c ) 2 C o ( g ) + o z ( g ) - > 2 C o z ( g )

( d ) C o ( g ) + F e o ( s ) + F e ( 1 ) . + c o z ( S )

( e ) 4 c o ( 9 ) + N i ( s ) + n i ( c o ¡ u 1 s ¡

. 2

L

I

pa i r .

( d ) A s i n P 4 O 5 I

in outward

O's around

but attach a fourth

posit ion, comPleting

e a c h P .

oxygen to each P

a tetrahedron of

1l

L26 L 2 7

l ¡

2 2 . 9

( a )

( b )

( a )2 2 . 4

( c )

( d )

( e )

( f )

(s)

" . " . ( " ) + 2HzO(1) + Ca(OH)2 (s ) + Cz[z (g)

A l+Cg ( " ) * 12HrO(1) -+ 3cH+ (g) r 4A1 (OH) s (s )

: N = N : ( b ) O , c = o , @

Q " = * ,

: ó = c = ñ , O * * O , ó - c = N ,

: ' d : c = ' S :

' : . L 2 H g B O g * H 2 O ? g . O * * H 2 B O 3

Represent the above structurally:

B ( o H ) g + H 2 o i H - ( . e ) + B ( o H ) r *

. l ;1 .13 (a ) 4Mg + B2O3 + 3MgO + MgBz, o r

3Mg + B2O3 + 3Mgo + 28 ( l imited ¡ lS)

(b) 2BBra + 3Hz '> 29 + 6HBr (g)

(c ) 2e + N2 + 23¡

(d) Mg + 28 + MgB2

( e ) B F 3 * F + B F 4

( f ) B 2 O 3 + H 2 O + . 2 H B O z ' a n d

B 2 O 3 * 3 H 2 O + 2 H s B O g ' o r 2 8 ( O H ) ¡

( 9 ) e ( o H ) 3 + o H + B ( o H ) a

( h ) B ( o H ) , h t t !

H B o 2 + H 2 o

( i ) 2 B ( o H ) s 5 B 2 o 3 + 3 H 2 o

( j ) 2L iH + B2H6 + 2L iBH+

. l ) .L4 A 3 center bond jo lns 3 a toms by means o f 2 e lec t ronsin contrast to a conventional bond which joins 2 atoms

by means o f 2 e lec t rons .

. ' . a . . L 5 B + H r o

' l ' l re Nob le Gases

r i i , ( h ) z r :ó¡ \ 7 "I

jg ^ : '1: u I - u - \ . 1 - :" l

I

. 1 - 1 . 3 resonance forms

a

l . : \ "

(a ) caco3 ( s ) + s i oz ( s ) + cas io3 ( t ) + coz (g )h a a f

( b ) c a c o 3 ( s ) " " * i c a o ( s ) + c o z ( g )

+ 2 +( c ) C a C O ¡ ( s ) + H ' + C a + H C O 3

caco3 ,^ , + 2H+ * ca2t + H2o + coz(g)t s , ,

OCN + H2O * 2e -> CN + 2OH - '970V

PbO ' l- H2O + 2e + Pb + 2OH - '580V

Yes, i f the upper equation is subtracted the net

v o l t a g e i s - ( - . 9 7 O ) - ' 5 8 0 = + ' 3 9 0 V

Pbo + cN -> Pb + ocN

(a) ' i - {F

2 2 . I T Two BH2 groups have conventional bonds' The B atom

are joined via two bonds using bridging H atoms w

are 3 center BHB bonds- The conventional bonds a

all in the same plane but the bridging bonds li-e

a plane at right angles so that the four bonds to

are roughlY tetrahedral-

/ L \ - r r .\ P / ' L : .

. / : . '\ . . 2

YÁ- / . . \

i 1 / \ : :

t lt l

( c )

AB2E3 l inear

(tr igonal bipyramidal pairs)

AB4E2 square planar

(octahedral pairs)

AB3E tr igonal pyramid

(tetrahedral pairs for I bonds)! e

l-n

B

128 L29

4r-

i

l*

' i i - ó ,

Xe

. F

ó ,t l

. . I I

: O = X et ll l

( d )

( e )

2 2 . L 7 ( a )

( b )

( c )

( d )

( e )

AB5E sguare PYramid

(octahedral Pairs)

F . .

ABu tetrahedron

(There are many other waYS to= o :

draw the Lewis structure usi

single bonds, + formal charge

on Xe, and - formal charge on

0, b\rt al l lead to same resul

fo r the shape. )

CHAPTER 23

METALS AND METALLURGY

I'he l4etal l ic Bond

Visible l ight of al l wavelengths (photons of al lenergies) are absorbed since an almost continuousband of leuels are avai lable; the re-radiat ion ofthese photons gives the metal i ts luster.

The sl ightest energy increase (thermal energy)raises electrons into empty orbitals which pervadethe whole crystal. These highly mobile electronsc o n d u c t c h a r g e ( i . e . , c u r r e n t ) . o r m o m e n t u m ( i . e . ,heat) ¡ v€r! readi ly compared to other sol ids in whichthe movement of ions is necessary for conduction ofeither electr ici ty or heat.

The extension of the molecular orbital ttrroughout themetal crystal is a consequence of the perfect period-ic l t y o f the e lec t r i c po ten t ia l - - i .e - , the per fec t l yregular arrangement of the atoms. At high temperaturethis regular arrangenent is disturbed by vibrat ions ofthe atoms about their latt j -ce points diminish"ing themobil i ty of the electrons. In an intr insic semirconductor the number of charge carr iers (electrons orholes) incr 'eases exponential ly \^r i th increasingtemperature and the conductivity increases directlywith the number of carr iers. Note that for highlydoped semiconductors the concentration of permanentcarr iers far exceeds the intr insic carr iers and thetemperature characterist ics of conductivi ty revert tometa l1 ic .

Conductors have a conduction band which is onlypart ial ly f i l led. with electrons. Insulators andsemiconductors have completely f i l led conductionbands, above which is a forbidden energy zone. Abovethis is an empty band of orbitals. In semiconductorsthe forbidden zone is narrow enough so that a fewelectrons can be promoted thermally up to the emptyband where they are conduction electrons. (See f ig.2 3 . 3 , p . 5 8 9 ) .

(r) O :-bt ó, -'bl O A86 octahedron'

\ ] l ' / '

many' many resonance formEXe

. . / Lt \oq. o: ..9: o

I

l l

x e ( g ) , + r e ( g ) + x e F 2 ( s )

x e ( g ) + 2 T z G ) + X e F q ( s )

x e ( g ) + 3 F 2 ( g ) + X e F e ( s )

X e F e ( s ) t H 2 o ( t ) + 2 H F ( 9 ) + x e o F + ( l )

X e F e ( s ) + 3 H z O ( I ) + 6 H r ( S ) + x o g ( s )

22.L8 The noble gases above Kr have too high an electro-

negativi ty (too high an ionization energy) to react

readi ly with another non-metal ' Only oxygen and

fluorine have a high enough electronegativi ty,to

extract el-ectron density from the completed szp6

^nn€ iar r r ¡ r . ion o f the nob le gases and fo rm cova len ts v r ¡ ! ¡ Y q r s E ¿

bonds.

22.Lg (a) 6¡,tn2+ + 5Xeo3 + gHzO + 5xe + 6Mnoa + 1BH+

( b ) [ M n O q ] = - L 2 O M ' 3 ' 5 9 9 o f X e O 3

London force depends on the dif fuseness

electron cloud. Both Hz and' He have two

but in He they are tightly bound to one

I

22 .20 of theelectrons

posit ive

center; while in Hz the two attract ing nuclei are

separa ted resu l t ing in an e l l ip t i ca l ' more eas i l y

po la r izab le r e lec t ron c loud '

1 3 0I 3 1

23.4 Neighboring atoms contribute atomic orbitals to

encompassing molecular orbitals' In accordance !Úl

the rule t trat there is one m'o' for each a"o" th'

mil l ions of atoms in a crystal share mil l ions of

m.o . ' s , a l l w i th in a t im i ted energy rang ie ' ca l leó"band" because the m.o . ' s must be so c lose in

within the band. Each band subtends an energy

centered roughly at the a.o. energy' A band mad€

from a higher energy a.o. wi l l cover a higher

range and may be completely separated from a

band by a forbidden zonet an energy range in

t h e r e a r e n o m . o - t s a t a l l '

Physical Propert ies, Occurrence of Metals

(b) Ba2+. sr2+, and pbz+ form very insoluble sulfates.

(c) Na' & Mg-' form very soluble salts with the anions

in sea water. A possible exception is M9CO3 but

in neutral sea water most of the COz is HCO3

ra ther than Co32- .

(d ) PbSr B i2S3, and N iS are very inso lub le .

lowetrwhich

, l ' r ,Llurgy

23 .5

¿ 3 . O

outer P electrons.

2 3 . 7 2 . 4 c m

23.8 native metal

oxide

carbonate

sulf ide

halide

sulfate

si l icate

phosphate

(a) The ions , e .g - , Au3+, have such h igh reduc t ion

potentials that any reducing materials in the

environment have reacted with them to reduce them

to the metal l ic state.

Physical ly metals are good conductors of electr ic

mechanically tough, and usually hard and high nel

invariably very high boi l ing. Chemical ly they are

usually good reducing agents (excepting the coinagt

meta ls ) .

Roasting reduces weight, may produce the metal

d i rec t l y (e .9 . , f rom i ts su l f ide) , y ie ld a mix tu re

of oxides more read.ily amenable to chemical puri-

f i ca t ion .

sulf j-des 2ZnS + 3O2 + 2ZnO + 2SO2

C U S + O z + C u + S O 2

carbonates PbCO3 +'PbO + CO2

(a) Froth f loatat ion is a physical process in which

the desired part icles in the crude ore segregate at

the surface of the air bubbles and can be skimmed off

with the froth, leaving the useless mud below the

l iquid surface.

(b) Parkes process. üiquid zinc extracts si lver form

liquid lead and floats to the top where it freezes

and can be l i f ted o f f .

(c ) Mond process . N i ( impure) + Aco(g) + t ¡ i (co) , * (S)

(d) Van Arkel process. zr( impure) + 2fzlg)

- > Z r l 4 ( 9 ) t h e n , z r I 4 ( g ) + z r ( s ) + 2 I z ( g )

( e ) K r o l t p r o c e s s . T i C l a ( 9 ) + 2 M g ( 1 ) + T i ( s )

+ 2utgcLz (L)

(f) r , iquation. An ore containing a native metal is

heated to just above the metal 's melt ing point and

the pure metal poured off '

(g) zone refÍning. An impure ingot is puri f ied by

causing a thin molten section to pass down its length,

concentrating the impurities in the molten portion-

(h) Thermite process: Cr2Oi + 2Al + AlzOg + 2Cr

App l ied a lso to o ther ox ides . € .9 . , Fe2O3 n luoOt -

Pt

M9CO3

HgS

KCl

BaSOa

ZrSiOa

LaPOa

z 3 . Y

132 1 3 3

23-L2 (a) ore i-s a sol id dug out of the ground which

contains an exploitable amount of a metal '

(b) Gangue is the unwanted part of the ore'

(c) Alün is a mixed sulfate containing a group lA

rnetaf and aluminum as cations; usually a hydrat 'o'

(d) An amalgam ís a solut ion of a metal in mer

(or . r i . " t " i= - i f you pre fer ) ; may be so l id o r

(e) Flux i-s 4n oxide (or carbonate) added in smol

to react with an imPuritY oxj-de'

(f) Flux plus impurity yields slag' which separ

as a hot l iquid and cools to a glassy sol id'

(9) Smelt ing is a high temperature chemical ref l

operation in which metals and slags are presenc

ñ i ^ ^ ^ l - - i * . -\ - t I u f > > 9 r v ¡ r ¡ 9

A 1 2 O 3 ( s ) +

Neut ra l i -ze ,

AI (OH) q +

2 A I ( o H ) g ( s )

to remove A1 from Fe:

2oH (aq) + 3H2o '> 2AI (oH) '+ (aq)

and then calcine to remove H2O:

l iqu ids .

2 3 . 1 3 F e 2 O 3 + 3 C O + 2 F e + 3 C o 2

C a O + S i O 2 + C a S i O 3

Caco3 -> cao + Coz

C + 0 2 + C o z

C O 2 + C - t 2 C O

Disso l ve A l2O3

anode : C , .f c t

cathode: 3e

+ A 1 ( o H ) 3 ( s ) + H 2 o

A 1 2 0 3 , _ , + 3 H 2 O ( g )t 5 ,

i n c ryo ly te and e lec t ro lyze, -

+ 2 0 - + C O z ( g ) + 4 e

1 !

+ A t - " ' + a l ( 1 )

l r | Produce Ca(OH)2 f rom CaCO3

CaCO3 + COz (g) + CaO

CaO + H2O + Ca(OH)2, water suspens ion

Precipitate Mg (OH) 2 from sea water2+ . -+

M g ' - + c a ( o H ) z + M i ( o H ) e + c a '

Form MgC12, d ry i t , and e lec t ro lyze

M g ( O H ) z + 2 H C L + M g C 1 z + 2 H 2 O

anode¿ 2CI + CLz + 2e

cathode, Mg2* + 2e + Mg

| | Very ac t ive meta ls (Eo very negat ive) can on ly be

made by electrolysis of the melt. fmportant examples

a r e M g , A I . L i r N a r C a r K .

Meta ls w i th pos i t i ve Eo such as Ag, Cu, Au, and '

others with only sl ightly negative Eo such as Ni and

Cr can be obtained by electrolysis of aqueous

so lu t ions .

'() (a) CuCog is treated hri th aqueous H2SO4 to form

aqueous CuSO4. (b ) CUS can be roas ted to Cu. In

both cases pure Cu is ob ta ined by e lec t ro lys is ; in/ h \ # 1 - � a i m n r r r a C U i S t h e a n O d e .\ v /

C +' I cu ( impure) + cu- ' + 2e a t anode2 +

Cu- + 2e + Cu (pure) at cathode-|.

' fhe anode voltage is insuff icient to form Ag' or3 +

Au so any Ag or Au in the Cu deposits under the

¿rnode as a sLudge.

In both the basic oxygen process and the open rtür

process the main step is the oxidation of t i : : : :

, - Tn add i t ion ' more imPur i t i c lcarbon with oxygen. ln addit ion' mt

removed in the slag. The basic oxygen process l l

fast as pure 02 is forced into the melt from abo

The open- hearth is very slow as a shal low puddJ'o

metal reacts with the hot air above i t '

r " 1 q

23.L6 (a ) UO3 + 2AL + U + AI2O3

(b) 3V2o5 + l0A1 + 6V + 5A l2o3

(c) Tazos + lONa -> 2Ta + 5Na2o

(d) ThO2 + 2Ca + Th + 2cao

( e ) W O g + 2 A 1 + W + A 1 2 O 3

In some cases i t is not a Powerful

agent ; in some cases i t d isso lves

the metal.

enough reducin or al loYs vlf

135

23 .22 4Ag(s) + 8CN + Oz(g) * 2H2O -+ 4Ag(CN)z

Aqueous cyanide in the presence of airsilver into solution, from r¿hich it canby e lec t ro lys is .

23 .23

The Representative Metals

23.24 (a) 2Na + Hz + 2NaH

(b) Na t N2 + no reaction

(c) 2Na * 02 + Na2O2

(d) 2Na + Cle + 2NaCl

( e ) 2 N a * S + N a z S

( f ) I2wa * P4 + 4Na3e

( g ) 2 N a + 2 C ' > N a 2 C 2

(h) 2Na + 2H2O + 2NaOH + H2

(i) 2l¡a + 2NH3 + 2NaNHe + Hz

23.25 Ionization potential reveals that i tremove an electron from Cs than fromthe free gaseous metal atoms. But inenvironment the ionizing tendency ofover that of Cs because of the largereleased when the very small Li+ ion\4rater molecules.

+ 4OH

brings the

(a) Pbco3 -> Pbo + CO2 roasting

P b o + c + P b + c o 1I r ed r r c t i onpbo + CO + pb + CO2 f

- - - - - - - -

(b) 2PbS + 302 + 2Pbo + 2Soz

2 P b O + P b S + 3 P b + S O 2

(also reactions involving sulfate, see ansr^rer

. ' t . 2 7 ( a ) C a * H 2 + g a ¡ ,

(b) 3Ca * N2 + Ca3N2

(c) 2Ca * 02 + lg¿g

(d) Ca + C12 -> CaC12

( e ) C a + S + C a S

(f) oca r p4 + 2Ca3p2

(g) Ca + 2C + CaC2

(h) Ca + 2H2O + Ca (OH) z * Hz

( i ) ca + 2NH3 -> ca(NH) z * Hz

I l .28 I t wou ld requ i re less inves tmentI c +

Ca ra ther than Ca- ' , bu t CaC12Iattice energy than CaCl that itfor the dif ference in ionizationfor the large latt ice energy of

chargie, and (b) smaller size of

of energy to form

has so much greatermore than compensatesenergy. T\¡ro reaSonS

CaCl2 are (a) greater

cu'n, compared to

23.26 Because of i ts verv small size the Li+ ion

resembles u92+ ldiagonal relationship) and conJ

to Na' forms a rather insoluble f luoride, cand hydroxide. Also Li reacts with N2 to form Lland i ts normal oxide is Li2O.

hypothetical Ca- ion.

2 +Be dif fers draÉtica11y from the rest of the groupin having very small size so that elements likÁ o*]rgunand halogen remain covalently bonded rather thanion iz ing .

(a ) 2AI + 3C12 + A12Ct5 (9 )

(note: A1C13 dimerizes in gas phase)

(b) 4A1 + 3Oz -+ 2ALzAs

( c ) 2 A 1 + 3 s + A t 2 S 3

(d) 2AL + Nz + 2AlN

(e) 2A1 + 2OH + 6H2O + 2AI (OH) + + 3He

( f ) 2AI + 6H+ + 2A l3+ + 3H2

¡ l= lL The sur face appears to be ' ,pass iver , , tha t i s reac tsvery slowly. The passivity may be the result of atough impervious f i lm of metal oxide.

is easier toLi when

an aqueousLi ishydrationis surroundEd

136L 5 I

23.32 Elements wil l behave

on the ion to ionic

about 1.5 t imes the

charge. Both Al and

cova len t ch for ides '

similarlY i f the rat io of

rad ius is the same' A13+

s ize o f Be2+ and l '5 t imes

Be are amPhoteric and bothth€

Transit ion Metals and Inner-transit ion Metals

(a) 2 fe + 3Cl2 + 2FeC1g, Cr same

Zn + CL2 -> ZnCLz

(b) 4Fe + 3O2 + 2FezOg ' Cr same

a l s o 3 F e + 2 o 2 + F e 3 O 4

2 Z n + O z + 2 Z n O

(c) Fe + S + FeS, Cr , Zn same

(d) 2Cr * N2 + 2CrN' others no reaction

(e) reactions with very hot steam only

3Fe + 4H2O -> Fe3O4 + 4H2

2 C r + 3 H 2 O + C r 2 O 3 + 3 H 2

Zn * H2O -+ ZnO + H2+ 2 +

( f ) F e + 2 H ' + F e * H z , o t h e r s s a m e

(g) Fe + OH + no reaction

2Cr + 6oH + 6H2o -> 2cr (oH) . t - * 3H,

Zn + zOH + 2H2O + Zn(OH) +2- + Hz

The transit ion metals are much more usefuL structur-a1ly, but are less metal l ic chemical ly speaking.They are usually colored in their salts, capable ofmany oxidation states, and frequently found combinedwith oxygen in the negative ion.

Zr and Hf are very close in size (which is the rule

for periods 5 & 6 beyond Hf) and considerably larger

than Ti. (Genera}ly period 5 elements are much

larger than per iod 4 . )

C r O + 2 H ' - > C r - ' + H 2 O

CrO + OH + no reaction

c r 2 o 3 + 6 H + + 2 c r 3 + + 3 H 2 o

Cr2Os + 2OH + 3H2O -> 2Cr (OH) +

CrO3 * H2O + H2CrO4 or 2CrO3 + H2O -> H2Ct2O7

CrO3 * 2OH -> CrO'+2 i- H2O

23.33 Both AI3+ and s2-

ne i ther i s Present

2 3 . 3 4 ( a )

( b )

( c )

( d )

( e )

\ r ,

/ ^ \¿ J . 5 4 \ 4 , '

(b )

( c )

( d )

( e )

( f )

(s )( h )

1 + - - - ^ - - 2 t - - +A l " ' + H 2 o Z A l o H + H

s ' + H 2 o ? H S + o H2 -

In basic solut i-on where S

alwninrun is converted to

where Al3+ can exist the

ver ! weak H2S'

S n + 2 C L 2 + S n C l 4

S n + 0 2 + S n O z

Sn + 25 -> SnS2' ^ J

s n + 2 H - - > s n " ' +

Sn + OH * 2H2O ->

or Sn + 2OH ->

3sn + Arf + 4Nos

the ampho

In acid solu

t ied uP as tha

hydrolYze so extensivelY that

Lo .tY extent in neutral solu

can ex is t

Al (oH) q

su l fu r i s

H2

Sn (oH) g2 -

SnO2 +

+ 3SnOz

2H2O + 2OH (aq)

+ p b s O 4 + O ¡ ( S )

PbO + H2O ' r OH - ' Pb(OH)e

-> 2PbO + Oz (g)i

+

) -SnS + S ' - + no reac t ion

2 - ^ ^ 2 -S n S z + S + s n 5 3

P b C l z + C I + P b C I g_ 2 -

S n F ¿ + + 2 F + S n F e

* H z

Hz

+ 4NO + ZHzO

2 -+ Pb (OH) oPbOz (s) +

3PbOz (s)

2PbOz (s)

l

l .

l iL

i

l

!. lr

l 3 B 1 3 9

r-

i

(b) 4r,a

(c) 2La

(d) 2r,a

(e) 2r,a

( f ) 2 fa + 6H- ->

23.40

metall ic trait of easy

23 .4L (a ) +0 .0592 V (b ) +0 '770 V

"1.0 M" would give a result that would be

(c) rhere would be no r¡tay to know that +0'788 V

the correct Eo measurement' The use of rrHgtil ¡

t" t ' t" ,R 24

COMPOUNDS

!t, ' t ure of Complexes

I (a) Coordination number is the nrn"nber of atoms in thefirst coordination sphere, i .e., the. number of atomsconnected to the central metal atom.

(b) A l igand is an atom, ion, or molecule attached tothe central metal atom via an unshared electron pair.

(c) A chelate is a coordination complex formed by abidentate or polydentate l igand, i .e., a l igand. withtv¡o or more coordinated atoms in the same molecule orr o n .

(d) Enantiomorphs are two isomers which are alikecxcept that one is the mirror image of the other.

(e) An inert complex has a very slow rate of dis-sociat ion or l igand exchange.

(f) The low spin state of a complex has as many ofi ts electrons paired as possible.

( a ) c o 3 + , ( b ) A u 3 + , ( c ) v o , ( d ) c o 3 + ,

(c ) Co 2+

' l ' ransit ion metals form complexes most readi ly, alsoohhers whose ions are smal_l and highly charged.. Ther¡rost common arrangements of ligands around the centralrrrctá.I are octahedral , tetrahedral , l inear (2 co-,¡rdinate), and square planar.

I . J I I3 , a base, uses i t s unshared pa i r to bond to Ag+,

,ru acid, via an empty orbital on the Ag+. Alter-

rr, t t . ively one can consider the si lver ion in aqueous

, ; , r lu t ion to be ag(u2O)2+. Then the reac t ion is arrtrcleophil ic substi tut ion or base displacementr ( ) . rc t ion , in wh ich the base NH3 d isp laces the basei l , O .

The noble metals have

potentials' associated

they lack the tYPical

an electron.

unusually high ionizatfonwith their small size.

as the correct Eo. For concentrat ion cel lg' l ¡ ' f

Eo is not involved- A calculat i-on for the cel l

described in part (a) and based on a "1'0 Mrl

concentration of rrHg+rr with I electron exc

v,rould give +0-0592 V as the result, and the ao

measurement would be +0.0296 v'

23.42 (a) 2La + 3Clz + 2laCls

-t- 3O2 + 2lazOg

+ 3 5 + L a 2 S 3

+ Nz + 2LaN

+ 6H2O + 2I¿ (OH) 3 + 3H2

2 L a - ' + 3 H 2

l lI

I

23 .43

the chemical propert ies dif fer also'

The inner-transit ion elements dif fer from

with respect to the number of f elect¡e¡5' büf

electrons are not generally involved in chenlo

bonding so there is not much difference betut'A

elements.

r , - = 1 . 2 6 x 1 O - 3 4 2 4 . 6 ' K = I . 3 5 x 1 0 - 3 2

141

E.

140

2 3 . 4 2

23 .43

23.40 The noble metals have unusually high ionization

potentials' associated with their small size. Thuli

they lack the typical metallic trait of easy loss

an electron.

2 3 . 4 L ( a ) + 0 . 0 5 9 2 V ( b ) + 0 - 7 7 0 v

(c) there would be no l^tay to know that +0.788 V iE

the correct Eo measurement. The use of t 'Hg*" "¡"1.0 M" would give a result that would be regarded

as the correct Eo. For concentrat ion cel lst

E" is not involved. A calculat ion for the cel l

described in part (a) and t¡ased on a " l .O M"

concentration of "Hg*" with I electron exchanged

would give +0.0592 V as the result, and the actual

measurement would be +O.0296 V.

| i lA r " tER 24

COMPOTJNDS

|, l- . j ture of Complexes

. | (a) Coordination number is the nr¡nber of atoms in thefirst coordination sphere, i .e.. the. number of atomsconnected to the central metal atom.

(b) A l igand is an atom, ion, or molecule attached tothe central metal aton via an unshared electron pair.

(c) A chelate is a coordination complex formed by abidentate or polydentate l igand, i .e., a l igand witht\n/o or more coordinated atoms in the same molecule ori o n -

(d) Enantiomorphs are t\^7o isomers which are alikeexcept that one is the mirror image of the other.

(e) An inert complex has a very slo\¡/ rate of dj-s-sociat ion or l igand exchange.

(f) The low spin -state of a complex has as many ofi t s e lec t rons pa i red as poss ib le .

r ( a ) c o 3 + , ( b ) A u 3 + . ( c ) v 0 , ( d ) c o 3 + .

(e ) Co 2+

t Transit ion metals form complexes most readi ly, alsoothers whose ions are sma1l and highly charged. Themost coÍunon arrangements of ligands around the centralmetal are octahedral, tetrahedral, l inear (2 co-ordj-nate), and square planar.

I NH3, a base, uses i ts unshared pair to bond to Ag+,

an acid, via an empty orbital on the Ag+. Alter-

natively one can consider the si lver ion in aqueous

so lu t ion to be Ag(H2O)2+. Then the reac t ion is anucleophilic substitution or base displacementreaction, in which the base NH3 displaces the baseH 2 O .

2 4 . 6 ' K = 1 . 3 5 x 1 0 - 3 2

L4t

(a ) 2La + 3Cl2 + 2LaC1s

(b) 4La + 3o2 + 2Lazos

(c ) 2 I¿ + 35 + La2S3

(d) 2La + N2 + 2¡¿¡

(e) 2r,a + 6H2O - ' 2Ia (OH) 3 + 3H2

( f ) 2La + 6H+ + 2La3+ + 3H2

The transition metals differ from each other with

respect to the number of d electrons' and since d

electrons are very much involved in chemical

the chemical propert ies dif fer also.

The inner-transition elements differ from each

with respect to the number of f electrons' but f

electrons are not generally involved in chemical

bonding so there is not much clifference between

elements.

L40

K = I . 2 6 x l O- 3 +

7-

2 4 . 7 ( a )

( c )

( e )

(s )

2 4 . a ( a )

( c )

( d )

t f \\ ¡ ,

( h )

2 4 . 9 ( a )( b )( c )( d )( e )/ 5 \\ ! /

2 4 . L o ( a )(b )( c )( d )( e )( f )

Kz [Rh (Hzo) cl s ]

Na3. [Reo2 (CN) + ]

K¡+ [Ni (CN) +]

l C u ( N H g ) r + l s [ C r C 1 e ] e

V (co) e

lcoCl (Noz) ( t lHs) + l z So+

Na3 [Ag (SzOs ) e ]

I P t C l + ( N H s ) z ]

(b ) l co (NHg) + ( so ¡+ ) ]Nog

(d) lco (wHg ) z (en) e] Clz

( f ) K+ [Ni (cN) 6 ]

(b) zn [Ptc l6 ]

( e ) [ P t ( N H 3 ) r + ] t P t c l g ( N H r )

(S) Ks [AuBr6 , ]

lN i (NH3) o l s [Co (Noz) e ] z

potassiun tetracyanonickelate (O)

potassium tetracyanonickelate ( I I )ammonium pentachloroaquof errate ( I I I)

tetraamminecopper (I I) tetrachloroplat inate (

nitr i topentaammineir idium (II I) chloride

hexaamminecobalt ( I Ir) tetracyanonickelate (I tr ' l

ni tro syltr icarbonylcobalt (o )diammine te trachlorop latinum ( IV )potassium tetracyanoplat inate (O)

hexaamminecobalt ( t I I) hexanitrocobaltate (I I)

sodium dicyanoaurate (I)

Bi s (e thylene di amine ) chloro thi oc yanatocobaLt

chloride

(a) [P t (NHs ) , * (oH) (so4 ) ] oH

(b) tPd (d ipy) (NCS) 2 l

( c ) [ C o ( N H g ) + C l z ] c l . H 2 O

( d ) [ c r ( N H 3 ) e ] l c r ( C z o + ) z ]

Note in compound (d) that i f Cr is 3+ in the cationand 1+ in the anion, then the original ions are +1and -1 and the ions in the isomer are +3 and -3.

F i rs t cons ider a1 l poss ib le coord ina t ion isomers ,then examine each for stereoisomerism.

C o o r d i n a t i o n i s o m e r s a r e ( a = N H 3 ) : [ P t a 4 ] [ P t C l o ] r

I P t a 3 C 1 ] [ P t a C l s ] , [ P t a 3 C 1 3 ] [ P t a C I 3 ] , a n d

[ P t a a C 1 2 ] [ P t C l 4 ] . I n a l f o f t h e s e t h e s i x -

coord ina te P t i s P t ( IV) and is oc tahedra l ; the four -coord i -na te P t i s P t ( I I ) and is square p lanar .

The f irst two above can have no stereoisomers (norany op t ica l i somers) .

F o r [ P t a 3 C 1 3 ] l P t a c l 3 ] t h e r e i s o n l y o n e p o s s i b l ean ion , bu t two ca t ions :

c1 c1

Isomerism of Complexes

24. IL (a ) KFe [Fe (cN) e ]

(c ) cuz [Fe (cN) o ]

2 4 . L 2

a A 1 1

(b ) Fe [Fe (cN) s ]

(d ) KzFe I re (cN) s ]

F o r P t a 4 c l 2anion, but twoother t rans .

None of the

a

PtCl4 there is on ly one poss ib leca t ions , one w i t .h c is C- l ' s and the

last four have optical isomers.

(a ) A is ICo (NHs ) s ( I i zo) C lBr ] Br . H2o

B i s [Co (NHs ) s (Hzo) zc l ] Brz

(b) hydrate isomers

( a ) l c o ( N H 3 ) s ( S o q ) ] N o a

(b) [rvrn (co) s (NCS) ]

( c ) [ P t ( N H 3 ) g c l ] [ P t ( N H 3 ) c l 3 ]

( d ) [ c o ( e n ) z ( H z o ) B r ] B r 2 ' H 2 o

In (c) both Pt are 2+ in both isomers.

(a) There are tt tro geometric isomers, cis or trans aiwhere a = NHs, t = NCS-

cc i s

L42 I43

(b) Set NH3 =

geometricedge (two

a

a andisomers ,of the a

NOz = n. There are two

fac ia l (a I I a a re c is ) o r

are t rans)

n

" nr¿eLar

( c ) S e t N H 3 = a .

i s an op t i ca l

a

c1(d) Only one

a

n edgr

I f bothisomer

a ' s a n d C l r s a r e c i s t

mirror

have e i ther a ts o r C l ' s t rans .

is an optical i

mlrror

The other is trans BrCl

( f )

c1I f NH3 are t rans ,

( e ) If Cl and Br are cis there

i . e . r a d ' 1 P a l r .

C a l I C 2 o r * 2 - = o x t N H 3 = ¿ .only one isomer

a

a

I f NH3 are c is , there is a d, I pai r

nLrror

L44' I

¿ q

r( g ) T h e r e i s a d , 1 p a i r

24 .L7 No op t ica l i somers are poss ib le fo r square p1and neither for tetrahedral i f any two l igandsr L ^ - - - ^L J I E S ¿ i I t I E . L E E N H c = A .

(a ) The th ree poss ib i l i t i es a re t ransa - C l , o r t rans a - Br . (b ) the twoa r e t r a n s C l ' s o r c i s C l ' s . ( c ) S a m e(d) On ly one isomer . See drawinqs on

2 4 . 1 8 The dipole moment of the trans isomer is zerothe Pt - C l bond d ipo les are a t 180" , and l i kewlthe Pt - N bond dipoles. In the cis isomer thePt - N bond dipoles are at 90o and have a resulopposite in direct ion but NOT EeUAL to the resulo f the Pt - C l bond d ipo les , so there is a ne tmolecular dipole momenE.

Noe

r,l_+ng_jAjsmp¿sxee

t -. 'O [N i (C l ¡ ) q ] - has no unpa i red e lec t rons .

lN ic l4 l ' - must have 2 unpa i red .

. l I [Fe (CN) e] -

is low spin, as deduced from the posi-t ion

of CN in the spectrochemical series, whereas

[Fe F6] -

i s h igh sp in . The conf igura t ions fo r

¡ e ( I l - l ) , á d - r o n a r e :3 - ? -

[ F e ( c N ) s ] [ F e F 5 ] -

III

II

miffor

" v J I

possib i l las (b ) .p . 7 5 6 o f

there canthere is I

+ t

+ + t

2 4 . 1 9 I f e i t h e r t h e N O ' s o r C l ' s a r e t r a n sno optical isomer, but for the al1-cis1 p a i r .

+ + + + +

(a) The Cn complex wil lexceeds P, wh i le the H2O

+ -( b ) [ F e ( c n ) e ]

++ ++ t+

be low spi-n because Acomplex is h igh sp in .

l F e ( H 2 O ) G l

++

t t

t t

mirror

I46

(a) The pa i r ing energy must 1 ie be tween 250 and 460

kJ mol -

and a value of 335 has been reported.

(b) Note in the spectrochemical series p 643 that2 -

CzOg induces even less o f a sp l i t t ing than H2O,

s o i f [ M n ( H z o ) o l 3 +

1 " h i g h s p i n t h e n [ M n ( c e o + ) s ] 3 -

must also be high spin.

I47

2 4 . 2 4 t c o ( N H 3 ) 6 1 ' '

+ t

d 7 ,

+lrzott

P=270kJ

Ao .P

2 ¿

¡ c o ( N H r ) 6 1 " '

+ + t + + +

d " , a o t P

(b) ct2*, d4

Low spin High spin

+

(a ) c r3+ d3 , same fo rstrong or weak f ieldl igands

(b) N i2+ d8 , same fo rstrong or weak

(c ) l ¡ i2 * d8 , squareplanar only occurs withstrong f ield l igands

+ + +

t +

l+ ++ r+,1u2 - ' r2

T

t * dxy

I I d .z2

+tl t+ dxz, dyz

as abovef

t+ +v t+

r + + + +

1l,,ou,I

P=2 lOkJ

+ + + + +

( a ) Z n - ' ,¿ 4 . ¿ )

Low spin

+ + + +

+ + +/ ,L

( i )

Low spin

++ ++ ++t+

( c ) N i - '

Low spin

+ +

+ + + + + +

, 1 0Cl

High spin

t + + +

++ +* r+

, i8

High spin

(e) I"k 2+ , d5

Low spin High sPin

( d ) c o 2 + d 7 ,onJ-y strongligands

a s i n ( c )f i e l d

+ + + +

(d ) N i " ' ,

Low spin

+ _

t+ r+ ++

( f ) F e " '

Low spin

+ + + + +

(h ) ,

Iow spin

High spin

High spin

sprn

u + a( r ) K n d r m u s tspin (high f j .eld)dj-amagnetic

r¡+( S ) I r ' d ' , m u s tsp in (h igh f ie ld )ONLY one unpaired

+ + L _

dL1

+ +

t + 1 + +

,5, o

+ t

+ + t + + +

t^

+ +

,¿o

High

+ t

+ + + + f + + +. ,L ?

(e) , d"

Low spin High sPin

A+ +

, o

High

(e ) co2+ d7 , a l l te t rahedraLcomplexes in 4th peÉiod areweak f ield

t+

t+ ++

+ + +

t+ ++

lowbei f

be lowi f

spin

+ + _ + + _<T

( i ) , d "

Low spin High sPin

t i3+ is d r . on ly one e lec t ron , and the hv o f absorbedlight is the measure of Ao. The red-violet transmitted

color is due to absorption of l i_ght in the mÍdd.le ofthe spec t rum, i .e . , g reen. A s t ronger f ie ld l igandcauses absorption more toward the vj-oLet end Leavinqthe red transmitted.

t +I49

' , .s (a) t?!r ' t 2!7-at *ó t ó f

t"l 1!r * rlo * l"l',.6 (a) t3lr" * 22"lna" *

(c) ll"" * i3* . i"

t ¡ ' l 66 r - , , * 66 rn * O, t ,

2g t t -

3 }o t , . _14

(d) 9" * r ] ]* * r ] lc"- r f , o 1 3

, , . r 1 4 - , 1 1 4 ^ . 0(D' 4-7^g

- 48uq

* _le

(d) le + l lFe + l lMn- r z o 2 3

42"-

4^HezRadioactivi ty

2 5 . L Vüithin the nucleus there must be very strongforces, operating within a very short range

(\' 2 x 1O-r 3cm)

. otherwise it would not be Posfor so many posit ively charged part icles, the

to coexist in such close proximity. rt is Postuthat the force originates from the exchange of

betr^reen nucleons. One can make a rough analogy

hydrogen molecule being held together by the

of electrons between the two nuclei.

25.2 (a) l¡ucl ide means a part icular combination of

and protonsr practical ly synonl¡mous with " isotop6¡

except nucl ide is appl ied to nuclei and isotope tO'atoms. (b) A nucleon is a nuclear part icle of

about lu , i .e . , a p ro ton or neut ron . (c ) The c r lmass is the least quanti ty of f issionable matter

wherein the number of neutrons captured exceeds

number of f ission reactions, so that a chain

can occur. (d) A thermal neutron has kinetic

of the magnitude RT per mole (for T = 29BK). l tg

speed is comparable to that of the average

molecule at room temperature. (e) Fission is a

nuclear reaction wherein a large nucleus spl i ts2 much smaller ones (ranging from 4O-60% of the

in weight), along with a few free neutrons (t

3 per f ission) - ( f) Fusion is the joining of tv¡o

small nucl ides into a larger one. (g) A Tra

element has an atomic number larger than 92.

) ' , . J 2 . 6 M e V 2 5 . 8 9 . 4 0 M e V

; l ' , .9 Energy must be 6 .34 - 6 .12 = 0 .22 MeV. Some decays ,those g iv ing a lphas o f on ly 6 .12 MeV, resu l t in an

- 2 L 7excited

-éSOa nucleus. This decays very short ly

thereafter to the ground state tll"t

releasing the

.22 MeV of excitat ion as a y ray.

l ' r . 1 0 6 . 5 6 - 6 . 2 3 = 0 . 3 3 M e V y r a y s . T h e e x p l a n a t i o n i sparal lel to problem 25.9 but involves the excited

' 2restate or -Jlnn.

. , ' ' . 1 1 1 9 . 9 9 9 9 9 u

. " , . 13 3 .52 MeV

= " , . 1 5 3 0 . 9 8 0 6 6 u

! ' , .L7 Electron capture must

2 5 . L 2 1 1 . 0 2 1 6 6 u

25 . I4 3 .42 MeV

2 5 . L 6 2 6 . 9 8 6 7 2 u

be the process.

2 5 . 3 t"t 2ffat*2rrar"i * |""

t"r llar*fl*s * f"t"r rflet * rl7ro, * |"u

o l2nc." * llNi * l.

tur 1l!a"*1!!"s * l"

{c) -fe * tl?"n * r!!na

tur llua * i;Ms *-1"

(d) -ie . i3* * llct

t¡ 'r t-c of Rlrdioactive Decay

" ' . 2 L . 0 0 3 6 5 9

. " . 23 2BB days

- " , .18 Must be e lec t ron capture ,

, ! r ' . I 9 5 0 . 9 4 4 8 u

:" -2O The nucleus captures the closest electron, namely als electron. Vühen a higher energy electron (from oneof the outer orbitals) fal ls into the empty ls orbitali ts loss in energy i-s emitted as an x-ray.

2 5 . 2 2 ' . o O 7 7 4 g

2 5 . 2 4 6 . 4 0 h r . I

151

. E A

150

-al

25 .26

2 5 . 2 7

2 5 . 2 9

2 5 . 3 l

- 1( a ) . 0 0 6 2 4 m i n

- l( a ) . 2 6 6 6 y x

2 5 5 m i n

1 . 5 5 x l o - 6 4 s

( a ) I . 5 5 7 x 1 0 r 7

( b ) 1 . 5 x 1 0 - g

(b) 111 ¡n in

( b ) 2 . 6 0 Y r .

25 .28 2900 Y rs . o l d

2 5 . 3 0 0 . a 2 7 c í

atoms

Nuclear Reactions

Di si 4tegrati,ojl !_eri e s

I 5 4 4 1 5 0 _25.32 *árur -

; ru n -¿¿ot' I (n-llov

o o

I q n

. , b oo+

0- > é +t -

A

r J v m L

b )

l-46 _- ^5mo z

) 2 4^^}(a

) 1 ?- l lPoó q

25.3s t.l !!4. + fn * !1"' n v

tnt ]!n + fn * |"" * 1r"i

t.r lfcr + fn * i" . if'(d) ]"i * in * l" * 7n""

t"r l l !t" + lH + zf" * rf lr

t t r 13c" * | r" ' ' l " * j !s.

(s) '3ltn * aru" * lr, + '33*

(h) '37u * l3*. * a|" + i33*ls.3o t"l 13e n l^" * N" * 1l*

{rl !r,i + ;n * 1". * l"(c) ]"i * ln * ¡" * luu

tal tfc n lrtr * ti* * t

<"t enlv" * l" * |" * l!t.trr 13"' * l* * lH * l:A=

A q A . 1 A p .{s) j ]sc + ;He

'+ lH + ; ; r í

c-'r !]v + fu * zf" * llc.,',.37 t"t 'lZu, (b) '3t^, o 2anl"cr,

(e) "n}u, trl 233u=, rur2f,lv,

l q n* IJ t ¡ +o f ,

l 46e_ +6 ¿

, q ? ? 2 3 2 Í h

9 0 - "

220-^-}(nó o

208- .P h

t5z

25.34 - i imp

9 3 -

) r q

89^"

209^.ó ¿

0 1 5 0 ^ .- e + - , u o-_L O¿l

4 u ^ * 1 4 2 n ^2"" 60."-

228-^ 228^^BB^o 89""

2\6Dn 2L6^tó 4 t J J

¿ J 5 _ Z J J - -

g r P a g 2 u

. . 1 1 1 1z ¿ L _

t s r A f

6 t . r )

209^.B ]

U J

22Brh9 0 ^ "

2 L 2 ^ .R 1

t J J

22911^9 0 - "

2 L 3 _ .83IJA

^^tréJ W

2L3_¿52

)1'7( d ) - ; 3 N p ,

? q n(h ) - : : c f

J Ó

L 5 ¿ 1 5 3

7

? 4 q 1 62 5 . 3 A

- : : c f +

t : N - >v u t

"X" is slmbol

. 133*,element #105

, 1*o*

for

26

CHEMISTRY

cHs

CH

CH¡II

cH3 -

?"toz*íA[

wJ-

@1 5

Nuc]eaI Fission and Fusion

2 5 . 3 9 7 . 5 9 M e V

2 5 . 4 O ( a ) 1 8 0 . 0 M e V r e l e a s e d

(b) . oB18e"

2 5 . 4 I ( a ) 5 . 5 M e V r e l e a s e d

( b ) . 1 e s %

2 ? q I I l ¿ R2 s . 4 2 ( a ) - I i u + l n + 3 i n + * l l c e

Y ¿ U U 5 A

/ < a | | t h(b) - I lu + ln + 3ln + l ln¡ +9 ¿ U U 3 /

(c) ' l lu * ln * g1r, * 134*"Y ¿ 0 "

- O " 5 2 - "

( a ) H

F - ^

, v n z - v B 2 - v n 3

H

C 2 H 5 C H 3

t l- c H - c H - c H 3

u n 3

IC H e - C = C H z

CH - CH3

I

C - C H 2 - C H 2 - C H 3

ur-

ICH - CH3

8 5+ q a

5 1

r3B^^f f

o q+ 11zr

4 U

c1II

II

c1

( b )

( c )

( d )

( e )

( f )

c H g -

U B 2 -

a H -

^ r a = ^u n - u

(s )

Noz

Noz

L54

( h )

---Tl

25.25 (a) .00624 * i r , - r (b) 111- min

25 .26 (a ) . 2666 yx - r ( b ) 2 .60 y r .

25 .27 255 m in 25 ,2A 2900 y rs . o l d

2 5 . 2 9 r . 5 5 x t o - 6 4 u 2 5 . 3 0 0 . 8 2 7 c i

2 5 . 3 1 ( a ) 1 . 5 5 7 x l 0 r 7 a t o m s

( b ) I . 5 x l O s 9

25.3s t"l 3i". * f ' . * ! lu. * y

(b) t3" n 1r, * 1"" * ]"i

t"r ]fcr * ],' * i" . if'(d) ]"i * in * á" * f,""(e) ti!t" * fH * zf" * il! '

rct 23oc" * !^. * |" * l!'.(s) '31*n * |ru * 1r, + '33*

(h) "nZu * i3*. * a|,' * í3!*15.36 (a) t3" * |n"

* | " * 1 l *

ol !r.i * f,, * 1"" * l"(c) ]"i * i" * ¿" * !""(d) '2" , lH * l ]x * y

(e) f,lr" * ln * á" * ?3'"(r) ll"" * lH * ln * l3o"tsr lls" n |nu * !" * llrt(h) ;1r . fH * zf" * Zi"'

,-37 (a) ""2u, (b) 'tt^, o 2f,lcr,

(e) '32u, rrI 2i3"=, (s) ."r2",

Nuclear Reactions

Di si ntegrati g4 !_e_-Li e s

r q ¿ ¿ l 5 n¿ 5 . 3 ¿ - ^ E r + ^ H e + - - D Y

oal z oo

' I 4 ^ n l q n' l ] r ¡ + l . + - l l c a

b f - r o +

1 ¿ " ^ o , * l 4 2 n ^-áit* - ir. 60-.-

) 1 ) 2 2 8 ^2 s . 3 3 - : : r h - I l n a

YV 'JÓ

z ¿ u _ z l o ^^ - K n ^ , P Oticr t +

208^,ó z

) 2 1 2 3 3 p ^25.34 -éárn er^*

2 2 5 n ^ 2 2 L r _g g ^ " B i ' -

29?no '9?"tt 3 ¿ U J

2287.^ 22grhB9^ ' " 90" . .

2 1 6 _ . 2 L 2 _ .^ _ A E ^ - l j a

233r r 229ah92" 90^ "

2 L 7 - . 2 L 3 _ .^ _ A E ^ . I J r( t ) óJ

tlSoo * f. *

t q o 4-;;"u -> )He

+

1 R ñ*" "T1^

146 ^o ¿

z z l _

óó

2L2D^at+

) ) q

9 9

ó¿

( d ) - ; ; N p ,

) \(r( h ) - : : c f

Y Ó

L52I f J

, ( ?a 249. ,F * r5n .1 260- -¿ 5 . 3 ó g 8 " t .

7 r u o n o t *

I O 5 ^ ,

"x'r is slzmbol for element #I05

Nuclear Fi-ssion and Fusion

2 5 . 3 9 7 . 5 9 M e V

2 5 . 4 0 ( a ) t B O . 0 M e V r e l e a s e d

( b ) . 0 8 1 8 ?

2 5 . 4 L ( a ) 5 . 5 M e V r e l e a s e d

(b) .L95"6

2s.42 t"l 23r!rv * fr' * :f" * t33""

u 2llru * 10" * tl" ' 3?*

o 2llru * f'' * s|" * '12'"

CHAPTER 26

ORGANIC CHEMISTRY

( a )2 6 . 7

( b )

( c )

H

ñ - ^

cHs -

c H gIICH

II

Ln2 -

CH¡

¡ u = f

v r ¡ 3

cHg -

. / " n r - C H 2 - C H 3

H

C z H s C H g

t l- C H - C H - C H 3

u n 3

IC H z - C = C H e

C H - C H 3

IC - C H 2 - C H 2 - C H 3

IC H - C H 3

J +

1 ? a

f f

o o+ - - v -

( d )

clIII

u a

( e )

( f )

(s )

Í " ,

o,n-ffrf xo,tV!o,oz

L54

( h )

rMlt ( ) t ( ) l\ / V

1 CI J

2 6 . 2 S e e s t r u c t u r e s p . 7 5 7 .

26.3 Draw the structures inpl ied and then name themcor rec t ly .

(a) CHs - CHz - CH : CH - CHs, by counting from theright a smaller index nunber is requi-red.

- CH3r the longes t cha in i s four

2-mel -hv l h r r f .ane

C - CH3 imposs ib le , 5 bonds oncarbon from left .

- C = CH, by counting from the r ight¡the index numbers are smal

d i (4 \ c l

t r i (5) c l

Ic l - c _ c _ c

II

c1

C1l

c _ c _ c _ c l _II

C 1

l { , . 6 C H 2 = C H - C H 2 - C H s

^ ' n

^ - ñ

cHs

H

^ - ñ

cHs

cHs

/ (Same scheme 2 6 . 5 ¡

butanes (6 st ructures) C1

c1I

I¡C I

c - c - c - c lI

cl-

C1

IC ] - C - C - C C l

?H, -

?'t't l

CH2 - CH2

cHs

IC H 2 = C - C H 3

cHe

C H z - C H - C H 3

chiral-

c1I- c -

carbons* )

- C

I¡cl-

- L

2-pentene

(b) cH2 - cH

t lC H s C H s

( c ) C H 3 - C =

IcHs

( d ) C H 3 - C H

IcHs

1C

IC1II

C]

I- c

cl ct c ll l lt t lr t t

2 - m a { - l r r ¡ l - l - l - ' r r 1J ¡ ¡ ' v e ¡ ¡ f * - - * E . y n e

( e ) C H sI¡

CHs - CH = C - CHg, imposs ib le , 5 bonds on 3 fdcarbon from left .

( f ) CHs - CH - CH2 - CHz - CHs, longes t cha in ia

ón,

IczH s

26.4 (a) a lkenet Crrnr .

s ix carbons .3-methylhexane

(b) alkvnes C H' n 2 n - 2

(c ) a lkad ienes "nnrn_,

(d) cycloalkanes Crr"r '

26.5 Formulas below show only the carbon skeleton and tchlorine; hydrogen was omitted for clari ty.

c lI

m o n o ( 2 ) C l - C - C - C C - C - C

H

a s i n

CII

C?t -

156 r57

c1II

n t - a - r - - r - ¡ t - C

r.1

I

II

cl-

propanes (3

(1

c l - c - c - c - c l

a l a

I - it l

s t r u c t u r e s ) C l - C - C - C

c l - c - c - c - c - c l

c l c lt tt ¡

c - c * - c * - c

II

c1

Br

r^vt'

B r B r - _ B r

r

T

Br2 6 . 9 ( a )

Br../,-*\t ( \ lI t ¡ tv

Br

OH

Noz Q:,

2 6 . 8 ( a )

OH

l'-r\"\?

( b )

OH

,Au'l ( ) l\:l't u'

OHI

rAr srt ( ) tV-

Br

OH

(b ) th ree

Br Br Br

[ñ*o, ñi ñr$u' werpn' Vu

*oz

(c ) one on ly

Noz

OH

r

c1

OH

e' frn'\l

two

A\lu'

rAr"lV,

OH

o,.,.c),.Br

l sB

c1/ \ / \

t ( ) t ( ) tCI

2 6 . L L

Z O . L ¿

z 6 . r 5

^ral.'OO\A\l

crr'

x - C = C - X these four atoms are l inea:r

Each of the carbons which form the double bond must

connect to t \nro unl ike groups.

( a ) C H g Q H ¡\ . /

r - = c N O,/

-\

C H s C H s

( b ) C H s C z H s\ . /'C = C YEs (trans shown), / \H C H 3

( c ) C H s C z H s\ . / \TA

u - !

, / \CHs H

(d) Hoooc ,cooH\ , /\ 'C = C yeS (c is shown)

(c) Cracking is a process in which a long molecule isthermally degraded into shorter molecules without anyother substance involved in the reaction.

(d) A conjugated system has alternating single anddouble bonds.

(e) In an addit ion reaction two molecules are joined

without any other reactant or product involved,usually via one or more double bonds.

(f) In a substi tut ion reaction some atom or groupreplaces the hydrogen attached to a carbon atom.

¿ 6 . I 4 ( a )

r 6 . 1 5 ( a )

,/H

( a ) A n o l e f i n i s a

bond.

(b ) An homologous ser ies i s

the same function dif fering

the nrmber of CHz grouPs in

the molecufe .

H

hydrocarbon contain ing a double

further CHs - CH = CH - CHz -

C H s - C H z - C H 2 - C H 2

v a ¡ 5 v r t z v L L ¿ .

I

CHg -CHz

H z + C H a - C HIIt l

v r r J v . r ¿

CH3 t H2 ->

- cHs

( b )

cHs

H

cHs -

,CH2-CH3./

C = C * H z

\H

C = C - C H z - C H 3 *

a series of compounds wl

from one another onlY ln

( c ) CHr -CHz .

H B r + \ c =

CH3-CH2

cHz

BtrI

v L t z v e r ¡ J

CH 3 -CH2

1 6 0

the hydrocarbon portron

1 6 1

+ l -T f . -

(d ) HBr + CH3 - C = C H + C H s - cHz

l -T{^ -} l -T{^ - a-Rr^ - a-T-I^v a ] J v u L z v ] ¡ J

(a) Br2 + CHa -> HBr + CH3Br

A series of substi tut ions l ike the above takes plaoOlead ing to CH2Br2¡ CHBr3, and CBr4 .

(b) CHs - CH = CH - CH3 * Br2 + CHg - CHBr - CHBrI

ci,

( c ) C H ¡ - C = C - C H 3 + 2 P . 1 2 + C H 3 - C B r 2 - C B r 2

cHs

( d )

.16.19 Substi tut ion on an alkane is dif f icult but can befaci l i tated by a free radical mechanism. An aromaticr ing is most readi ly attacked by a Lewis acid, usuallya posit ive ion. After i t joins the r ing by using apair of "double bond,' electrons, the aromatici ty isrestored by spl i t t ing out H+ from the carbon which wasattacked.

. ' .6 .2O (a) o - and p-n i t ropheno l(b) o- and p-bromophenol(c) m-dinitrobenzene(d) m-bromonitrobenzene(e) m-nitrobenzoic acid(f) o- and p-bromonitrobenzene(S) o- and p-ni_trotoluene(h) o- and p-xylene

i\Lcoholsr Ethers, Carbonyl Compounds,' ' @

' 6 . 2 L ( a )

( b )

arTf ^ - - CHz - CHs same as (c )

II

Br

II

Br

2 6 . 1 6

2 6 . I 7 ( a )

( b )

( c )

Br, * tTlU

tlt l

_ C - H

CH2 = CH2 + Br2 + CH2Br - CH2Br

C H = C H + 2 H B r + C H g - C H B r 2( d ) c H 3

( e j U n 3

U

tlC

- O - C - C H 3t ll l

- O - C H 2 - C H 3

-. v..¿ -^.<r

CHz CHz\ . /

u n - u n

CH2-CH2

, , , ^ H 2 S O ¡ ^ , , / \+ H2O --

; ""rr ./"r,

CH2 - g¡¡

IOH

( f )

f^v+

N a '

(s ) cHsI

C H s - C H z - C H z - C H - C H 2

o

I o -26.L8 (a) In an unslnünetr ical addit ion the hydrogen adds E0

the carbon with the more hydrogens already attached+ ^ i +

(b) The hydrogen, as H-, adds f irst, to form acarbocation. The more stable carbocation has the+ charge on the more substi tuted carbon, so the Hatom goes to the less subs t i tu ted carbon, i .e . , thecarbon wj-th the more hydrogens.

ill l

- C _ O H

L62163

ill t

- c -

(h ) 26.25 Hydrogen bonding leads to very strong intermoLecularfo rces in a lcoho1s.

26 .26 (a ) Gr ignard reagent , C4HeMgBr

(b) NaBr and cyanobutane (n-butyl cyanide)

C+HgCN

(c) Br and n-butyl alcohol (I-butanol)

C + H g O H

(d) NaBr and ethyl n-butyl ether

C a H e O C 2 H 5

26.27 (a ) Hz and C3H7O Na+

(b) HzO and CH3 - CH = CHz

(c) H2O and CH3 - CH - O - CH - CHs

t tCHs CHe

(d ) o

tlH2O and CeHs - C - O - CH - CH3

- lcHs

( e )

C r - ' a n d C H 3

2 6 . 2 8 (a*f) Molecules with double or tr iple:bonds arecleaved to the resultant acids or ketones; aLcohol-sand aldehydes are oxidized to the corresponding acids.

(a) acetic acid and butanoic acid(b) acetic acid and propanone (acetone)(c) 2 molecules of propanoic acid(d) propanoic acid(e) propanone (acetone)(f) propanone(9) p-nitrotoluene is oxidized to p-nitrobenzoic acid(h) too mi ld to c leave, p roduc t i s d io l , 2 ,

3-hexanediol( i) under mild. condit ions reaction stops at aldehyde,

propanal

cHs v L ! z

cHs

( i )

OH

Br

26.22 (a ) 4 -methy l -1 -Pentano l(b) 4-methyl-2-Pentanol(c) 2-methYl-2-Pentanol(d) 2-methyl-3-pentanone(e) ethyl isoProPYl ether(f) 4-methylpentanal(g) methyl PhenYl ketone(h) cyclohexanone(i) methyl benzoate/- i \ *-- i +-obenZOic aCid\ J ¡ ' I r ¡ ¡ ¿ u r v

26.23 Four a lcoho ls and th ree e thers

CaHeOH C2H5 - CH -

IOH

C H ¡ - C H - C H 2 O H

IcHs

C H g - O - C H - C H s

IcHs

v L t z v r ¡ 5

CHs (d , 1 pa i r )

OH

IC H e - C - C H 3

IcHe

C z H s - O - C 2 H 5

t lt lil

C H s - O - C ¡ H z

26.24 (a) methyl propanoate; propyl formate(b) methyl propyl etheri methyl isopropyl(c) propanal(d) 2-butanone(e) 2-pentanol, 3-Pentanol(f) methyl proPanoate

Many o thers a re Poss ib le .

ether

t64 165

26.29 (a ) l -pentano l(b) 2-pentanol(c) pentane(d) f irst 2-pentene' then pentane

26.30 (a ) seo and e thy l b romide

(b) ethane and MgBr2

(c) MaBr and propanoic acid

26.3I (a) (CHg)z CItCOO-l¡a+ sodium 2-methylproPanoate

(b) (cHs) z cH-oH' isopropyl alcohol

(c) ceHz Coo Na- sodium butanoate

(d) CaHz Coo Na+ and Ho-CH2-CH3, ethanol

26 .32 (a ) p ropano l (b ) 3 -hexano l

(c ) 3-methyl-3-pentanol

2 6 . 3 3 ( a ) o H C H s

t t

H2 SOa( c ) 2 C H 3 C H 2 O H 7 + . H z O + ( C H g C H z ) z O

medrum temp.

(d)

( e )

( f )

(s)

( h )

f rom (a) cHc=cHe reduc t ion . ñu^ .H^wTffi-rr-+

un3Ln3

cHgcHzoH strong ox'

> cH3 cooH

CHgCHzOH + HBr + HzO + CH3CH2BT

from (f) CH3-CH2Br*NaCN + CHs-CNz-CN+NaBr

Mg B r * CH3 CHO + CHg-CH-CH-CHs-."

+ (CHs ) zCHCHO /

OH

I+ CH3CH2MgBT + CHg - C - CHz - CHs

ICH¡

from (b) CH3CHO + HCN

o H ot l

cHs-éri-c-o-Na+ strong+ cHa

:id

26.35 CH3CH2CH2OH + HBr + CH3CHzCH2BT

CH3CH2CH2BT + Mg + CHsCHzCH2M9BT

similarly prepare (CH3) 2CHI49Br

OH

I+ C H s - C H

OH

I+ C s H z - C - C H 3

; , ,. u n 3

base- ur\ --_+hydrolysis

OH

I- cH - cooH

(CHg ) z CH

or CH3 Mg Br

( b )

(CHg ) z CO

( c ) HII

v r ¡ J v t t z

II

C H a

C H 3 C H 2 C H - C H 2 - O HII

c H g

^I¡ l

M g B r + C - H - )

H

mr_Id ox .url3Ur12(-r12rjr1 --> L.n3\-n2Lnu

similarly prepare CH3COCH3 (acetone)

(a) CHgCHzCHo + CHgCHzCHzBT - ) (a f te r hydro lys is )

CH gCHeCHOHCHzCHzCHg

(b ) oH

I(CHg)zCH Mg B r + CH3COCH3 + CH3 - CH - C - CH3

I ICHg CHs26.34 (a) cnecnzoH

H'sol Hzo + cH2 = g¡ l t

high temp.( c )

C3HTM9Br + CH3COCH3(b) cHacHzoH *ild o";

cH3cHo (dist.irt the product to

avoid further oxidation)

L66 L67

l

(d ) (CH3)2CHt{gBr * CH3CHzCHO + CH3-CH2-CH-CH-CH3

t lOH CH3

z o . J o

H2 SOaCHgCHzCH2OH --+

high temp.CH3CH = CHz

CH3CH = CHe + HBr + CH3CHBTCHg

cH3cHBrCH3 + NaOH + CH3CHOHCH3 * NaBr

26-37 (a) The alkyl hal ide ionizes to a sl ight, extent togive a hal ide ion and a carbocation which is apowerful Lewis aci.d. that displaces a proton fromwater.

(b) A Lewis base such as a bromide ion displacesthe OH- ion from the alcohol.

(c) HCN ionizes to a sl ight extent and the CNa strong Lewis base, joins the carbon of thegroup. The excess electron is then local_izedoxygen which becomes negatively charged and abase and picks up the hydrogen ion (1eft fromion iza t ion o f HCN) .

Amides

2 6 . 3 8 ( a )

NIH

( b ) CH3NHC2H5

( c ) H 2 N C 5 H 1 2 N H 2

( d )

- NHz

( e )

NO^L

26 .39 P r ima ry am ines ¡

CaHeNH2 (CH3 ) 2CH CH2NH2 (CHg ) g CNHz

C H 3 C H 2 - C H - N H zII

cHe

Secondary amines: C3H7 NH CH3

(C2H5) 2NH (CHs) z CHNHCH3

Ter t ia ry amine (CHg)z NC2H5

26.40 (a ) methy l e thy l amine ' o thers

2 6 . 4 1

(b) one HzNCHzCooH isomer is 2-hydroxyacetamide

HOCHzCONHz. Another is nítroethane C2H5NO2.

(c) N-methyl acetamide CHg - I

- NH - CHs

o

(a) ethylamine CeHsNHz

(b) p - to lu id i -ne , i .e . , p -aminoto luene ' o r

p-methylani l ine

HeN{( )F",V/

(c) HBr and methyl butyl amine

(d ) ot lt l

= n a { - ¡ m i Á a ¡ T ] ^ - C - . N H Z

ion,

on th€stronEthe

Ami-nes

cIo

168 L69

(e) HN+er and CHgCHzCOOH

(f) diethylammonium bromide

(g) base hydrolYsis Yields

( c z H s ) e N H e * B r

CHgCHzCOOH

Pol)¡mers

26"47 (a) Addit ion pol lzmersopens up and joins one(vinyl chloride) is an

are formed when the double bondmolecule to another. Polyexample:

C H z - C H - C H 2 -

Ic1

(h) neutral izat ion yields water' Cl and

ethylamine¡ C2H5NH2

Applied to amines they count the number of carbon

atoms attached directly to the nitrogen. Applied

to alcohols they count the carbon atoms attached

directly to the carbon which carries the -OH group.

(A) CH¡CHBTCH3 * NH3 + CHS - CHCH3 + HBTII

NHz

(b) CzHsCOOH + NH3 + CzHsCOO NH¡+r

(c ) CzHsCOOCH3 + NH3 + CzHsCONHz + CH3OH

(in aqueous medium one gets hydrolysis to- +

CzHsCOO NH+ + CHgOH)

- C H - C H 2 - C H -

t lcl c l

26 .42

2 6 . 4 3

2 6 . 4 4 C H 3 C H 2 O H 2

CHaCHeOH r

26 .45 CH3CH2O ¡

(b) CoBolymers are formed \,/hen two differentunsaturated compounds add. Butadiene-styrenerubber is an example:

¡-CH? - CH = CH - CHn - CH - CH^t| ' "

| ¿ l

r r A ) ri l t ) l I

i repeat unit \2 i

(c) Cond.ensation polymers are formed by condensatj-onreactions, including such compounds as polyesters andpolyamides. Nylon 66 is an example of a polyamide:t ,

H3o* , cH3cooH. H2o

CHsCHzNHz

OH ¡ CH3CH2NH2¡ CH3COO r CHTCHzOH

O Hi l ic - N -

H Ol : ÍN l ' C

II

O Hl l l- c r * H g - c - N

H Ol r I- N ¡ c

II

- c4Hg -- aoEtz -

26.46 (a) The conjugate base of phenol is highly stabi l ized

by resonance (see structures top of page 695) '

(b) The unshared pair of electrons on the nitrogen in

ani l ine is " less avai lable" because i t is withdrawn

into the ring somewhat as suggested by the resonanc€

forms:

repeat unit

26.4A In a Fr iede l -Cra f ts reac t ion the A1C13 is a Lewisacid whi-ch adds a hal ide and generates a reactivecarbocation. In addit ion polymerization AlC13 is aLewis acid which adds to an olef in local izing anegative charge on the AL and generating a carbocationat the other end of the mo1ecu1e, which is propagatedat the end of the growing chain.

26 .49 (a ) E thy lene, o r e thene (b ) te t ra f luoroe thene(c) styrene (phenylethylene, or vinyl benzene)(d) vinyl cyanide (or cyanoethylene, or a-crylonitr i le)(e) adipic acid and 1, 6-diamino-hexane (see problem26.4 '7c) ( f ) i soprene (o r 'methy lbu tad iene)(9) phenol and formaldehyde (h) terephthalic acid

and the ethylene glycol. '

t lI",

a@N*, t

ll"t

fis**"'ñ\ r ' V

L70 L7T

--¡5

2 6 . 5 0cHz

cF2

CH

(a)

(b)

(c )

c 7 d z - c H 2 + c H 2

CF, - CF, r- CF,L L t -

c H 2 - f H t',A''lt ( ) t l\ 7 r

c H ^ - c n +' l lc N l

- cH2-

- cF2-

Amino Acids, proteins

2 7 . L(d) - c}lz -

( e ) S e e 2 6 .

(f) 1-.", -

I

(e)

C = C H - C H o -

J",f c H r -

c H lt lC N I

47 ( c )

C =CH

IcH3

OH

- cH2

cH. ol J I

n - ó H - ¿ -IH

cH. ol J f

H z N - ó H - { ! i

OH

cHz+A

I \ /cfz

(h) I ---., f, fl nf ?lo-.,",-o-'ó{O>bio-c2lt4-o- b<( ))-,-', . u \\__Z \_ /t \ - + r l -

t l

27'2

fl ,,* + flH2N - f r

- b - oH H ' , ' " rñ - cH _ [ - o "CHr-óH-r:r+ \ - |

I

"':

\

cH3-cH-cH3o

o r i+ l l "

H3N - cH - [ - o- H2N - cH - c - o-

, .u- cHr_éu_cH,

cH3-CH_CH3

27.3 There is no chiral carbon atom in glycine so thereis no optical isomerism.

Primary: the sequence of amino acid resid.ues.

Secondary: the local structural_ confj-guration of thec h a i n ( e . 9 . , a l p h a h e l j . x ) .

f", I Í"r flH 2 N - ó - b - u - c H - b - o H

H

O H H T/

repeaü of the above reactionpept ide (shown on page 7Sg).

oH -' H2o +

the dlpept ide. A

gives the tri-

L72 r73

Tert iary: the overal l configuration of the chain;how it is folded.

Quaternary: how several chains (not connected bycovalent bonds) are bonded to form one functj-onalun i t .

Secondary structure depends primarily on hydrogenbonding. Tert iary structure, in addit ion to hydrogenbonding, uses salt formation

+ -/ ñ i r ! '

'

(R-NH3 OOC-R' ) , d isu l f ide l inks (R-S-S-R ' ) ,and van der Waals forces.

1 , o o o , o o o , / l 2 o ! 8 3 3 0 r e s i d u e s

From A and B there are four possible peptides AA,AB, BA, and BB. Note that AB is NOT the same as BA.For tr ipeptides, you can f i l l each of these slots intwo ways: 2 x 2 x 2 = B possible tr ipeptides. Inthe general caser lol l can f i l l each of y slots in x

Vways gj-ving x' possibi l i t ies.

The + sign means that r ight handed rotat ion isobserved for the compound., whereas D slmbolizes theabsolute configuration in accordance with theconvention shown at the top of page 713.

(a) CH3 - CHz - CHz - CH3 vs. CHs - CH - CH3II

cHs

( b ) C z H s - C H - C 2 H 5 v s . C H 3 - C H - C 3 H 7

t lCHs CHa

cH¡ - - OH vs. i ts mirror imaqe

27.LO The chiral carbon is shown by an asterisk.

zHsIc

IH

( e )

t . 5

( a ) H ( b )

IC H g - C * - C O O H

II

UN

HO-CHzCOOH

none possible

f . o

t . 7 (c) CHs - CCIBr - CH¡ none possible

(d) c1 (e )I

C H s - C * - H C H e -

IBr

Carbohydrates

27.LL A carbohydrate is a polyhydroxywith ether, aldehyde, or ketoneformula CH2O¡ or close to it.

Cz Hs

alcohol, frequentlygroups, with empi-r ical

OH

Ic * -I

CN

. B

27.L2 A f ive or six carbon carbohydrate is a monosaccharide,also ca11ed a sugar. An ether formed from two sugarsis a disaccharide. A polyether formed from a sinqletype of sugar (glucose) is a polysaccharide.

II

OH

*- u

II

OH

*

ItOH

2 7 . L 3 C H z -

IOH

2 7 . L 4 C H z -

IOH

The middle carbon is chiral;a d and 1 pair exists.

carbons, 4 J- Somers

one chiral carbon, 2 isomers

( c ) C H e - O - C H 3

(d) t t

1I

** C H - C H O

II

OH 2 chiral

e v r r z

i l li l |

o o Hcf

v s . C 2 H 5 O H

c1. / "

II

OH

L74L75

' a ts and Oi ls

i7 .15 C1 TCOOCH2

ICr zCOOCH

+3NaOH->

C1 TCOOCH2

7 - L 6 C 3 H 6 = C s H h = C 3 H 4 =

C 3 H 6 = C 3 H a = C 3 H 4 =

C 3 H 5 = C s H 4 = C a H + =

Cr zHa 5COOCH2

ICr zHe 5COOCH

ICr zHg 5COOCH2

7 - L 7 C r T H z 3 C O O C H z

ICTTHz3COOCH + 54 /02 +

I

3C1 TCOOT

at r^ - ñHv L t z

t lt lt l

OH OH

CaHr s COOCH2

ICeHls COOCH

u 8 n l S L L J L H 2

39CO2 + 37HzO

Ct I Hz 3COOCH2

7.lB The fatty aeids from whích oi ls are derived areunsaturated.

l .Lg Ibtal of 18 carbons; stearic acid.

rcleic Acids

7.2O An alpha hel ix is a protein conformation; a singlestrand held in place by intrachain hydrogen bonding(approximately paral1e1 to the axis). A double hel ixis a DNA conformationi a double strand held in placeby interchain hydrogen bonding (approximatelyperpendicular to the axis).

' .2I Not identical but complementary. tr{here one chainhas A the other has T, and where one has G the otherhas C.

2 7 . 2 2 Call the two complementary chalns X and Y. Theyseparate and X generates a complementary partner Yrwhich must be identical_to Y, while Y generates acomplementary partner X' rrhich must be identical toX. The two new double hel ixes XYr and YXl, areidentical to the original XY.

27.23 A mutation is a mistake in the sequence of nucleotidesin a DNA chain.

27 .24 90 ,OOO/L2O = 750 amino ac id res idues . I t requ i res3 bases to code one amino acid, so 3 x 75O x 340= 765,000 molecular ! ' ¡ t . RNA.

27.25 (a) complement of A-c-A is U-C-U

(b) complement of U-C-U on DNA is A-G-A

(c) oppos i te 's t rand is T-C-T

Enz)¡mes and Metabolism

27.26 Turnover nrnnber is the number of t imes per minuteone enzl¡me molecule performs i ts catalyt ic function.

27.2'7 Competit . ive inhibj-tors form a complex wi-th enzyme inequil ibr ium with the free species (as does the sub-strate). Non-competit ive inhibitors form permanent(non-reversible) complexes with the enzl¡me.

27.28 If the product of an enzlmatic reaction is a mildlycompetit ive inhibitor i t wi l l " turn-off" the enzymeand prevent it from producing more product than isneeded. The term "feedback" inhibit ion is used whenit is not the primary product i tself but somesubsequent product which competes with the substratefor the enz!¡me and turns off production of the primaryproduc t .

27.29 The enzyme forms a loose complex with the substratewhich causes a particular bond to become d.istortedand made prone to react with minimal energy ofactivation. They are very specif ic because onl-y thecorrect substrate will fit into the template providedl - r r r ' l - l r a

v ¡ ¡ a f I r v

Tt \q

v t L z

I

I

OH

+ 9H2 ->

L76 r77

3 0

3 1

Catabolic means breaking down, and anabolic meansbuilding up or synthesizing.

Suppose a substance A is to be convertedC which has a higher free energy than A.accomplished i f the reaction A + B -) C +negative AG; such a reaction is ca11ed areaction.

to substanceIt can be

ñ L - ^ ^U ¡ ¡ 4 > 4

coupled

3 2 The substance B in the preceding example j_s a "high_energy" molecule. I ts spontaneous reaction to lowenerg:y products can be coupled to a desirable reactionwhich otherwise would be thermodynamically impossible.An example is ATp; in which case "D" represents ADpplus inorganic phosphate.

L t ó